You are on page 1of 207

5/10/2021 print.

html

A cerebellar lesion is most likely to cause hypotonia A gray elevated retina on funduscopic exam is associated with A hemorrhagic stroke in the internal capsule will result in
what? conjugate gaze deviated towards which side of the lesion?
Resembles a peripheral lesion vs central lesion.
Retinal detachment Ipsilateral (due to damage of frontal eye field efferents in the
anterior limb)
Retinal detachment refers to the separation of the layers of the
retina. It usually occurs in people aged 40-70 years. Patients Putaminal hemorrhage almost always involves the adjacent
complain of photopsia (flashes of light) and floaters (spots in the internal capsule. This leads to contralateral hemiparesis and
visual field). The most classic description is that of "a curtain hemianesthesia (due to disruption of the corticospinal and
coming down over my eyes." Usually, the inciting event occurs somatosensory fibers in the posterior limb) and conjugate gaze
months before retinal detachment. Myopia or trauma can cause deviation toward the side of the lesion (due to damage of frontal
retinal breaks, through which fluid seeps in and separates the eye field efferents in the anterior limb).
retinal layers. Ophthalmoscopic examination reveals a grey,
elevated retina. Laser therapy and cryotherapy are done to create
permanent adhesions between the neurosensory retina, retinal
pigment epithelium, and choroid.

file:///C:/Users/vikas/OneDrive/Desktop/print.html 1/207
5/10/2021 print.html

A hemorrhagic stroke in which area of the brain is associated A high-stepping gate is most commonly caused by radiculopathy A lesion in the vertebrobasilar system (eg, midbrain) presents
with pinpoint pupils? affecting which spinal level or nerve? with ipsilateral cranial nerve involvement and contralateral
hemiplegia.
Pons Level: L5
Nerve: common peroneal A detailed history and neurologic examination of a patient who has
Due to loss of the sympathetics. a stroke can often localize the region of brain dysfunction and
High-stepping or "steppage" gait is due to right foot drop. Foot affected vasculature. The blood supply of the brain is summarized
drop is due to weakness in foot dorsiflexion. To compensate for as follows: anterior vasculature is comprised of the internal carotid
this, patients must flex the hip and knee to raise the foot and avoid artery and its branches, especially the paired anterior and middle
dragging the toe with each step. L5 radiculopathy is typified by cerebral arteries; posterior circulation is comprised of the paired
back pain radiating to the foot, often with weakness of foot vertebral arteries, which unite to form the basilar artery that then
inversion and plantar flexion. Peroneal neuropathy is due to further divides into the paired posterior cerebral arteries.

file:///C:/Users/vikas/OneDrive/Desktop/print.html 2/207
5/10/2021 print.html
compression of the nerve at the lateral aspect of the fibula (eg, due
to prolonged crossing of the legs or squatting) and is associated Anterior cerebral artery (ACA) stroke is characterized by
with paresthesias and sensory loss over the dorsum of the foot but contralateral motor and/or sensory deficits, which are more
normal foot inversion and plantar flexion. The diagnosis can be pronounced in the lower limb than in the upper limb. Other
clarified with electromyography and nerve conduction studies. features may include urinary incontinence (from damage to the
cortical micturition centers of the mesial frontal lobe), gait
The common fibular nerve, also called the common peroneal dyspraxia, primitive reflexes (eg, grasp, sucking), abulia, and
nerve, travels near the fibular head and is susceptible to emotional disturbances.
compressive injuries from leg immobilization (eg, cast, bedrest),
prolonged leg crossing, or protracted squatting. Manifestations are Lacunar infarcts (small-vessel infarcts typically affecting the deep
typically transient and reflect impaired functioning of the fibular subcortical structures) are caused by occlusion of a single, deep-
nerve as follows: unilateral foot drop; numbness/tingling over the penetrating branch of a large cerebral artery. They have well-
dorsal foot and lateral shin; impaired ankle dorsiflexion (walking recognized syndromes: pure motor hemiparesis, pure sensory
on heels) and great toe extension; and preserved plantar flexion stroke, sensorimotor, dysarthria-clumsy hand, and ataxic
(walking on toes) and reflexes. hemiparesis. The face, arm, and leg are all equally affected with a
sensorimotor lacunar stroke, and there are no associated cortical
Diagnosis is typically made with electromyography and nerve symptoms (ie, no urinary incontinence).
conduction studies. Treatment has limited effect but includes
reducing pressure on the nerve (eg, avoiding crossing the legs), an A middle cerebral artery (MCA) stroke is characterized by
ankle-foot orthosis splint, and physical therapy. contralateral motor and/or somatosensory deficits (more
pronounced in the face or upper limb than lower limb), and
homonymous hemianopia or quadrantanopia. If the dominant lobe
(left) is involved, a patient may have aphasia; if the nondominant
lobe (right) is involved, a patient may have hemineglect or
anosognosia (lack of awareness regarding one's illness).

Occlusion of the internal carotid artery, which supplies the anterior


circulation, generally results in extensive neurologic deficits as
both the MCA and ACA territories are affected. Symptoms include
dense, contralateral hemiplegia (face, arm, and leg equally
affected) with contralateral sensory, visual, language, or spatial
impairments.

A posterior cerebral artery stroke is characterized by homonymous


hemianopia, alexia without agraphia (dominant hemisphere),
visual hallucinations (calcarine cortex), sensory symptoms
(thalamus), third nerve palsy with paresis of vertical and horizontal
eye movements, and contralateral motor deficits (cerebral
peduncle, midbrain).

file:///C:/Users/vikas/OneDrive/Desktop/print.html 3/207
5/10/2021 print.html

A lesion in which hemisphere is most commonly associated with A lesion in which part of the brain is most commonly associated A patient with a middle cerebral artery stroke will present with
aphasia? with hemineglect syndrome? contralateral hemiparesis, contralateral hemisensory loss, aphasia,
and visual-spatial deficits.
Left (commonly the dominant side) Right parietal lobe (commonly the non-dominant side)
file:///C:/Users/vikas/OneDrive/Desktop/print.html 4/207
5/10/2021 print.html
A detailed history and neurologic examination of a patient who has
Aphasia is loss of ability to understand or express speech. Due to The parietal lobes process and interpret visual, auditory, and motor a stroke can often localize the region of brain dysfunction and
left MCA lesion. signals received from other brain areas. Damage results in affected vasculature. The blood supply of the brain is summarized
difficulties with spatial and visual perception. The patient neglects as follows: anterior vasculature is comprised of the internal carotid
the left side of a space and responds to the stimuli coming only artery and its branches, especially the paired anterior and middle
from the right side. Patients may shave only the right side of their cerebral arteries; posterior circulation is comprised of the paired
face, comb the right side of their hair, and ignore the subject vertebral arteries, which unite to form the basilar artery that then
located in the left side of a space (this is different than the right- further divides into the paired posterior cerebral arteries.
left confusion seen in Gerstmann syndrome, below). Anosognosia
(lack of insight, or profound lack of awareness of an obvious Anterior cerebral artery (ACA) stroke is characterized by
deficit) may also be present. contralateral motor and/or sensory deficits, which are more
pronounced in the lower limb than in the upper limb. Other
Constructional apraxia may be present, which is a motor disorder
features may include urinary incontinence (from damage to the
caused by damage to the brain (specifically the posterior parietal
cortical micturition centers of the mesial frontal lobe), gait
cortex), in which the individual has difficulty with the motor
dyspraxia, primitive reflexes (eg, grasp, sucking), abulia, and
planning to perform tasks or movements when asked, provided that emotional disturbances.
the request or command is understood and he/she is willing to
perform the task. Asking a patient to fill in the numbers of a clock Lacunar infarcts (small-vessel infarcts typically affecting the deep
is a typical test that is used to detect hemi-neglect syndrome. This subcortical structures) are caused by occlusion of a single, deep-
syndrome is caused by the lesion of the right (non-dominant) penetrating branch of a large cerebral artery. They have well-
parietal lobe, which is responsible for spatial organization. recognized syndromes: pure motor hemiparesis, pure sensory
stroke, sensorimotor, dysarthria-clumsy hand, and ataxic
Dominant (left)-sided parietal lobe lesions result in Gerstmann hemiparesis. The face, arm, and leg are all equally affected with a
syndrome (eg, right-left confusion, agraphia, acalculia). A frontal sensorimotor lacunar stroke, and there are no associated cortical
cortex lesion results in hemiparesis (with motor aphasia if the
symptoms (ie, no urinary incontinence).
dominant lobe is involved). Left temporal lobe involvement leads
to receptive aphasia.
A middle cerebral artery (MCA) stroke is characterized by
contralateral motor and/or somatosensory deficits (more
pronounced in the face or upper limb than lower limb), and
homonymous hemianopia or quadrantanopia. If the dominant
lobe (left) is involved, a patient may have aphasia; if the
nondominant lobe (right) is involved, a patient may have
hemineglect or anosognosia (lack of awareness regarding one's
illness).

Occlusion of the internal carotid artery, which supplies the anterior


circulation, generally results in extensive neurologic deficits as
both the MCA and ACA territories are affected. Symptoms include
dense, contralateral hemiplegia (face, arm, and leg equally
affected) with contralateral sensory, visual, language, or spatial
impairments.

A posterior cerebral artery stroke is characterized by homonymous


hemianopia, alexia without agraphia (dominant hemisphere),
visual hallucinations (calcarine cortex), sensory symptoms
(thalamus), third nerve palsy with paresis of vertical and horizontal
eye movements, and contralateral motor deficits (cerebral
peduncle, midbrain).

file:///C:/Users/vikas/OneDrive/Desktop/print.html 5/207
5/10/2021 print.html

file:///C:/Users/vikas/OneDrive/Desktop/print.html 6/207
5/10/2021 print.html

A patient with an anterior cerebral artery stroke will present with A patient with an posterior cerebral artery stroke will present with A(n) hordeolum (stye) is an acute inflammatory disorder of the
contralateral motor and sensory lower extremity deficits. vision deficits. eyelash follicle or tear gland and presents as an erythematous,
tender nodule at the lid margin.
A detailed history and neurologic examination of a patient who has A detailed history and neurologic examination of a patient who has
a stroke can often localize the region of brain dysfunction and a stroke can often localize the region of brain dysfunction and A hordeolum is an acute inflammatory disorder of the eyelash
affected vasculature. The blood supply of the brain is summarized affected vasculature. The blood supply of the brain is summarized follicle or tear gland and presents as an erythematous, tender
as follows: anterior vasculature is comprised of the internal carotid as follows: anterior vasculature is comprised of the internal carotid nodule at the lid margin. It is often due to infection with
artery and its branches, especially the paired anterior and middle artery and its branches, especially the paired anterior and middle Staphylococcus aureus but can be sterile in many cases. A similar
cerebral arteries; posterior circulation is comprised of the paired cerebral arteries; posterior circulation is comprised of the paired process arising in the meibomian gland (internal hordeolum)
vertebral arteries, which unite to form the basilar artery that then vertebral arteries, which unite to form the basilar artery that then presents as a tender nodule visible at the palpebral conjunctiva but
further divides into the paired posterior cerebral arteries. further divides into the paired posterior cerebral arteries. is less common. Within a few days, a minute pustule may appear at
the lid margin (pointing), which will then rupture with discharge of
file:///C:/Users/vikas/OneDrive/Desktop/print.html 7/207
5/10/2021 print.html
Anterior cerebral artery (ACA) stroke is characterized by pus and relief of pain.
Anterior cerebral artery (ACA) stroke is characterized by contralateral motor and/or sensory deficits, which are more
contralateral motor and/or sensory deficits, which are more pronounced in the lower limb than in the upper limb. Other Warm compresses are advised to accelerate the process. Following
pronounced in the lower limb than in the upper limb. Other features may include urinary incontinence (from damage to the resolution of infection, some patients have a residual
features may include urinary incontinence (from damage to cortical micturition centers of the mesial frontal lobe), gait granulomatous nodule (chalazion) that regresses slowly over
the cortical micturition centers of the mesial frontal lobe), gait dyspraxia, primitive reflexes (eg, grasp, sucking), abulia, and several months. For patients with a persistent hordeolum (eg, >1-2
dyspraxia, primitive reflexes (eg, grasp, sucking), abulia, and emotional disturbances. weeks) or a large chalazion, additional management options
emotional disturbances. include incision and curettage. Topical antibiotics are often
Lacunar infarcts (small-vessel infarcts typically affecting the deep prescribed for hordeolum but are usually unnecessary.
Lacunar infarcts (small-vessel infarcts typically affecting the deep subcortical structures) are caused by occlusion of a single, deep-
subcortical structures) are caused by occlusion of a single, deep- penetrating branch of a large cerebral artery. They have well-
penetrating branch of a large cerebral artery. They have well- recognized syndromes: pure motor hemiparesis, pure sensory
recognized syndromes: pure motor hemiparesis, pure sensory stroke, sensorimotor, dysarthria-clumsy hand, and ataxic
stroke, sensorimotor, dysarthria-clumsy hand, and ataxic hemiparesis. The face, arm, and leg are all equally affected with a
hemiparesis. The face, arm, and leg are all equally affected with a sensorimotor lacunar stroke, and there are no associated cortical
sensorimotor lacunar stroke, and there are no associated cortical symptoms (ie, no urinary incontinence).
symptoms (ie, no urinary incontinence).
A middle cerebral artery (MCA) stroke is characterized by
A middle cerebral artery (MCA) stroke is characterized by contralateral motor and/or somatosensory deficits (more
contralateral motor and/or somatosensory deficits (more pronounced in the face or upper limb than lower limb), and
pronounced in the face or upper limb than lower limb), and homonymous hemianopia or quadrantanopia. If the dominant lobe
homonymous hemianopia or quadrantanopia. If the dominant lobe (left) is involved, a patient may have aphasia; if the nondominant
(left) is involved, a patient may have aphasia; if the nondominant lobe (right) is involved, a patient may have hemineglect or
lobe (right) is involved, a patient may have hemineglect or anosognosia (lack of awareness regarding one's illness).
anosognosia (lack of awareness regarding one's illness).
Occlusion of the internal carotid artery, which supplies the anterior
Occlusion of the internal carotid artery, which supplies the anterior circulation, generally results in extensive neurologic deficits as
circulation, generally results in extensive neurologic deficits as both the MCA and ACA territories are affected. Symptoms include
both the MCA and ACA territories are affected. Symptoms include dense, contralateral hemiplegia (face, arm, and leg equally
dense, contralateral hemiplegia (face, arm, and leg equally affected) with contralateral sensory, visual, language, or spatial
affected) with contralateral sensory, visual, language, or spatial impairments.
impairments.
A posterior cerebral artery stroke is characterized by
A posterior cerebral artery stroke is characterized by homonymous homonymous hemianopia, alexia without agraphia (dominant
hemianopia, alexia without agraphia (dominant hemisphere), hemisphere), visual hallucinations (calcarine cortex), sensory
visual hallucinations (calcarine cortex), sensory symptoms symptoms (thalamus), third nerve palsy with paresis of vertical
(thalamus), third nerve palsy with paresis of vertical and horizontal and horizontal eye movements, and contralateral motor
eye movements, and contralateral motor deficits (cerebral deficits (cerebral peduncle, midbrain).
peduncle, midbrain).

file:///C:/Users/vikas/OneDrive/Desktop/print.html 8/207
5/10/2021 print.html

file:///C:/Users/vikas/OneDrive/Desktop/print.html 9/207
5/10/2021 print.html

AST can rise in Abulia is associated with a lesion in which cerebral artery? Acquired causes of ataxia include alcohol intoxication,
Vitamin B12 or Vitamin B1 deficiency, cerebellar damage,
Skeletal muscle injuries Anterior cerebral artery demyelinating disease, and tabes dorsalis.
Abulia is a lack of will or initiative. Tabes dorsalis = tertiary syphilis
A detailed history and neurologic examination of a patient who has
a stroke can often localize the region of brain dysfunction and
affected vasculature. The blood supply of the brain is summarized
as follows: anterior vasculature is comprised of the internal carotid
artery and its branches, especially the paired anterior and middle
cerebral arteries; posterior circulation is comprised of the paired
vertebral arteries, which unite to form the basilar artery that then
further divides into the paired posterior cerebral arteries.

Anterior cerebral artery (ACA) stroke is characterized by


contralateral motor and/or sensory deficits, which are more
pronounced in the lower limb than in the upper limb. Other
features may include urinary incontinence (from damage to the
cortical micturition centers of the mesial frontal lobe), gait
dyspraxia, primitive reflexes (eg, grasp, sucking), abulia, and
emotional disturbances.

Lacunar infarcts (small-vessel infarcts typically affecting the deep


subcortical structures) are caused by occlusion of a single, deep-
penetrating branch of a large cerebral artery. They have well-
recognized syndromes: pure motor hemiparesis, pure sensory
stroke, sensorimotor, dysarthria-clumsy hand, and ataxic
hemiparesis. The face, arm, and leg are all equally affected with a
sensorimotor lacunar stroke, and there are no associated cortical
symptoms (ie, no urinary incontinence).

A middle cerebral artery (MCA) stroke is characterized by


contralateral motor and/or somatosensory deficits (more
pronounced in the face or upper limb than lower limb), and
homonymous hemianopia or quadrantanopia. If the dominant lobe

file:///C:/Users/vikas/OneDrive/Desktop/print.html 10/207
5/10/2021 print.html
(left) is involved, a patient may have aphasia; if the nondominant
lobe (right) is involved, a patient may have hemineglect or
anosognosia (lack of awareness regarding one's illness).

Occlusion of the internal carotid artery, which supplies the anterior


circulation, generally results in extensive neurologic deficits as
both the MCA and ACA territories are affected. Symptoms include
dense, contralateral hemiplegia (face, arm, and leg equally
affected) with contralateral sensory, visual, language, or spatial
impairments.

A posterior cerebral artery stroke is characterized by homonymous


hemianopia, alexia without agraphia (dominant hemisphere),
visual hallucinations (calcarine cortex), sensory symptoms
(thalamus), third nerve palsy with paresis of vertical and horizontal
eye movements, and contralateral motor deficits (cerebral
peduncle, midbrain).

file:///C:/Users/vikas/OneDrive/Desktop/print.html 11/207
5/10/2021 print.html

Acute angle-closure glaucoma is characterized by a(n) sudden Acute angle-closure glaucoma presents with a unilateral red, Acute insomnia is commonly caused by psychological stress or jet
increase in intraocular pressure and may be precipitated by painful, rigid eye(s) that is (are) dilated and nonreactive; headache; lag. Chronic insomnia is typically secondary to psychiatric
prolonged dilation of the iris. and blurry vision that resembles "halos." conditions, medications/substances, or medical problems.

Anterior angle chamber: area between the cornea and the iris Acute angle-closure glaucoma (ACG) is due to sudden narrowing
where the trabecular meshwork is located or closure of the anterior chamber angle. In patients with ACG, the
lens is located anatomically more forward and rests against the iris.
After a patient has spent a prolonged period in low light situations This partially covers the anterior chamber angle and prevents
(i.e. a movie theater) the iris dilates, blocking the aqueous humor normal flow of aqueous humor (through the pupil and into the
outflow from the anterior chamber, which suddenly spikes IOP. anterior chamber), thereby increasing intraocular pressure. Sudden
angle closure typically occurs as a response to pupillary dilation
Acute angle-closure glaucoma (ACG) is due to sudden narrowing from medications (eg, anticholinergics such as tolterodine,
or closure of the anterior chamber angle. In patients with ACG, the sympathomimetics, and decongestants) or other stimuli (eg, dim
lens is located anatomically more forward and rests against the iris. light).
This partially covers the anterior chamber angle and prevents
normal flow of aqueous humor (through the pupil and into the Acute ACG is more common in women (especially age >40),
anterior chamber), thereby increasing intraocular pressure. Sudden Asian and Inuit populations, and individuals with farsightedness.
angle closure typically occurs as a response to pupillary dilation Patients typically develop rapid onset of severe eye pain and may
from medications (eg, anticholinergics such as tolterodine, see halos around lights. The affected eye will appear injected and
sympathomimetics, and decongestants) or other stimuli (eg, dim the pupil will be dilated and poorly responsive to light. Patients
light). can also develop tearing and severe headache ("never had a
headache like this before") with subsequent nausea and vomiting
Acute ACG is more common in women (especially age >40), as the intraocular pressure increases. Untreated acute ACG can
Asian and Inuit populations, and individuals with farsightedness. lead to severe and permanent vision loss within 2-5 hours of
Patients typically develop rapid onset of severe eye pain and may symptom onset.
see halos around lights. The affected eye will appear injected and
the pupil will be dilated and poorly responsive to light. Patients
can also develop tearing and severe headache ("never had a
headache like this before") with subsequent nausea and vomiting
as the intraocular pressure increases. Untreated acute ACG can

file:///C:/Users/vikas/OneDrive/Desktop/print.html 12/207
5/10/2021 print.html
lead to severe and permanent vision loss within 2-5 hours of
symptom onset.

file:///C:/Users/vikas/OneDrive/Desktop/print.html 13/207
5/10/2021 print.html

Age-related macular degeneration is characterized by painless Age-related macular degeneration is characterized by painless
progressive bilateral loss of central vision and shows drusen in the progressive bilateral loss of central vision and shows drusen in the
retina when dry and blood/fluid in the retina when wet. retina when dry and blood/fluid in the retina when wet.
After the initial presentation of an ischemic stroke, what Risk factors: ↑age, female, Caucasian race, smoking, HTN, and Risk factors: ↑age, female, Caucasian race, smoking, HTN, and
additional tests should you do to assess for a cause? family history. family history.
Wet ARMD (20% of cases): abnormal vessel formation under the Wet ARMD (20% of cases): abnormal vessel formation under the
ECG to assess for atrial fibrillation retina → leakage of serous fluid and blood retina → leakage of serous fluid and blood
Echo to assess the cardiac valves Dry ARMD (80% of cases): atrophy and degeneration of the Dry ARMD (80% of cases): atrophy and degeneration of the
Carotid duplex ultrasound to assess for carotid stenosis central retina → drusen (yellowish-white deposits) under the central retina → drusen (yellowish-white deposits) under the
pigment epithelium pigment epithelium

Macular degeneration is a painless, progressive loss of central Macular degeneration is a painless, progressive loss of central
vision that typically occurs in older patients, especially those with vision that typically occurs in older patients, especially those with
vascular risk factors. Both eyes are often affected, and funduscopic vascular risk factors. Both eyes are often affected, and funduscopic
abnormalities (eg, drusen [yellow deposits]) are prominent. One of abnormalities (eg, drusen [yellow deposits]) are prominent. One of
the earliest findings in macular degeneration is distortion of the earliest findings in macular degeneration is distortion of
straight lines such that they appear wavy. The grid test is straight lines such that they appear wavy. The grid test is
frequently used to screen for patients with macular degeneration. frequently used to screen for patients with macular degeneration.
The primary risk factor for macular degeneration is increasing age, The primary risk factor for macular degeneration is increasing age,
although smoking can increase the risk as well. Patients may be although smoking can increase the risk as well. Patients may be
asymptomatic, but others complain of visual problems in either asymptomatic, but others complain of visual problems in either
one or both eyes. Driving and reading are often some of the first one or both eyes. Driving and reading are often some of the first
activities that are affected since they require fine visual acuity, activities that are affected since they require fine visual acuity,
which is provided primarily by the macula. Visualization of which is provided primarily by the macula. Visualization of
straight lines is a task that requires fine visual acuity, which is straight lines is a task that requires fine visual acuity, which is
controlled primarily by the centrally-located macula as opposed to controlled primarily by the centrally-located macula as opposed to
the more peripheral retina. the more peripheral retina.

file:///C:/Users/vikas/OneDrive/Desktop/print.html 14/207
5/10/2021 print.html

Alcoholic cerebellar degeneration is a(n) irreversible condition


that damages Purkinje cells.

Alcoholic cerebellar degeneration presents in heavy alcoholics


with slowly progressive postural gait dysfunction. Cerebellar
degeneration typically occurs after >10 years of heavy alcohol use
and is due to the degeneration of Purkinje cells in the cerebellar
vermis. As these neurons are largely responsible for truncal Allergic conjunctivitis that affects the cornea, producing more
coordination, patients with cerebellar degeneration usually develop severe symptoms, is called atopic keratoconjunctivitis
slowly progressive wide-based gait and postural instability.
Physical examination typically shows evidence of postural Atopic keratoconjunctivitis is a severe form of ocular allergy. The
incoordination (eg, inability to tandem walk, abnormal heel-knee- most common symptoms are itching, tearing, thick mucus
shin testing) with relatively preserved limb coordination (normal discharge, photophobia, and blurred vision. It can be differentiated
finger-nose testing). Cognitive function is usually preserved. from AC by more severe symptoms with a prolonged course,
Although alcohol cessation and nutritional supplementation potential visual impairment due to corneal involvement, and
prevent disease progression, ambulatory difficulties do not thickening of the eyelids and surrounding skin.
typically improve. Patients therefore usually benefit from
ambulatory assist devices such as canes or walkers.

Allergic conjunctivitis is very common in patients with atopic


disease and usually seasonal.

bilateral redness, itching, tearing, and nasal conjunctivitis, eyelid


edema

file:///C:/Users/vikas/OneDrive/Desktop/print.html 15/207
5/10/2021 print.html

Alzheimer disease is a(n) slowly progressive dementia that takes


memory first, then comes back for personality. A CT/MRI will
show atrophy of the temporal and parietal lobes, but is not
necessary to diagnose.
Amaurosis fugax is a temporary unilateral curtain-like loss of
Neuroimaging is typically performed to exclude alternative vision most commonly due to atheroemboli from the ipsilateral
diagnoses such as a lesion (eg, tumor, subdural hematoma, carotid artery
infarction) or normal-pressure hydrocephalus (NPH). In patients
with early AD, MRI may be normal. However, temporal and It is essentially a carotid artery TIA, which can also result in
parietal lobe atrophy, most prominent in the medial temporal lobes temporary loss of speech, paresthesias of contralateral extremities,
and hippocampi, is typically seen in later stages of AD. or clumsiness of one limb.
Atrophy of the temporal and frontal lobes on neuroimaging is Amaurosis fugax is describes as a curtain descending over the
consistent with frontotemporal dementia, which has an earlier age Alzheimer disease is associated with neurofibrillary tangles, visual field. The most common etiology is retinal ischemia due to
of onset than AD (usually age 50-60) and typically presents with neurotic plaques, and amyloid deposition, which are noted on atherosclerotic emboli originating from the ipsilateral carotid
personality changes and loss of social inhibition. Unlike AD, autopsy. artery; therefore, patients with vascular risk factors (eg,
memory impairment is usually mild in the initial stages of the hypertension, hyperlipidemia, smoking) should be evaluated with a
disease. Amyloid angiopathy most commonly affects parietal Senile plaques—focal collections of dilated, tortuous neuritic duplex ultrasound of the neck. Funduscopic examination is often
and occipital lobes. processes surrounding a central amyloid core (amyloid beta- normal but may show embolic plaques and retinal whitening (due
protein) to ischemia). It may show swelling of the optic disk. Patients with
Neurofibrillary tangles—bundles of neurofilaments in cytoplasm amaurosis fugax and concomitant carotid artery disease have an
of neurons that denote neuronal degeneration increased risk of stroke.

Other causes of transient monocular vision loss are uncommon and


include cardioembolic disease, giant cell arteritis, retinal vein
occlusion, retinal vasospasm, and papilledema.

file:///C:/Users/vikas/OneDrive/Desktop/print.html 16/207
5/10/2021 print.html

Amyotrophic lateral sclerosis is a chronic progressive disease


that produces asymmetric upper and lower ( ͡~ ͜ʖ ͡°) motor neuron
Amyotrophic lateral sclerosis in the late-stage is dominated with
lesions. An enlarging blind spot in the location of the optic nerve head
lower motor neuron symptoms and presents with emotional lability
and weight loss. can because by what?
Generally spares the eyes. Look for atrophy and fasciculations of
the tongue and extremities commingled with upward Babinski and Papilledema
hyper-reflexia of the extremities.
Normal individuals have a blind spot in their visual fields at the
location of the optic nerve head; this blind spot enlarges in those
with papilledema. Papilledema can cause momentary vision loss
that varies according to changes in head positioning. It can also
lead rapidly to permanent vision loss and requires urgent
diagnostic evaluation.

An ischemic thrombotic stroke may alternative with periods of Anomic aphasia (inability to speak nouns) is classically Anti parkinsons drugs causing hallucinations
improvement. associated with a lesion in which part of the brain?
Dopamine agoinsts and Levodopa
Ischemic strokes can be thrombotic or embolic in etiology, with Dominant (left) temporal lobe
atherosclerotic-related thrombotic strokes the most common.
However, the presence of an irregularly irregular rhythm in a Dominant parietal lobe lesions can present with contralateral
patient with a history of structural heart disease (severe left atrial sensory loss (eg, pain, vibration, agraphesthesia, astereognosis),
enlargement likely due to poorly controlled hypertension) is Gerstmann syndrome (eg, right-left confusion, agraphia,
strongly suggestive of a cardioembolic etiology in paroxysmal acalculia) and contralateralinferior homonymous quadrantanopsia
atrial fibrillation. Embolic strokes present with a very rapid onset due to superior optic radiations involvement.
of severe focal neurologic deficits. In contrast, an unstable
ruptured atherosclerotic plaque with a propagating thrombus on its Nondominant parietal lobe lesions typically cause anosognosia
surface cause the development of focal neurologic symptoms at a (denial of one's disabilities), hemineglect and contralateral apraxia
relatively slower rate. (inability to carry out learned purposeful movements).

Dominant temporal lobe lesions can affect comprehension


(receptive aphasia), ability to speak nouns (anomic aphasia),
repetition (conductive aphasia) due to arcuate fasciculus
involvement, and contralateral superior homonymous
quadrantanopsia due to inferior optic radiations (Meyer's loop)
involvement.

Nondominant frontal lobe lesions typically affect the way a person


conveys emotion through speech (motor aprosodia), contralateral
weakness, and apraxia.

Nondominant temporal lobe lesions can impair ability to


comprehend emotional gestures (sensory aprosodia). These
patients can also develop a contralateral homonymous
quadrantanopsia due to the inferior optic radiations involvement.

Neurological terms:
Anosognosia: denial of one's disabilities, lack of insight.
Prosopagnosia: inability to recognize the faces of familiar people.
Agraphesthesia: disorientation of the skin's sensation across its
space. It is a difficulty recognizing a written number or letter
traced on the skin after parietal damage.
Astereognosis: inability to identify an object by active touch of the
hands without other sensory input.
Aphasia: inability to understand or express speech, caused by brain
damage.

file:///C:/Users/vikas/OneDrive/Desktop/print.html 17/207
5/10/2021 print.html
Aprosodia: inability to properly convey or interpret emotional
prosody. Prosody in language refers to the ranges of rhythm, pitch,
stress, intonation, etc.
Apraxia: inability to perform particular purposive actions, as a
result of brain damage.

Are hemorrhagic strokes associated with focal neurologic Are patients with chronic back pain more likely to benefit from Are the following associated with follicular or papillary
symptoms? muscle relaxants or tricyclic antidepressants? conjunctivitis?

Intracerebral hemorrhage: yes Tricyclic antidepressants Allergies: papillary conjunctivitis


Subarachnoid hemorrhage: no Bacteria: papillary conjunctivitis
Management of chronic back pain should include an exercise Trachoma: follicular conjunctivitis
Hypertensive intracerebral hemorrhage generally evolves over a program emphasizing stretching and strengthening of the back Viruses: follicular conjunctivitis
course of minutes to hours. Patients initially present with focal muscles and aerobic conditioning. Acetaminophen or nonsteroidal
neurologic symptoms (eg, hemiplegia, hemiparesis, hemisensory anti-inflammatory drugs can be used intermittently. Some patients Trachoma is due to Chlamydia trachomatis serotypes A, B, and C
disturbances). These are quickly followed by findings suggestive may benefit from tricyclic antidepressants or duloxetine, but and is the leading cause of blindness worldwide. C trachomatis
of increased intracranial pressure (eg, vomiting, headache, opioids, benzodiazepines, and muscle relaxants are not advised. spreads effectively in crowded or unsanitary conditions. The active
bradycardia, decreased alertness). In contrast, patients with phase is most common in children and is characterized by
subarachnoid hemorrhage complain of severe headache at the follicular conjunctivitis and pannus (neovascularization) formation
onset and usually do not have focal neurologic findings. Brain in the cornea. There is often a concomitant nasopharyngeal
computed tomography imaging confirms intracranial hemorrhage infection (eg, rhinorrhea, pharyngitis). Repeated or chronic
(both intracerebral and subarachnoid) and excludes other causes. infection leads to scarring of the eyelids and inversion of the
eyelashes (trichiasis). Over time, the lashes rub on the eye and
cause ulcerations and blindness (cicatricial trachoma). The
diagnosis can be made clinically by examination of the tarsal
conjunctivae. C trachomatis may be visible by Giemsa stain
examination of conjunctival scrapings. Oral azithromycin is
effective against C trachomatis; in general, the entire region (eg,
village, refugee camp) should be treated simultaneously. For
individuals with trichiasis, eyelid surgery is needed to preserve
vision.

file:///C:/Users/vikas/OneDrive/Desktop/print.html 18/207
5/10/2021 print.html

file:///C:/Users/vikas/OneDrive/Desktop/print.html 19/207
5/10/2021 print.html

Aseptic meningitis will present under 1000 WBCs that are mostly
macrophages.

It will also have normal glucose and normal protein.

As it is difficult to distinguish aseptic meningitis from acute


bacterial meningitis, treat all equivocal meningitis patients as if
they have acute bacterial meningitis.

Ataxia is characterized by gait instability, balance loss, and


impaired limb coordination.

Autonomic symptoms are common in Lambert-Eaton syndrome. Babesiosis is a tickborne illness that may cause intravascular Bacterial conjunctivitis presents with irritation, hyperemia,
hemolysis and is associated with maltese crosses on peripheral tearing, and mucopurulent exudate with crusting.
blood smear.

file:///C:/Users/vikas/OneDrive/Desktop/print.html 20/207
5/10/2021 print.html
Note: autonomic symptoms are also common in Guillain-Barre Most commonly Staph aureus
syndrome Babesiosis is a tick-borne protozoal illness endemic to the
northeastern United States. Human transmission occurs via Ixodes
Lambert-Eaton syndrome (LE) is a neuromuscular junction scapularis approximately 48-72 hours after attachment. I scapularis
disorder that involves autoantibodies to presynaptic Ca-channels. also may transmit Borrelia burgdorferi (Lyme disease) and
In LE syndrome, weakness progressively improves with Anaplasma phagocytophilum (human granulocytic anaplasmosis);
activity. LE also has autonomic symptoms like dry mouth or therefore, coinfection with multiple organisms is possible. Peak
impotence. LE syndrome starts weak but gets prgoressively prevalence is in July and August, when tick populations are at their
stronger, thus illiciting initial reflexes is diffilcut. In contrast, MG highest.
has muscle weakness that worsens with repetitive motions, but can
illicit strong initial reflexes. Infection is often asymptomatic or mild, but patients with
immunocompromise, age >50, or a history of splenectomy are at
higher risk for severe illness. Symptoms typically include the
gradual onset of fatigue, malaise, weakness, chills, and fever. The
organism multiplies in red blood cells, so patients may develop
anemia with signs of intravascular hemolysis (jaundice, dark urine,
indirect hyperbilirubinemia, reticulocytosis, elevations of
aminotransferases and lactate dehydrogenase). Thrombocytopenia
is common and mild hepatosplenomegaly may occur. Diagnosis is
made by identifying organisms on peripheral blood smear
("Maltese cross"). Treatment includes 7-10 days of atovaquone
plus azithromycin or quinine plus clindamycin (for severe illness).
Symptoms may take up to 3 months to fully resolve.

file:///C:/Users/vikas/OneDrive/Desktop/print.html 21/207
5/10/2021 print.html

file:///C:/Users/vikas/OneDrive/Desktop/print.html 22/207
5/10/2021 print.html

Bacterial meningitis will present on CSF analysis with 1000+


WBCs that are mostly neutrophils.

It will also have low glucose and high protein.

Bell palsy can be treated with a short course of prednisone, as long


as you don't suspect Lyme disease as a cause.
Bell palsy is a hemifacial paralysis due to swelling of CN VII,
The prognosis is very good; 80% of patients recover fully within often preceeded by a(n) upper respiratory infection.
weeks to months. Patient should wear eye patch at night to prevent
corneal abrasion, because the cornea is exposed due to weakness The prognosis is very good; 80% of patients recover fully within
of orbicularis oculi muscle. Surgical decompression of CN VII is weeks to months.
indicated if the paralysis progresses or if tests indicate
deterioration.

Botulism is treated with antitoxin as soon as lab specimens are Botulism presents with symmetric, descending flaccid paralysis Botulism results from ingestion of preformed toxins produced by
obtained. and is diagnosed by identification of the toxin in bioassay. spores of Clostridium botulinum.

file:///C:/Users/vikas/OneDrive/Desktop/print.html 23/207
5/10/2021 print.html
Do not wait for the results. Gastric lavage is helpful only within The severity of illness ranges widely, from mild and self-limiting, Improperly stored food can be contaminated with the spores.
several hours after ingestion of suspected food. For contaminated to rapidly fatal disease. Abdominal cramps, nausea, vomiting, and Toxins can be inactivated by cooking food at high temperatures.
wounds, in addition to antitoxin administration, clean the wound diarrhea are common. Paralysis often starts with a dry mouth, Wound contamination is another source.
and administer penicillin. diplopia, and/or dysarthria.

A bioassay examines serum, stool, or gastric contents.

Foodborne botulism, a potentially life-threatening illness, is caused


by the ingestion of food contaminated with botulinum toxin
formed through germination of Clostridium botulinum spores.
Improperly canned foods and cured fish provide the appropriate
environment for spore germination. Ingestion results in absorption
of a preformed toxin that inhibits presynaptic acetylcholine release
at the neuromuscular junction.

Symptoms develop acutely (within 36 hours) following toxin


ingestion. Prodromal symptoms can include gastrointestinal
discomfort, dry mouth, and sore throat. Patients classically develop
bilateral cranial neuropathies often leading to blurred vision (due
to fixed pupillary dilation), diplopia, facial weakness, dysarthria,
and dysphagia. Symmetric descending muscle weakness usually
follows, initially affecting the upper extremities and trunk and then
the lower extremities. Respiratory failure requiring mechanical
ventilation can occur secondary to diaphragmatic paralysis.

Patients with suspected foodborne botulism should be checked for


toxin in the blood. In addition to supportive care, treatment for
adults and children age >1 involves passive immunity through
administration of horse-derived antitoxin.

file:///C:/Users/vikas/OneDrive/Desktop/print.html 24/207
5/10/2021 print.html

Broca's area is located in the posterior inferior frontal gyrus and Brown-Sequard syndrome is hemisection of the spinal cord, Brown-Sequard syndrome presents with contralateral loss of
when lesioned can result in Broca's aphasia and conjugate gaze usually at the cervical levels, that is most commonly caused by pain/temperature (one or two levels below the lesion), ipsilateral
deviation to the ipsilateral side. trauma. hemiparesis (spastic below the lesion, flaccid at the level of the
lesion), and ipsilateral loss of position/vibration (below the level of
Nearly 95% of right-handed and up to 70% of left-handed people Fracture, stab wound, crush injury, tumor, abscess the lesion).
are left-hemisphere dominant for verbal and written language
function. Patients with lesions affecting the posterior inferior Brown-Séquard syndrome usually occurs when there is an injury
frontal gyrus (Broca's area) can comprehend and follow commands (eg, stab wound, cord compression) of a hemisection of the spinal
but are unable to verbalize or write properly (expressive aphasia). cord. Findings include:
Broca's lesions can also affect the neurologic pathway from that 1. Ipsilateral hemiparesis: Lateral corticospinal tract; occurs at the
brain region to the motor function of the larynx and mouth. level of the cord injury and below
2. Ipsilateral diminished proprioception, vibratory sensation, and
In rare instances, the lesions can extend and affect the arcuate light touch: Dorsal columns; occurs at the level of the cord injury
fasciculus to impair the patient's ability to repeat phrases and below. DCML system decussates at the medulla.
(conduction aphasia). An easy way to remember is that Broca's 3. Contralateral diminished pain and temperature: Lateral
aphasia often represents a broken speech system. Patients with spinothalamic tract (LST); tends to occur 1-2 levels below the cord
injury to the lateral frontal lobe can also develop contralateral injury because the LST decussates (crosses over) 1-2 levels above
weakness of the face and extremities (motor cortex) and conjugate the entry point for the corresponding sensory neuron
gaze deviation to the side of the lesion (contraversive frontal eye
fields). In contrast, patients with Wernicke's aphasia have difficulty When Brown-Séquard syndrome occurs in the cervical region,
comprehending and following commands but are able to speak there can also be associated ipsilateral Horner syndrome (ptosis,
fluently. However, their speech tends to be rambling without miosis, anhidrosis). In addition, there can be absent reflexes
concrete meaning. (including Babinski) initially due to spinal shock, but later,
hyperreflexia and positive Babinski ipsilateral to the
injury. Prognosis is very good.

file:///C:/Users/vikas/OneDrive/Desktop/print.html 25/207
5/10/2021 print.html

file:///C:/Users/vikas/OneDrive/Desktop/print.html 26/207
5/10/2021 print.html

file:///C:/Users/vikas/OneDrive/Desktop/print.html 27/207
5/10/2021 print.html

file:///C:/Users/vikas/OneDrive/Desktop/print.html 28/207
5/10/2021 print.html

CMV retinopathy is painless and presents with blurry vision.

Cytomegalovirus (CMV) is a widely prevalent DNA virus of the


herpes family that typically causes an asymptomatic initial
infection followed by a life-long latent infection. In patients with Cardiac syncope is caused by arrhythmias, mechanical
significantly compromised cell-mediated immunity (eg, advanced obstructions to blood flow, or a massive MI.
HIV with CD4 count <100/mm3), CMV reactivation may result in
viremia or end-organ disease.

CMV retinitis is characterized by full-thickness retinal CSF differenetial


inflammation that moves centripetally along the vasculature,
causing edema and scarring. Patients experience blurred vision,
floaters, and photopsia (sensation of flashing lights). Lesions near
the fovea and optic nerve may cause blindness, and scarring
greatly increases the risk of retinal detachment. Diagnosis is made
by fundoscopy showing yellow-white, fluffy, hemorrhagic
lesions along the vasculature. Blood tests for CMV (polymerase
chain reaction) are not sufficient for diagnosis as viremia may
develop independently of end-organ disease. Patients are usually
treated with oral antivirals (eg, valganciclovir); if lesions are near
the fovea or optic nerve, intravitreal injections are added. All
patients should be initiated on antiretroviral therapy (usually 2
weeks after beginning CMV treatment) to prevent recurrence and
progression.

Carotid artery stenosis is treated with carotid endarterectomy. Carpal tunnel syndrome is caused by median nerve compression Carpal tunnel syndrome is diagnosed with Tinel sign and Phalen
presenting with numbness, pain, or tingling in the first 3.5 digits, test on physical exam, confirmed with nerve conduction studies,
Asymptomatic carotid artery stenosis can progress to transient that spares the palmar portion of the ventral hand. and treated initally with nocturnal wrist splinting, NSAIDs, and
ischemic attack (TIA) or embolic stroke. Symptomatic carotid local corticosteroid injection.
stenosis is defined by the occurrence (within the past 6 months) of Common risk factors include obesity, diabetes, hypothyroidism,
sudden-onset focal neurological symptoms corresponding to a and pregnancy. Although CTS is frequently reported in Hand pain in the distribution of the median nerve is consistent with
carotid artery lesion. occupations requiring prolonged typing or computer work, current carpal tunnel syndrome (CTS). CTS is caused by compression of
evidence for a causative link between CTS and light-duty office the median nerve where it passes under the transverse carpal
Carotid endarterectomy (CEA) is recommended for men and work is conflicting. ligament (flexor retinaculum) in the wrist. Patients have
women with symptomatic carotid stenosis of 70%-99% and is also reproduction of pain on percussion over the median nerve at the
beneficial for men with asymptomatic carotid stenosis of wrist (Tinel sign) or by holding the wrists in full flexion with the
60%-99%. Some experts recommend CEA for asymptomatic dorsum of the hands pressed together (Phalen test). But a negative
women with high-grade stenosis (70%-99%), but the data are less Phalen test or Tinel sign does not exclude carpal tunnel
convincing. CEA can be considered in men with symptomatic syndrome. Diagnosis of CTS is usually made on clinical grounds.
carotid lesions of 50%-69%. Patients are selected for surgery based Most patients with mild CTS will respond to conservative
on their individual surgical risk and predicted survival, the measures, including nocturnal wrist splinting, which holds the
file:///C:/Users/vikas/OneDrive/Desktop/print.html 29/207
5/10/2021 print.html
presence of a surgically accessible carotid lesion, and the wrist in a neutral position and prevents excessive flexion during
experience of the surgeon/center. sleep.

Lesions <50% are typically monitored with annual Duplex


ultrasound. Surgery is not indicated at this stage, and patients are
medically managed with pharmacotherapy (eg, antiplatelet agents,
statins) and riskfactor optimization (eg, tobacco cessation, diabetes
mellitus and hypertension control).

Aspirin is recommended for the prevention of cardiovascular


disease (eg, coronary heart disease, cerebrovascular disease,
peripheral artery disease). In patients with existing cardiovascular
disease (such as this patient), aspirin reduces the risk of a
cardiovascular event and, in the case of carotid artery stenosis,
helps prevent progression of the lesion.

file:///C:/Users/vikas/OneDrive/Desktop/print.html 30/207
5/10/2021 print.html

Carpal tunnel syndrome is diagnosed with Tinel sign and Phalen Cataracts presents with progressive chronic vision loss and Cauda equina syndrome presents
test on physical exam, confirmed with nerve conduction studies, opacification of the natural lens of the eye on physical exam. with hyporeflexia/areflexia, saddle anesthesia,
and treated initally with nocturnal wrist splinting, NSAIDs, and and asymmetric motor weakness.
local corticosteroid injection. Cataract is a vision-impairing disease characterized by progressive
thickening of the lens. Oxidative damage of the lens occurs with CES can be caused by disc herniation or rupture, spinal stenosis,
Hand pain in the distribution of the median nerve is consistent with aging and leads to cataract formation. Patients usually complain of tumors, infection, hemorrhage, or iatrogenic injury. In adults, the
carpal tunnel syndrome (CTS). CTS is caused by compression of blurred vision, problems with nighttime driving, and glare. spinal cord ends at the L1-L2 vertebral level, and the lumbosacral
the median nerve where it passes under the transverse carpal Definitive treatment is lens extraction. nerve roots below this level form the cauda equina, which floats in
ligament (flexor retinaculum) in the wrist. Patients have cerebrospinal fluid. The cauda equina provides sensory innervation
reproduction of pain on percussion over the median nerve at the to the saddle area, motor innervation to the sphincters (anal and
wrist (Tinel sign) or by holding the wrists in full flexion with the urethral), and parasympathetic innervation to the bladder and
dorsum of the hands pressed together (Phalen test). But a negative lower bowel. A cauda equina lesion causes lower motor neuron
Phalen test or Tinel sign does not exclude carpal tunnel signs as the nerve roots are part of the peripheral nervous system.
syndrome. Diagnosis of CTS is usually made on clinical grounds. Patients with CES usually present with gradual onset of severe
Most patients with mild CTS will respond to conservative back pain with unilateral radiculopathy, saddle area numbness,
measures, including nocturnal wrist splinting, which holds the hyporeflexia, and marked asymmetric lower-extremity weakness.
wrist in a neutral position and prevents excessive flexion during Management includes emergency magnetic resonance imaging,
sleep. neurosurgical evaluation, and possibly intravenous
glucocorticoids.

file:///C:/Users/vikas/OneDrive/Desktop/print.html 31/207
5/10/2021 print.html

file:///C:/Users/vikas/OneDrive/Desktop/print.html 32/207
5/10/2021 print.html

Cause of coma can be remembered with the mneumonic Cause of coma can be remembered with the mneumonic Cause of coma can be remembered with the mneumonic
"SMASHED": "SMASHED": "SMASHED":

S: Structural brain pathology S: Structural brain pathology S: Structural brain pathology


M: {{c1:Meningitis}}, mental illness M: {{c1:Meningitis}}, mental illness M: {{c1:Meningitis}}, mental illness
A: Alcohol, acidosis A: Alcohol, acidosis A: Alcohol, acidosis
S: Seizures, substrate deficiency S: Seizures, substrate deficiency S: Seizures, substrate deficiency
H: Hyper- or hypoglycemia, hyper- H: Hyper- or hypoglycemia, hyper- H: Hyper- or hypoglycemia, hyper-
or hypothermia, hyponatremia, hypoxia, hypercapnia, hypotension or hypothermia, hyponatremia, hypoxia, hypercapnia, hypotension or hypothermia, hyponatremia, hypoxia, hypercapnia, hypotension
E: Endocrine causes (↓cortisol, ↑ or ↓T3), extreme electrolyte E: Endocrine causes (↓cortisol, ↑ or ↓T3), extreme electrolyte E: Endocrine causes (↓cortisol, ↑ or ↓T3), extreme electrolyte
disturbances (Ca++, Mg++, PO43-) disturbances (Ca++, Mg++, PO43-) disturbances (Ca++, Mg++, PO43-)
D: Drugs (opioids, barbituates, benzos), dangerous D: Drugs (opioids, barbituates, benzos), dangerous D: Drugs (opioids, barbituates, benzos), dangerous
compounds (CO, cyanide, methanol) compounds (CO, cyanide, methanol) compounds (CO, cyanide, methanol)

file:///C:/Users/vikas/OneDrive/Desktop/print.html 33/207
5/10/2021 print.html
Central cord syndrome typically occurs with hyperextension Central retinal artery occlusion is most commonly caused by Central retinal vein occlussion is a(n) acute unilateral painless
injury (car accident) and presents with weakness that is more a(n) embolized plaque from the ipsilateral carotid artery, presents loss of vision that is diagnosed with fundoscopy and confirmed
pronounced in the upper extremities, and loss of pain and with acute new vision loss and a(n) cherry red spot on fundoscopy, with fluorescein angiography.
temperature sensation. and is treated immediately with an ocular massage.
The characteristic changes on funduscopic examination are
Central cord syndrome (CCS) typically occurs with A cardioembolic source (eg, due to atrial fibrillation) is also sometimes referred to as the "blood and thunder" appearance, and
hyperextension injuries in elderly patients with pre-existing possible. The cherry red spot is seen because the macula include optic disk swelling, retinal hemorrhages, dilated veins, and
degenerative changes in the cervical spine. This type of traumatic receives its blood supply from the choroid, supplied by the cotton wool spots. CRVO should be considered in the differential
injury causes selective damage to the central portion of the anterior posterior ciliary arteries, while the surrounding retina is pale diagnosis for acute or subacute monocular loss of vision, but it is
spinal cord, specifically the central portions of the corticospinal due to retinal artery infarction. typically not quite as acute as the vision loss seen in patients with
tracts and the decussating fibers of the lateral spinothalamic tract. central retinal artery occlusion.
CCS is characterized by weakness that is more pronounced in the Central retinal artery occlusion primarily affects elderly patients
upper extremities than the lower. This unique motor deficit occurs and causes acute, severe, and painless monocular vision loss. CRVO is caused by thrombosis of the central retinal vein and is
because the motor fibers serving the arms are closer to the central Funduscopy can reveal a whitened retina (due to edema) and, in most common in patients with coagulopathy, hyperviscosity,
part of the corticospinal tract. A patient with CCS may also have the macula, the central fovea appears red from underlying choroid chronic glaucoma, and atherosclerotic risk factors (eg, age,
occasional selective loss of pain and temperature sensation in the (cherry red spot). Patients usually have a defect in the afferent diabetes, hypertension). The diagnosis can be confirmed with
arms due to damage to the spinothalamic tract. pupil reflex. CRAO is an ophthalmologic emergency. A delay in fluorescein angiography. Patients with no significant macular
treatment may result in permanent loss of vision. Immediate edema or neovascularization are often managed conservatively
intervention includes ocular massage, which dislodges the embolus with close observation. Significant macular edema can be treated
to a point further down the arterial circulation and improves retinal with intravitreal injection of vascular endothelial growth factor
perfusion. Medical management and anterior chamber paracentesis inhibitors. No treatment is particularly effective, but some patients
to lower the intraocular pressure may be used, but ocular massage may have partial recovery of vision within the first 3 months.
has the most rapid action. Carbogen therapy (5 % CO2 and 95%
O2) or hyperbaric oxygen (HBO) therapy have been shown to be
beneficial if given early. Thrombolytics may be useful if initiated
within 4-6 hours of visual loss but they are administered
intraarterially. Their efficacy has not been systematically studied.

Workup also includes noninvasive imaging of the carotids to


evaluate for stenosis. Atherosclerotic treatment (eg, aspirin, statin)
and, in cases of cardioembolic phenomenon, long-term
anticoagulation (eg, warfarin) are often initiated.

file:///C:/Users/vikas/OneDrive/Desktop/print.html 34/207
5/10/2021 print.html

Cerebral infarcts can recover within 24 hours, termed a(n) Cervical myelopathy is diagnosed with MRI and treated
transient ischemic attack; recover within two weeks, termed surgically. Cervical myelopathy is neurologic dysfunction secondary to
reversible ischemic neurologic deficit; or lead to brain death, spinal cord compression that initially presents with gait
termed a(n) stroke, which can either still be evolving or completed. Conservative treatment does not play a role in the treatment of disturbance.
cervical stenosis with myelopathy. Main goal of surgery is to
In TIAs, the blockage of blood flow doesn't last long enough to prevent further neurologic worsening which is why it is important Other symptoms include loss of hand dexterity, bowel and bladder
cause permanent infarction, due to either collateral circulation or to make the diagnosis early. Once the patient loses ability to dysfunction are late findings.
emboli break-up. ambulate, it is unlikely to return even with surgery.

Cervical radiculopathy is diagnosed with MRI and treated


Cervical radiculopathy is most commonly caused by cervical Chronic fatigue syndrome is characterized by unexplained
conservatively (NSAIDs, time, physical therapy, epidural
spondylosis and disc herniation and most commonly presents with fatigue that presents with four of the following for longer than 6
injections)
unilateral neck pain that radiates down the arm in a dermatomal months: ↓memory/concentration, muscle pain, sore throat, tender
pattern. lymph nodes, unrefreshing sleep, joint pain, headaches, and
Conservative treatment consists of NSAIDs, time, physical
postexertional malaise.
therapy, epidural injections. Surgery is helpful patients who do not
Other causes: shoulder pathology, peripheral nerve entrapment,
respond to conservative management or who have significant
thoracic outlet syndrome, zoster, Pancoast tumor. Depression is common.
weakness in one muscle groups.

Cluster headaches present with a severe, unilateral periorbital Compression fractures typically occur in patients with Compression fractures typically occur in patients with
pain that begin at night (a few hours after going to sleep), is osteoporosis and present with tenderness at the affected osteoporosis and present with tenderness at the affected
associated with increased lacrimation and miosis, and is vascular level, palpable unevenness/disruption of vertebral process level, palpable unevenness/disruption of vertebral process
in origin. alignment, paravertebral hematoma, weakness/numbness, alignment, paravertebral hematoma, weakness/numbness,
and/or neurogenic shock. and/or neurogenic shock.
Accompanied by ipsilateral lacrimation, facial flushing, nasal
stuffiness/discharge. Clusters of headaches occur in periods of Patients with gradual onset VCF can be asymptomatic. However, Patients with gradual onset VCF can be asymptomatic. However,
weeks and are often symptom free for months (attacks “cluster” an acute VCF can present with sudden onset of low back pain and an acute VCF can present with sudden onset of low back pain and
together). Pain may be so severe that the patient may even become decreased spinal mobility after bending, coughing, or lifting. The decreased spinal mobility after bending, coughing, or lifting. The
suicidal. pain typically increases with standing, walking, or lying on the pain typically increases with standing, walking, or lying on the
back. Examination can show tenderness at the affected level. Each back. Examination can show tenderness at the affected level. Each
The paroxysms of cluster headache attacks begin during sleep, VCF can decrease a patient's height by ≥1 cm and lead to VCF can decrease a patient's height by ≥1 cm and lead to
peak rapidly, last approximately 90 minutes, and occur up to 8 kyphosis, which is associated with decreased respiratory capacity kyphosis, which is associated with decreased respiratory capacity
times daily for 6-8 weeks, followed by a period of remission. As and increased risk for atelectasis pneumonia. Kyphosis can also and increased risk for atelectasis pneumonia. Kyphosis can also
the attacks are short but severe, prophylactic medication (eg, cause a protuberant abdomen, early satiety, and weight cause a protuberant abdomen, early satiety, and weight
verapamil, lithium) is the key to management and should be started loss. VCF is associated with a significantly increased risk for loss. VCF is associated with a significantly increased risk for
as soon as possible after the onset of an acute attack. Ergotamine future vertebral and nonvertebral fractures. future vertebral and nonvertebral fractures.
may be used to abort an acute cluster headache attack, especially
when started early, but 100% oxygen is the most rapid-acting and
effective treatment without side effects. Dihydroergotamine is used
for acute treatment of migraine headaches, which are typically
pulsatile and accompanied by nausea, vomiting, or
photophobia. Nonsteroidal anti-inflammatory drugs (eg,
indomethacin) are used for acute management of tension
headaches (usually bilateral and not accompanied by lacrimation,
conjunctival injection, or miosis) and some cases of migraine
headaches. Topiramate is an anticonvulsant used for prophylaxis of
migraine headaches. These medications are rarely used for
prophylaxis of cluster headaches.

file:///C:/Users/vikas/OneDrive/Desktop/print.html 35/207
5/10/2021 print.html

Contrasting strokes, patients who have suffered a(n) thrombotic Conus syndrome presents with hyperreflexia, perianal anesthesia, Crepitus and pain that occurs when squeezing the metatarsal joints
stroke, classically awaken from sleep with symptoms that and symmetric motor weakness. is suggestive of what foot injury?
fluctuate in a stuttering progression with periods of improvement;
whereas the onset of symptoms for a(n) embolic or subarachnoid In adults, the spinal cord ends at the L1-L2 vertebral level, and the Morton neuroma
hemorrhagic stroke is very rapid, within seconds, with maximal lumbosacral nerve roots below this level form the cauda equina,
deficits; and a(n) intraparenchymal hemorrhagic stroke which floats in cerebrospinal fluid. A lesion in the conus Interdigital (Morton) neuroma commonly occurs in runners. The
gradually worsens over minutes to hours. medullaris causes both upper and lower motor neuron symptoms disorder is not a true neuroma but is a mechanically induced
file:///C:/Users/vikas/OneDrive/Desktop/print.html 36/207
5/10/2021 print.html
as the conus is part of the spinal cord. Patients with conus neuropathic degeneration of the interdigital nerves that causes
Carotid artery atherosclerosis can produce stroke by flow
medullaris syndrome typically present with severe back pain with numbness, aching, and burning in the distal forefoot from the
limitation or embolization. Symptoms result from ipsilateral ocular lesser radiculopathy and earlier onset of bowel and bladder metatarsal heads to the third and fourth toes. The symptoms are
and cerebral hemisphere ischemia. Examination can show carotid
disturbances compared to CES. Numbness is confined to the worsened by walking on hard surfaces and wearing tight or high-
bruit. Large hemispheric strokes can produce cerebral edema over perianal area. Motor weakness is generally symmetric with heeled shoes.
several hours to days but very rarely cause such severe symptoms
hyperreflexia.
so early. The diagnosis of Morton neuroma is primarily made clinically. On
examination, squeezing the metatarsal joints will cause pain on the
Ischemic strokes can be thrombotic or embolic in etiology, with
plantar surface of the foot along with crepitus between the third
atherosclerotic-related thrombotic strokes the most common.
and fourth toes (Mulder sign). Treatment involves metatarsal
However, the presence of an irregularly irregular rhythm in a
support with a bar or padded shoe inserts to decrease pressure on
patient with a history of structural heart disease (severe left atrial
the metatarsal heads. Surgery is usually reserved for patients who
enlargement likely due to poorly controlled hypertension) is
fail conservative treatment.
strongly suggestive of a cardioembolic etiology in paroxysmal
atrial fibrillation. A very rapid onset of severe focal neurologic
deficits suggests an embolus causing obstruction of blood flow
through the left MCA. In contrast, an unstable ruptured
atherosclerotic plaque with a propagating thrombus on its surface
would cause the development of focal neurologic symptoms at a
relatively slower rate.

Cardiogenic emboli usually occur in patients with arrhythmias,


hypercoagulability, or other conditions that cause embolic
formations (eg, endocarditis). When the source of embolus is the
heart or aorta, multiple infarcts occur within different vascular
territories, producing patchy neurologic findings. In embolic
stroke, symptom onset is abrupt and usually maximal at the start
and sometimes results in rapid recovery.

file:///C:/Users/vikas/OneDrive/Desktop/print.html 37/207
5/10/2021 print.html

file:///C:/Users/vikas/OneDrive/Desktop/print.html 38/207
5/10/2021 print.html

Creutzfeldt-Jakob disease is a mostly sporadic fatal Deficits caused by multiple sclerosis can improve within days. Delirium is distinguished from dementia, due to its acute course
neurodegenerative disease that is diagnosed with the presence of and fluctuating severity.
PRNP gene mutations; and associated with sharp wave complexes Subcortical white matter demyelination is seen in multiple
on EEG, and elevated amounts of 14-3-3 regulatory proteins in the sclerosis, but deficits typically take days or weeks (not hours) to Treat with haloperidol + underlying cause
CSF. improve.

Creutzfeldt-Jakob disease is a rare, invariably fatal,


neurodegenerative disease caused by prions (abnormally folded
proteins that act as transmissible pathogenic agents). These prion
proteins cause rapid neuronal loss and most patients die within 1
year. Key features of this disease include a long incubation period,
characteristic spongiform changes, and lack of inflammatory
response. Most cases are sporadic (85%), with the remainder being
hereditary or iatrogenic (due to contaminated transplants or
surgical instruments).

This condition should be suspected in older patients (between age


50-70) who present with rapidly progressing dementia and
myoclonus. Other symptoms include insomnia, apathy, behavioral

file:///C:/Users/vikas/OneDrive/Desktop/print.html 39/207
5/10/2021 print.html
changes, and impaired vision. Patients eventually lose the ability to
move and speak and become comatose.

Spongiform changes on postmortem brain biopsy have been


the gold standard for diagnosis. However, genetic testing for the
prion protein (PRNP) gene is now used. Bi- or tri-phasic sharp
wave complexes on electroencephalogram (EEG) are also typical.
Elevated cerebrospinal fluid (CSF) levels of 14-3-3 regulatory
proteins are characteristic but not required for diagnosis. Treatment
is supportive.

Dementia is characterized by a significant functional deficit, Dementia with lewy bodies is a dementia characterized Dexamethasone is added to empiric treatment of meningitis in
severe memory or social impairment, and preserved consciousness. by waxing and waning cognition (especially in the areas of adults to prevent the neurological complications caused by which
attention and alertness), Parkinsonian symptoms, and visual organisms?
hallucinations.
Streptococcus pneumoniae
Two of the following core features are essential for diagnosis of
probable DLB, and one is essential for possible DLB: fluctuating However, it should be discontinued once pneumococcal
cognition with pronounced variations in attention and alertness, meningitis is ruled out. Note: Dexamethsone is added for
recurrent visual hallucinations that are typically well formed and Haemophilus meningitis in children.
detailed, parkinsonism with onset subsequent to cognitive
decline. Features that can support the diagnosis include the Empiric treatment for bacterial meningitis depends on risk factors
following: repeated falls, syncope/transient loss of consciousness, and specific pathogens. In immunocompromised patients (eg,
autonomic dysfunction, neuroleptic sensitivity, and systematized transplant recipients on immunosuppressants), the treatment of
delusions choice is cefepime or ceftazidime plus vancomycin plus
ampicillin. Cefepime is a fourth-generation cephalosporin that
covers most of the major organisms of bacterial meningitis (eg,
Streptococcus pneumoniae, Neisseria meningitidis, group B
streptococci, Haemophilus influenzae) as well as Pseudomonas

file:///C:/Users/vikas/OneDrive/Desktop/print.html 40/207
5/10/2021 print.html
aeruginosa. Vancomycin provides coverage against cephalosporin-
resistant pneumococci, and ampicillin covers Listeria
monocytogenes. Patients should also empirically receive
dexamethasone to help prevent the neurologic complications (eg,
deafness, focal deficits) of S pneumoniae meningitis, but it should
be discontinued once pneumococcal meningitis is ruled out.

Cefepime and ampicillin would cover most organisms responsible


for meningitis, including Pseudomonas and Listeria; however,
vancomycin is also necessary to cover cephalosporin-resistant
pneumococci.

Bacterial meningitis should be suspected in patients with ≥2 of the


following manifestations: headache, fever (usually >38 C [100.4
F]), nuchal rigidity, and altered mental status

Symptoms usually arise acutely and leukocytosis is common.


Blood cultures are often positive, but diagnosis requires a lumbar
puncture. Cerebrospinal fluid typically shows high opening
pressure (>350 mm H2O), low glucose (<40 mg/dL), high protein
(>200 mg/dL), and neutrophilic leukocytosis (>1000/mm3).

file:///C:/Users/vikas/OneDrive/Desktop/print.html 41/207
5/10/2021 print.html

Disk herniation is typically caused by heavy lifting and most


commonly involves L4-L5 or L5-S1 and presents with lower back
pain and a(n) sciatica that is exacerbated by hip flexion.

Sciatica: lightning, shooting pain down the leg

Do patients with osteoarthritis of the spine typically present with


pain?
Dissection of vertebral artery causes
No
Wallenberg syndrome.(mostly due to severe neck manipulations)
Although this is true for both cervical and lumbar, pain is seen
more commonly in cervical osteoarthritis.

Duchenne muscular dystrophy is a(n) X-linked recessive disease


Does Duchenne or Becker muscular dystrophy have an earlier involving a mutated dystrophin protein that presents with NO ( ͡~ ͜ʖ
onset and more severe course? ͡°) inflammation, proximal muscle weakness that eventually
Duchenne muscular dystrophy is diagnosed with ↑↑creatine
involves the respiratory muscles, and calf pseudohypertrophy
phosphokinase and DNA testing.
Duchenne muscular dystrophy
Psuedohypertrophy: true muscle hypertrophy that turns into fat.
Ultimately results in wheelchair confinement, respiratory failure,
and death in third decade. Also presents with Gowers maneuver, in which patients use hands
to get up off floor

file:///C:/Users/vikas/OneDrive/Desktop/print.html 42/207
5/10/2021 print.html

Duchenne muscular dystrophy is treated with prednisone, which


is associated with significantly increased strength, muscle
function, and pulmonary function. Early signs of SC compression are LMN signs Early signs of SC compression are LMN signs
Late signs are UMN signs Late signs are UMN signs
Surgery to correct progressive scoliosis is often necessary once
patient becomes wheelchair dependent.

Ehrlichiosis is spread by the lone star tick; presents similarly to Encephalitis often presents with signs and symptoms resembling Encephalitis often results in a(n) bloody lumbar puncture with
Rocky mountain spotted fever, but lacks rash; and is typically meningitis, as well as focal neurologic deficits, altered mental lymphocytosis; and is definitively diagnosed with HSV PCR.
associated with a high fever, which can help distinguish it from status, and/or seizures.
other tick-borne infections. Lumbar puncture is only bloody 30% of the time, but it still a
classic teaching.
Ehrlichiosis is a tick-borne infection caused by Ehrlichia
chaffeensis and E ewingii. These bacteria are carried by ticks,
including the lone star tick (Amblyomma americanum), found in
the southeastern and south central United States. The principal
reservoir is the white tail deer. Ehrlichiosis is characterized by an
acute febrile illness with malaise and altered mental status.
Ehrlichiosis is not often associated with a rash (<30% in adults)
and is described as "Rocky Mountain spotted fever (RMSF)
without the spots." Neurologic symptoms may include confusion,
mental status changes, clonus, and neck stiffness. Laboratory
studies often show leukopenia and/or thrombocytopenia, along
with elevated aminotransferases and lactate dehydrogenase. The
diagnosis is often clinical, although definitive diagnosis can be
made through visualization of intracytoplasmic morulae in white
blood cells or through polymerase chain reaction testing.
Doxycycline is the appropriate treatment and is often initiated
empirically when the diagnosis is suspected while confirmatory
testing is pending.

Ixodes scapularis, the tick that carries Borrelia burgdorferi, is not


commonly found in the southern United States. The majority
(80%) of patients with early localized (primary) Lyme disease
(within days of the tick bite) have the classic erythema migrans
rash with or without constitutional symptoms. Lyme disease
typically presents with a rash and the presence of a high fever and
hematologic and liver enzyme abnormalities Early disseminated
(secondary) Lyme disease exhibits prominent neurologic (eg,
encephalitis, cranial nerve palsy, radiculopathy) and cardiac (eg,
heart block, pericarditis) manifestations several weeks to months
after the initial tick bite. Doxycylcine is sufficient to treat Lyme
disease. Ceftriaxone is used for the treatment of neurologic and
cardiac manifestations of Lyme disease.

In 90% of RMSF patients, a maculopapular rash (involving the


soles and palms) occurs a few days after the onset of fever.
Doxycycline is the first-line treatment. Chloramphenicol is a
second-line treatment option for ehrlichiosis and RMSF. It is
associated with blood dyscrasias and is not commonly used.

file:///C:/Users/vikas/OneDrive/Desktop/print.html 43/207
5/10/2021 print.html

file:///C:/Users/vikas/OneDrive/Desktop/print.html 44/207
5/10/2021 print.html

Encephalitis that is definitively caused by HSV, may present with


temporal lobe bilateral discharges on EEG and anosmia.

Endophthalmitis is most commonly caused by surgery and is


treated in these instances with intravitreal antibiotic injection or
vitrectomy
Enlarged cup-to-disk ratio is associated with which type of
Endophthalmitis is an invasive infection of the globe (bacterial or
vision loss?
fungal) and is due to disruption of the external surface of the eye
(eg, trauma). Postoperative endophthalmitis is the most common
Glaucoma
form of endophthalmitis. It usually occurs within six weeks of
surgery. It is an infection within the eye, particularly the vitreous.
Open-angle glaucoma is usually characterized by gradual painless
Patients usually present with pain and decreased visual acuity.
peripheral vision loss. Funduscopy typically demonstrates optic
Examination reveals swollen eyelids and conjunctiva, hypopyon,
disc enlargement with an increased cup:disc ratio.
corneal edema and infection. Endophthalmitis may show
conjunctival irritation as well, but patients will usually have
purulent haziness of the ocular contents and may have a layering-
out of pus in the anterior chamber (hypopyon). The vitreous can be
sent for Gram stain and culture. Based on the severity, intravitreal
antibiotic injection or vitrectomy is done.

file:///C:/Users/vikas/OneDrive/Desktop/print.html 45/207
5/10/2021 print.html

Epidural glucocorticoid can be considered in patients with what


Episcleritis is associated with which inflammatory conditions?
type of back pain?
1. Rheumatoid arthritis
Lumbosacral radiculopathy
2. Inflammatory bowel disease
They are not helpful for nonradicular LBP.
Episcleritis is characterized by inflammation seen at the white of
Epileptic seizures are classifed based on onset of electrical the eye, without involvement of the uveal tract. It is most strongly
Patients with acute (<4 weeks) LBP should be advised to maintain activity; when electrical activity involves the entire cortex, the
moderate activity with short courses of acetaminophen or associated with rheumatoid arthritis and inflammatory bowel
seizure is termed generalized; and when electrical activity involves disease but is not associated with ankylosing spondylitis.
nonsteroidal anti-inflammatory drugs (NSAIDs) to provide pain a focal region of the brain, the seizure is termed focal.
relief. Most patients will have resolution without any additional
intervention. However, those with subacute (4-12 weeks) or
chronic LBP are more likely to continue to have recurrent or
persistent pain.

Essential tremor is absent at rest, worsens with movement (worse Essential tremor is absent at rest, worsens with movement (worse Fasciculations indicate a(n) lower motor neuron damage.
with outstretched arms), and decreases markedly with alcohol use. with outstretched arms), and decreases markedly with alcohol use.
Control symptoms with propranolol (first-line) or primidone Control symptoms with propranolol (first-line) or primidone Spasticity, bulbar symptoms, and exaggerated deep tendon reflexes
(barbituate). (barbituate). are signs of upper motor neuron damage, while fasciculations
indicate a lower motor neuron damage. Weakness and wasting of
There’s typically a family history of a tremor and it’s often a There’s typically a family history of a tremor and it’s often a the muscles may be due to both upper and lower motor neuron
man. ET is characterized by tremors that disturb goal-directed man. ET is characterized by tremors that disturb goal-directed lesions.
actions (eg, drinking from a glass, buttoning a shirt). actions (eg, drinking from a glass, buttoning a shirt).
Approximately 5% of the population is affected, and a family Approximately 5% of the population is affected, and a family
history of tremors is often present. Symptoms usually arise in the history of tremors is often present. Symptoms usually arise in the
bilateral hands and forearms; the head is sometimes involved, but bilateral hands and forearms; the head is sometimes involved, but
the legs are generally spared. Physical examination often shows the legs are generally spared. Physical examination often shows
worsening tremor with outstretched arms or finger-to-nose worsening tremor with outstretched arms or finger-to-nose
testing. Treatment is indicated if tremors cause significant testing. Treatment is indicated if tremors cause significant
disability or adversely affect quality of life. Beta blockers such as disability or adversely affect quality of life. Beta blockers such as
propranolol are first-line therapy; patients may also receive an propranolol are first-line therapy; patients may also receive an
anticonvulsant such as primidone alone or in combination. anticonvulsant such as primidone alone or in combination.
Clonazepam is second line. Clonazepam is second line.

file:///C:/Users/vikas/OneDrive/Desktop/print.html 46/207
5/10/2021 print.html

file:///C:/Users/vikas/OneDrive/Desktop/print.html 47/207
5/10/2021 print.html

For migraines, are the following medications abortive or


preventive therapy?

β-blockers: preventive therapy


Divalproex sodium (converted to valproic acid in stomach):
preventive therapy
Ergots: abortive therapy
Prochlorperazine: abortive therapy
Topiramate: preventive therapy
Triptans: abortive(5-HT agonists) therapy

Abortive medications (eg, sumatriptan) can be taken as needed at


the onset of an attack, but prophylactic medication should be
For a first time seizure, order an EEG and CT scan and consider For a first time seizure, order an EEG and CT scan and consider considered in patients who:
the VITAMINS mneumonic to identify a cause: the VITAMINS mneumonic to identify a cause: 1. Have frequent (eg, >4/month) or long-lasting (eg, >12 hours)
episodes
Vascular Vascular 2. Experience disabling symptoms that prevent regular activities
Infection Infection despite abortive treatment
Trauma Trauma 3. Are unable to take or have had no relief with abortive
Autoimmune Autoimmune medications
Metabolic Metabolic 4. Overuse abortive medication (eg, nonsteroidal anti-
Ingestion or wIthdrawal Ingestion or wIthdrawal inflammatory drugs [NSAIDs]) and have rebound headache
Neoplasm Neoplasm
PSych PSych Prochlorperazine is antipsychotic (D2 antagonist) and antiemetic.

file:///C:/Users/vikas/OneDrive/Desktop/print.html 48/207
5/10/2021 print.html
For patients that are inelgible for t-PA, what is appropriate Fundoscopy is normal in optic neuritis as inflammation is behind Fundoscopy is normal in optic neuritis as inflammation is behind
antiplatelet/antithrombotic therapy following a thrombotic optic nerve head optic nerve head
stroke?

No prior antiplatelet therapy: aspirin alone


Previously on aspirin therapy: aspirin + dypyridamole OR
clopidogrel alone
Previously on aspirin therapy + intracranial large artery
atherosclerosis: aspirin + clopidogrel

1. Aspirin is the only antiplatelet agent that is effective in


reducing the risk of early recurrence of ischemic stroke.
Patients should therefore be given aspirin as early as
possible (within 24 hours of stroke onset) and continue it
indefinitely to prevent future strokes. Long-term
anticoagulation (eg, dabigatran, warfarin) is recommended
for stroke prevention in patients with atrial fibrillation based
on the CHADS2 criteria and stroke risk.
2.
3. Unfractionated heparin or low molecular weight heparin (eg,
enoxaparin) should be avoided in the management of acute
stroke due to high risk of bleeding. Current evidence
suggests that early administration of these anticoagulants
does not reduce the risk of recurrent stroke, even in patients
with underlying atrial fibrillation. The role of these agents in
patients with cardioembolic stroke from underlying
mechanical heart valves or intracardiac thrombus is unclear.
4.
5. Dipyridamole is a PDE inhibitors that increases cAMP,
which inhibits platelet aggregation. Clopidogrel is a
P2Y12 receptor (ADP receptor) inhibitor, which prevents
expression of GPIIb/IIIA receptors on the platelet surface.

file:///C:/Users/vikas/OneDrive/Desktop/print.html 49/207
5/10/2021 print.html

file:///C:/Users/vikas/OneDrive/Desktop/print.html 50/207
5/10/2021 print.html
Generalized seizures are classifed based on motor status; while Gerstmann syndrome is classically associated with a lesion in Guillain-Barre syndrome can lead to respiratory arrest, so assess
focal seizures are classified based on level of awareness and motor which part of the brain? the patient's need for intubation.
status.
Dominant (left) parietal lobe Frequently progresses to involve facial and bulbar muscles.
Usually symmetric. Autonomic features are dangerous
Dominant parietal lobe lesions can present with contralateral complications.
sensory loss (eg, pain, vibration, agraphesthesia, astereognosis),
Gerstmann syndrome (eg, right-left confusion, agraphia,
acalculia) and contralateral inferior homonymous quadrantanopsia
due to superior optic radiations involvement.

Nondominant parietal lobe lesions typically cause anosognosia


(denial of one's disabilities), hemineglect and contralateral apraxia
(inability to carry out learned purposeful movements).

Dominant temporal lobe lesions can affect comprehension


(receptive aphasia), ability to speak nouns (anomic aphasia),
repetition (conductive aphasia) due to arcuate fasciculus
involvement, and contralateral superior homonymous
quadrantanopsia due to inferior optic radiations (Meyer's loop)
involvement.

Nondominant frontal lobe lesions typically affect the way a person


conveys emotion through speech (motor aprosodia), contralateral
weakness, and apraxia.

Nondominant temporal lobe lesions can impair ability to


comprehend emotional gestures (sensory aprosodia). These
patients can also develop a contralateral homonymous
quadrantanopsia due to the inferior optic radiations involvement.

Neurological terms:
Anosognosia: denial of one's disabilities, lack of insight.
Prosopagnosia: inability to recognize the faces of familiar people.
Agraphesthesia: disorientation of the skin's sensation across its
space. It is a difficulty recognizing a written number or letter
traced on the skin after parietal damage.
Astereognosis: inability to identify an object by active touch of the
hands without other sensory input.
Aphasia: inability to understand or express speech, caused by brain
damage.
Aprosodia: inability to properly convey or interpret emotional
prosody. Prosody in language refers to the ranges of rhythm, pitch,
stress, intonation, etc.
Apraxia: inability to perform particular purposive actions, as a
result of brain damage.

file:///C:/Users/vikas/OneDrive/Desktop/print.html 51/207
5/10/2021 print.html

Guillain-Barre syndrome is characterized by an autoimmune


demyelinating polyneuropathy that produces a(n) ascending
Guillain-Barre syndrome is diagnosed with ↑protein in the CSF
paralysis and is usually preceded by a(n) viral or mycoplasmal
and ↓motor nerve conduction velocity.
upper respiratory or GI infection

Other causes: Hodgkin disease, lupus, post-op, post-HIV


seroconversion Guillain-Barre syndrome is treated with IV immunoglobulin
when the patient has significant weakness and plasmapheresis if
progression continues.

Do NOT give steroids.

file:///C:/Users/vikas/OneDrive/Desktop/print.html 52/207
5/10/2021 print.html

Herpes encephalitis has a(n) acute onset and most commonly


presents in immunocompetent patients.

HSV Encephalitis is treated with acyclovir. The majority of acute viral encephalitis cases are due to unknown
causes. However, herpes viruses (eg, herpes simplex, varicella,
CMV encephalitis is treated with ganciclovir or foscarnet. HSV retinopathy is painful and can cause acute retinal necrosis Epstein-Barr) most commonly cause encephalitis in
Encephalitis is otherwise treated with supportive care. syndrome. immunocompetent adults. Patients typically present with fever,
altered mental status, agitation, headaches, and seizures that can
Ophthalmologic problems occur in approximately half of patients rapidly progress to complete unresponsiveness or coma.
with advanced HIV infection. Retinitis can occur as a complication Examination can show neurologic abnormalities, including
of opportunistic infections in AIDS patients. Both varicella-zoster hemiparesis, cranial nerve palsies, and exaggerated deep-tendon
(VZV) and herpes simplex virus (HSV) can cause severe, reflexes. Clinical signs of meningeal irritation (eg, photophobia,
devastating intraocular inflammation. Most cases of retinal disease nuchal rigidity) are usually absent in patients with pure
are believed to be a reactivation of a previously acquired infection. encephalitis.
In an immunocompromised individual, HSV retinitis may be Lumbar puncture usually shows cerebrospinal fluid (CSF) findings
characterized by rapidly progressing bilateral necrotizing retinitis of elevated white blood cell count with lymphocytic
(referred to as the "acute retinal necrosis syndrome"). The initial predominance, normal glucose, and normal to slightly elevated
symptoms are keratitis and conjunctivitis with eye pain, followed protein concentration (generally 100 mg/dL). Most patients have
by rapidly progressive visual loss. Funduscopy reveals widespread, an increased red blood cell count as well. Diagnosis is usually
pale, peripheral lesions and central necrosis of the retina. confirmed by detecting viral DNA by polymerase chain reaction
(PCR) in the CSF. Empiric treatment with intravenous acyclovir
HSV infection of the eye is the most common cause of corneal should be started while awaiting PCR results as encephalitis is
blindness in the United States. often associated with significant morbidity and mortality.

file:///C:/Users/vikas/OneDrive/Desktop/print.html 53/207
5/10/2021 print.html

How are reflexes affected by Guillain-Barre syndrome?

How are deep tendon reflexes affected by brain death? Decreased or absent

They are preserved since the spinal cord may still be functioning Guillain-Barré syndrome is an acute or subacute ascending
polyradiculoneuropathy that is often preceded by an upper
Brain death is defined as the irreversible cessation of brain respiratory tract infection or diarrheal illness (eg, Campylobacter
activities. Interestingly, the criteria for brain death are not uniform jejuni). It typically presents as bilateral leg weakness that ascends
in different countries, but the essential elements in all such criteria to the arms, respiratory muscles, and face to generalized flaccid
include: (1) evaluating cortical and brain stem functions, and (2) paralysis. Distal paresthesias are common, but other sensory
proving the irreversibility of brain activity loss (e.g., sufficient symptoms are uncommon or mild. Reflexes are diminished or
observation length, no hypothermia, etc.). absent. Autonomic disturbances are common and include
Horner Syndrome presents with ipsilateral ptosis, miosis, and tachycardia, bradycardia, hypertension, orthostatic hypotension,
anhidrosis due to interruption of the cervical sympathetic nerves. Brain death is a clinical diagnosis. The characteristic findings are and urinary retention.
absent cortical and brain stem functions. The spinal cord may still
Preganglionic interruption is more worrisome than postganglionic. be functioning; therefore, deep tendon reflexes may be present. An
Causes: idiopathic, pancoast tumor, internal carotid dissection, isoelectric EEG can be used as a confirmatory test, but it is not
brainstem stoke, neck trauma. absolutely necessary. Other diagnostic tools (e.g., Doppler
ultrasonography, angiography) can demonstrate cerebral blood
flow cessation, but these are not commonly employed.

Pupillary light reaction and oculovestibular reaction are absent in


patients with brain death, thereby reflecting global brainstem
unresponsiveness. The heart rate fails to accelerate after atropine
injection because vagal control of the heart is lost, and the heart
rate becomes invariant. An apnea test shows no spontaneous
respiration at Pco2 values of 50 mmHg and more, thereby
demonstrating no functioning of the lower brainstem centers.

How do lesions due to a plexuopathy present?


How do lesions due to a peripheral neuropathy present?
Motor and sensory deficits involving more than one nerve
Asymmetric weakness
How do lesions due to a radiculopathy present?
Findings are variable depending on which part of the plexus is
Presents with diminished deep tendon reflexes; may include
involved. Trauma is the most common cause overall, especially for Pain along a(n) myotome and dermatome.
sensory changes (numbness, paresthesias, tingling), muscle
the brachial plexus. A postsurgical hematoma in the pelvis is a
atrophy, and fasciculations. Can be due to diabetes (nerve
more common cause in lumbosacral plexopathy. Patients may present with weakness, atrophy, and sensory deficits
infarction), trauma, entrapment, or vasculitis.
in a dermatomal pattern; may include fasciculations and
Brachial plexus—Erb–Duchenne type is the more common (upper diminished deep tendon reflexes.
Common neuropathies include
trunk—C5-6 roots). Lower trunk (C8-T1) is less common.
radial/ulnar/median/musculocutaneous nerves, long thoracic nerve,
axillary nerve, common peroneal nerve, and femoral nerve.
Lumbosacral plexus (L5-S3).

How do lesions in a neuromuscular junction present? How do lesions in the brainstem present? How do lesions in the cerebral cortex present?

Fatigability 1. Ipsilateral face deficit 1. Contralateral motor or sensory deficits


2. Contralateral body deficit 2. Aphasia
Muscles become weaker with use and recover with rest. Normal
sensation, no atrophy The corticospinal tract, dorsal columns, and spinothalamic tracts Primarily affects the face, arms, and trunk. The legs may be
cross but the cranial nerves do not. affected, but typically that deficit is not as severe, because the
neurons that control the lower extremities are in the
interhemispheric fissure

file:///C:/Users/vikas/OneDrive/Desktop/print.html 54/207
5/10/2021 print.html

file:///C:/Users/vikas/OneDrive/Desktop/print.html 55/207
5/10/2021 print.html

file:///C:/Users/vikas/OneDrive/Desktop/print.html 56/207
5/10/2021 print.html

file:///C:/Users/vikas/OneDrive/Desktop/print.html 57/207
5/10/2021 print.html

How do lesions in the spinal cord present? How do lesions in the subcortical area present? How do you cure cataracts?

1. Decreased sensation below the damaged spinal level. Complete hemiparesis and complete hemisensory loss Surgical resection
2. UMN signs below the damaged spinal level.
These involve the internal capsule, cerebral peduncles, thalamus, Cataract is a vision-impairing disease characterized by progressive
With acute injuries, spinal shock may be present and upper motor and pons; therefore the hemiparesis is complete b/c the neurons thickening of the lens. Oxidative damage of the lens occurs with
neuron signs may not be apparent initially. UMN signs may be controlling these structures all merge together subcortically and are aging and leads to cataract formation. Patients usually complain of
present with lesions in the brainstem and cortical/subcortical very close together. blurred vision, problems with nighttime driving, and glare.
regions as well. Definitive treatment is lens extraction.

Transverse injuries above C3 involve cessation of respiration and


are often fatal if acute. Cervical cord lesions that spare the phrenic
nerve but impair intercostal nerve function can produce respiratory
insufficiency. Lesions above the L2 cord level will cause
impotence and spastic paralysis of bladder. There is loss of
voluntary control of the bladder, which will empty automatically
by reflex action.

file:///C:/Users/vikas/OneDrive/Desktop/print.html 58/207
5/10/2021 print.html

file:///C:/Users/vikas/OneDrive/Desktop/print.html 59/207
5/10/2021 print.html

file:///C:/Users/vikas/OneDrive/Desktop/print.html 60/207
5/10/2021 print.html

How do you empirically treat bacterial meningitis? (I am so How do you empirically treat bacterial meningitis? (I am so How do you treat Alzheimer disease?
sorry, but you MUST know this) sorry, but you MUST know this)
Mild-to-moderate AD: cholinesterase inhibitors, which may slow
Infants under 1 month: dexamethasone + ampicillin + cefotaxime Infants under 1 month: dexamethasone + ampicillin + cefotaxime clinical deterioration by 6–12 months in up to 50% of patients
or aminoglycoside or aminoglycoside Moderate-to-severe AD: NMDA receptor antagonists
1+ month to <50 years: dexamethasone + vancomycin + third- 1+ month to <50 years: dexamethasone + vancomycin + third- (memantine), which may provide a modest benefit
generation cephalosporin (ceftriaxone or cefotaxime) generation cephalosporin (ceftriaxone or cefotaxime)
50+ years: dexamethasone + vancomycin + ampicillin + third- 50+ years: dexamethasone + vancomycin + ampicillin + third- Although the etiology of AD is not well understood,
generation cephalosporin (ceftriaxone or cefotaxime) generation cephalosporin (ceftriaxone or cefotaxime) histopathologic examination of brain tissue in affected patients
Immunosuppressed: dexamethasone + vancomycin + ampicillin Immunosuppressed: dexamethasone + vancomycin + ampicillin indicates a selective loss of cholinergic neurons. The first-line
+ cefepime or meropenum + cefepime or meropenum treatments for cognitive symptoms of AD are cholinesterase
inhibitors, which may improve quality of life and cognitive
The purple is there to help you quickly identify what was The purple is there to help you quickly identify what was functions (eg, memory, language, thought, reasoning) although
added at each increasing level of severity. added at each increasing level of severity. they have not been shown to alter the disease course in any
dementia. Donepezil, galantamine, and rivastigmine are effective
Treat aseptic meningitis with supportive care. The disease is self- Treat aseptic meningitis with supportive care. The disease is self- in patients with mild to moderate AD. Memantine, an N-methyl-D-
limited. Analgesics and fever reduction may be appropriate. limited. Analgesics and fever reduction may be appropriate. aspartate receptor antagonist, is approved for moderate to severe
dementia.
Empiric treatment for bacterial meningitis depends on risk factors Empiric treatment for bacterial meningitis depends on risk factors
and specific pathogens. In immunocompromised patients (eg, and specific pathogens. In immunocompromised patients (eg,
transplant recipients on immunosuppressants), the treatment of transplant recipients on immunosuppressants), the treatment of
choice is cefepime or ceftazidime plus vancomycin plus choice is cefepime or ceftazidime plus vancomycin plus
ampicillin. Cefepime is a fourth-generation cephalosporin that ampicillin. Cefepime is a fourth-generation cephalosporin that
covers most of the major organisms of bacterial meningitis (eg, covers most of the major organisms of bacterial meningitis (eg,
Streptococcus pneumoniae, Neisseria meningitidis, group B Streptococcus pneumoniae, Neisseria meningitidis, group B
streptococci, Haemophilus influenzae) as well as Pseudomonas streptococci, Haemophilus influenzae) as well as Pseudomonas
aeruginosa. Vancomycin provides coverage against cephalosporin- aeruginosa. Vancomycin provides coverage against cephalosporin-
resistant pneumococci, and ampicillin covers Listeria resistant pneumococci, and ampicillin covers Listeria
monocytogenes. Patients should also empirically receive monocytogenes. Patients should also empirically receive
dexamethasone to help prevent the neurologic complications (eg, dexamethasone to help prevent the neurologic complications (eg,
deafness, focal deficits) of S pneumoniae meningitis, but it should deafness, focal deficits) of S pneumoniae meningitis, but it should
be discontinued once pneumococcal meningitis is ruled be discontinued once pneumococcal meningitis is ruled
out. Ampicillin is added for patients age >50 or those who are out. Ampicillin is added for patients age >50 or those who are
immunocompromised due to increased risk of Listeria immunocompromised due to increased risk of Listeria
monocytogenes. Dexamethasone for pneumococcal in adults, monocytogenes. Dexamethasone for pneumococcal in adults,
or meningococcal in newborns. or meningococcal in newborns.

Cefepime and ampicillin would cover most organisms responsible Cefepime and ampicillin would cover most organisms responsible
for meningitis, including Pseudomonas and Listeria; however, for meningitis, including Pseudomonas and Listeria; however,
vancomycin is also necessary to cover cephalosporin-resistant vancomycin is also necessary to cover cephalosporin-resistant
pneumococci. pneumococci.

Bacterial meningitis should be suspected in patients with ≥2 of the Bacterial meningitis should be suspected in patients with ≥2 of the
following manifestations: headache, fever (usually >38 C [100.4 following manifestations: headache, fever (usually >38 C [100.4
F]), nuchal rigidity, and altered mental status F]), nuchal rigidity, and altered mental status

Symptoms usually arise acutely and leukocytosis is common. Symptoms usually arise acutely and leukocytosis is common.
Blood cultures are often positive, but diagnosis requires a lumbar Blood cultures are often positive, but diagnosis requires a lumbar
puncture. Cerebrospinal fluid typically shows high opening puncture. Cerebrospinal fluid typically shows high opening
pressure (>350 mm H2O), low glucose (<40 mg/dL), high protein pressure (>350 mm H2O), low glucose (<40 mg/dL), high protein

file:///C:/Users/vikas/OneDrive/Desktop/print.html 61/207
5/10/2021 print.html
(>200 mg/dL), and neutrophilic leukocytosis (>1000/mm3). (>200 mg/dL), and neutrophilic leukocytosis (>1000/mm3).

How do you treat Lyme disease? How do you treat age-related macular degeneration? How do you treat conjunctivitis?

file:///C:/Users/vikas/OneDrive/Desktop/print.html 62/207
5/10/2021 print.html
Non-invasive disease: PO doxycycline or PO amoxicillin if Wet: intraocular VEGF inhibitors (ranibizumab, bevacizumab, Viral: self-limited (adenovirus) or topical antiviral (HSV)
pregnant pegaptanib) or laser coagulation (or photodynamic therapy) Acute bacterial: topical broad spectrum antibiotics
Invasive disease: IV ceftriaxone Dry: antioxidants therapy (with vitamins A, C, E, β-carotene, and Neisserial bacterial: IM ceftriaxone + PO azithromycin
zinc) Chlamydial bacterial: PO azithromycin
Confined to skin: 10 days Allergic, intermittent: topical antihistamines
Spread beyond skin: 20-30 days Them antioxidants ain't doing shit, tho... Allergic, recurrent: azelastine or olopatadine, which are mast cell
Non-CNS complications: 30-60 days stabilizers
CNS complications: IV for 4 weeks
Mast cell stabilizing agents, such as olopatadine and azelastine, are
Doxycycline is an excellent treatment option for most patients as it indicated for treatment of allergic conjunctivitis.
has the advantage of simultaneously preventing or treating
coexisting human granulocytic anaplasmosis, an infection also Symptoms of allergic and viral conjunctivitis are often similar, but
carried by I. scapularis. However, doxycycline is contraindicated allergic conjunctivitis typically occurs episodically with a shorter
in young children as well as pregnant and lactating women because duration of symptoms.
it can cause permanent discoloration of teeth and retardation of
skeletal development in exposed children and fetuses. Oral Although many viral conjunctivitis symptoms may be due to
amoxicillin is the treatment of choice in pregnant and lactating release of histamine from mast cells, mast cell stabilizers have not
women as well as children age <8 years. The rash and been well studied for this purpose and are generally not
constitutional symptoms should resolve within 3 weeks of needed. Prednisolone ophthalmic drops are approved for use in
treatment. Pregnant patients should be reassured that Lyme disease ocular inflammatory and allergic conditions. They are relatively
is not known to cause congenital anomalies or fetal contraindicated in viral conjunctivitis and are not needed as
demise. Although intravenous third-generation cephalosporins (eg, symptoms will resolve spontaneously.
ceftriaxone and cefotaxime) are effective in treating early localized
Lyme disease, patients would be unnecessarily hospitalized and
exposed to broader-spectrum antibiotics. These should be reserved
for meningitis, encephalopathy, or carditis.

Treatment of early localized Lyme: Clinical trials have shown


that oral doxycycline, amoxicillin, and cefuroxime have equivalent
efficacy for treating early localized Lyme disease. Doxycycline is
often used because it is effective in treating potential coexisting
Anaplasma phagocytophilum infection, which is also transmitted
by the Ixodes tick. However, doxycycline is contraindicated in
children age <8 years and pregnant women. It can slow bone
growth in exposed fetuses and cause enamel hypoplasia and
permanent teeth stains during tooth development in young
children. Therefore, oral amoxicillin or cefuroxime is the treatment
of choice for children age <8 years and pregnant women.

Treatment of early or late disseminated Lyme: intravenous


ceftriaxone is reserved for Lyme meningitis and heart block,
manifestations of early disseminated Lyme disease.

file:///C:/Users/vikas/OneDrive/Desktop/print.html 63/207
5/10/2021 print.html

file:///C:/Users/vikas/OneDrive/Desktop/print.html 64/207
5/10/2021 print.html

How do you treat conjunctivitis?

Viral: self-limited (adenovirus) or topical antiviral (HSV)


Acute bacterial: topical broad spectrum antibiotics
Neisserial bacterial: IM ceftriaxone + PO azithromycin
Chlamydial bacterial: PO azithromycin
Allergic, intermittent: topical antihistamines
Allergic, recurrent: azelastine or olopatadine, which are mast cell
stabilizers How do you treat epileptic patients with the following recurrent
seizures?
Mast cell stabilizing agents, such as olopatadine and azelastine, are
indicated for treatment of allergic conjunctivitis. Focal seizures: lamotrigine, levetiracetam, or phenytoin
How do you treat convulsive status epilepticus?
Generalized tonic-clonic seizures: valproate or lamotrigine
Symptoms of allergic and viral conjunctivitis are often similar, but Generalized typical absence seizures: ethosuximide or valproate
IV lorazepam + IV fosphenytoin → repeat and intubate → IV
allergic conjunctivitis typically occurs episodically with a shorter Generalized atypical absence, myoclonic, or atonic
midazolam → IV propofol → IV phenobarbital
duration of symptoms. seizures: valproate, lamotrigine, or topiramate
Number one priority is to control ABCs.
Although many viral conjunctivitis symptoms may be due to Number one priority is to control ABCs.
release of histamine from mast cells, mast cell stabilizers have not
IV lorazepam alternatives: IV diazepam, IM midazolam (no IV
been well studied for this purpose and are generally not The SANAD trial concluded that the drug of first choice should
access)
needed. Prednisolone ophthalmic drops are approved for use in be lamotrigine for focal epilepsy and valproate for generalized
IV fosphenytoin alternatives: IV phenytoin, IV valproic acid, IV
ocular inflammatory and allergic conditions. They are relatively epilepsy. Because the SANAD trial was unblinded, however, there
levetiracetam
contraindicated in viral conjunctivitis and are not needed as was potential for bias. Also, it provided only sparse data regarding
Theoretically, midazolam, propofol, and phenoarbital are all on the
symptoms will resolve spontaneously. the potential of rare, often idiosyncratic, serious adverse events
same level, but in order of safety, use in that order.
(eg, potential for teratogenicity with valproate). These results also
do not account for other patient-specific preferences regarding the
likelihood of different side effects, need for drug monitoring,
potential for drug interactions, and dosing frequency.

How do you treat glaucoma? How do you treat glaucoma? How do you treat idiopathic intracranial hypertension?

Chronic open-angle, conservative: topical prostaglandins Chronic open-angle, conservative: topical prostaglandins Idiopathic cases: weight loss and acetazolamide (to ↓CSF
(latanoprost) +/- topical β-blocker (timolol) or topical α2-agonist (latanoprost) +/- topical β-blocker (timolol) or topical α2-agonist production, +/- furosemide)
(apraclonidine) (apraclonidine) Progressive vision loss: surgical intervention with optic nerve
Chronic open-angle, alternative: laser trabeculoplasty Chronic open-angle, alternative: laser trabeculoplasty sheath decompression (or lumbar peritoneal shunting)
Acute angle-closure, initial: topical β-blocker (timolol) + Acute angle-closure, initial: topical β-blocker (timolol) +
topical α2-agonist (apraclonidine) + topical cholinergic topical α2-agonist (apraclonidine) + topical cholinergic Acetazolamide inhibits choroid plexus carbonic anhydrase, thereby
(pilocarpine, once IOP <40 mm Hg) + systemic carbonic (pilocarpine, once IOP <40 mm Hg) + systemic carbonic decreasing CSF production and IH.
anhydrase inhibitor (acetazolamide) anhydrase inhibitor (acetazolamide)
Acute angle-closure, definitive: laser iridotomy Acute angle-closure, definitive: laser iridotomy Patients with IIH typically present with holocranial headache,
vision changes (blurry vision and diplopia), and pulsatile tinnitus
Mneumonic: "Activate α and block β" Mneumonic: "Activate α and block β" ("whooshing" sound in the ears). Although IIH is frequently seen
in young obese women, it has also been associated with certain
For patients who do not wish to, or are unable to take eye drops, For patients who do not wish to, or are unable to take eye drops, medications (eg, isotretinoin, tetracyclines, growth hormone,
excessive vitamin A).
studies have found laser trabeculoplasty to be equivalent in safety studies have found laser trabeculoplasty to be equivalent in safety
and efficacy to monotherapy. and efficacy to monotherapy.
Initial evaluation of IIH includes complete ocular examination and
Goal in open angle is to constrict the pupils. Goal in closed angle Goal in open angle is to constrict the pupils. Goal in closed angle neuroimaging to exclude secondary causes of intracranial
hypertension (eg, mass, hemorrhage, cerebral vein thrombosis).
is to decrease the intraocular pressure. is to decrease the intraocular pressure.
Magnetic resonance imaging possibly with magnetic resonance
file:///C:/Users/vikas/OneDrive/Desktop/print.html 65/207
5/10/2021 print.html
venography (to rule out cerebral vein thrombosis) is the preferred
Iridotomy is curative, reduces the risk of recurrence, and is often Iridotomy is curative, reduces the risk of recurrence, and is often imaging modality. Empty sella is present in about 70% of patients
also performed on the unaffected eye for prophylactic reasons. also performed on the unaffected eye for prophylactic reasons. but is not diagnostic. Lumbar puncture (LP) is then indicated to
document elevated opening pressure.
Open angle glaucoma is usually asymptomatic in the earlier stages. Open angle glaucoma is usually asymptomatic in the earlier stages.
It is more common in African Americans, and has an increased It is more common in African Americans, and has an increased Papilledema is not a contraindication to LP unless the patient has
prevalence in those with a family history of glaucoma and prevalence in those with a family history of glaucoma and evidence of obstructive or noncommunicating hydrocephalus
diabetes. There is a gradual loss of peripheral vision over a period diabetes. There is a gradual loss of peripheral vision over a period and/or a space-occupying lesion with/without mass effect or
of years, and eventual tunnel vision. On examination, the of years, and eventual tunnel vision. On examination, the midline shift. Pseudotumor cerebri causes a communicating
intraocular pressure is high. There may be cupping of the optic intraocular pressure is high. There may be cupping of the optic hydrocephalus (ie, pressures in the ventricular and subarachnoid
disc with loss of peripheral vision. Annual examination in high- disc with loss of peripheral vision. Annual examination in high- spaces are equilibrated with the lumbar cistern) and therefore LP is
risk populations is of great benefit in prevention of the condition. risk populations is of great benefit in prevention of the condition. considered safe. LP is performed with the patient in the lateral
Beta-blockers such as Timolol eye drops are effective in the initial Beta-blockers such as Timolol eye drops are effective in the initial decubitus position with legs extended. An opening pressure of
management of the patient. Laser trabeculoplasty is used as an management of the patient. Laser trabeculoplasty is used as an >250 mm H2O is considered abnormal and in the appropriate
adjunctive measure. If there is a continuous increase in intraocular adjunctive measure. If there is a continuous increase in intraocular clinical setting is diagnostic of IIH. Cerebrospinal fluid studies are
pressure, surgical trabeculectomy is done. pressure, surgical trabeculectomy is done. normal in IIH patients.

Angle closure glaucoma occurs with closure of a pre-existing Angle closure glaucoma occurs with closure of a pre-existing Short-term use of corticosteroids or serial lumbar puncture (LP)
narrow anterior chamber angle. It predominantly occurs in people narrow anterior chamber angle. It predominantly occurs in people can serve as bridging therapy for patients awaiting definitive
aged 55-70 years. It presents with an acute onset of severe eye pain aged 55-70 years. It presents with an acute onset of severe eye pain surgical treatment. These are not recommended as primary
and blurred vision associated with nausea and vomiting. It usually and blurred vision associated with nausea and vomiting. It usually intervention due to side effects associated with long-term
occurs following pupillary dilation, which may occur in darkened occurs following pupillary dilation, which may occur in darkened corticosteroid use and the complications associated with
movie theaters, during times of stress, or due to drug intake. movie theaters, during times of stress, or due to drug intake. LPs. Mannitol is an osmotic diuretic used to reduce intracranial
Examination reveals a red eye with steamy cornea and moderately Examination reveals a red eye with steamy cornea and moderately pressure associated with cerebral edema. It is an intravenous
dilated pupil that is non reactive to light. The anterior chamber is dilated pupil that is non reactive to light. The anterior chamber is infusion and requires careful monitoring of renal function and
shallow with inflammatory changes. Tonometry reveals increased shallow with inflammatory changes. Tonometry reveals increased electrolytes. Mannitol is used in the inpatient setting for those with
intraocular pressure. Intravenous acetazolamide (with subsequent intraocular pressure. Intravenous acetazolamide (with subsequent severely elevated intracranial pressure (obtunded patients). It is not
oral administration) may lower the intraocular pressure. Permanent oral administration) may lower the intraocular pressure. Permanent used in the management of IIH.
cure is offered with laser peripheral iridotomy. cure is offered with laser peripheral iridotomy.

How do you treat intracerebral hemorrhage? How do you treat intracerebral hemorrhage? How do you treat lower back pain?

file:///C:/Users/vikas/OneDrive/Desktop/print.html 66/207
5/10/2021 print.html
Acute stabilizing measures: fluid replacement, potential intubation, Acute stabilizing measures: fluid replacement, potential intubation, Musculoskeletal: NSAIDs + exercise
head elevation (30°) head elevation (30°) Herniation, mild-to-moderate: NSAIDs + bed rest
Medical therapy: gradual reduction of BP with Medical therapy: gradual reduction of BP with Herniation, severe: surgery
IV labetalol, nicardipine, enalapril, or hydralazine; maintain IV labetalol, nicardipine, enalapril, or hydralazine; maintain Osteophyte: surgery
normal glucose and monitor anticoagulation parameters normal glucose and monitor anticoagulation parameters Compression fracture: surgery
Surgical intervention: craniotomy and subsequent clot evacuation Surgical intervention: craniotomy and subsequent clot evacuation Spinal stenosis: NSAIDs +/- surgery
in patients with signs of brain herniation, obstructive in patients with signs of brain herniation, obstructive
hydrocephalus, or hemorrhage extension > 3 cm. hydrocephalus, or hemorrhage extension > 3 cm. Patients with acute (<4 weeks) LBP should be advised to maintain
moderate activity with short courses of acetaminophen or
↑BP → ↑ICP → further bleeding. But ↓BP → ↓cerebral blood ↑BP → ↑ICP → further bleeding. But ↓BP → ↓cerebral blood nonsteroidal anti-inflammatory drugs(NSAIDs) to provide pain
flow, worsening the neurologic deficits. Treat when BP >160/105 flow, worsening the neurologic deficits. Treat when BP >160/105 relief. Most patients will have resolution without any additional
to 180/105. to 180/105. intervention. However, those with subacute (4-12 weeks) or
chronic LBP are more likely to continue to have recurrent or
Mannitol used only when ICP is elevated, not prophylactically. Mannitol used only when ICP is elevated, not prophylactically. persistent pain. In patients with chronic LBP, exercise has proven
Use of steroids is harmful. Surgery not helpful in most cases of Use of steroids is harmful. Surgery not helpful in most cases of to be beneficial in reducing pain and improving function. Patients
ICH. ICH. often start with a supervised exercise program that emphasizes
stretching and strengthening of the back muscles. Aerobic exercise
is also helpful. Subsequently, patients can transition to a home
exercise program, which should be continued on a long-term basis.
Short courses of acetaminophen or NSAIDs can be used
intermittently. Some patients also may benefit from tricyclic
antidepressants or duloxetine, but opioids, benzodiazepines, and
muscle relaxants are not advised for routine use. Prolonged activity
restriction in patients with uncomplicated LBP is associated with
increased long-term pain and stiffness. Patients should be
counseled to maintain normal activity. Spinal manipulation can
provide small benefits for LBP that has failed other interventions.
However, the benefits are transient, and patients should first
optimize their exercise regimen.

file:///C:/Users/vikas/OneDrive/Desktop/print.html 67/207
5/10/2021 print.html

How do you treat lower back pain? How do you treat rocky mountain spotted fever and How do you treat the following headaches?
ehrlichiosis?
Musculoskeletal: NSAIDs + exercise Tension headache: NSAIDs
Herniation, mild-to-moderate: NSAIDs + bed rest RMSF: doxycycline or chloramphenicol Cluster headache, acute: 100% O2 (by facemask) +/- sumatriptan
Herniation, severe: surgery Ehrlichiosis: doxycycline or chloramphenicol (or ergotamine)
Osteophyte: surgery Cluster headache, prophylaxis: prednisone → verapamil
Compression fracture: surgery Migraine, acute, mild: NSAIDs

file:///C:/Users/vikas/OneDrive/Desktop/print.html 68/207
5/10/2021 print.html
Spinal stenosis: NSAIDs +/- surgery Chloramphenicol is an antibiotic that acts on bacterial 50S Migraine, acute, severe: ergotamine (+/- NSAID) or sumatriptan
ribosome. (+/- NSAID)
Patients with acute (<4 weeks) LBP should be advised to maintain Migraine, acute, refractory: assess for rebound analgesic
moderate activity with short courses of acetaminophen or Ehrlichiosis is a tick-borne infection caused by Ehrlichia headaches (likely not a migraine)
nonsteroidal anti-inflammatory drugs(NSAIDs) to provide pain chaffeensis and E ewingii. These bacteria are carried by ticks, Migraine, prophylaxis: lifestyle changes + β-blocker (or TCA)
relief. Most patients will have resolution without any additional including the lone star tick (Amblyomma americanum), found in
intervention. However, those with subacute (4-12 weeks) or the southeastern and south central United States. The principal DO NOT COMBINE SUMATRIPTAN AND ERGOTAMINE
chronic LBP are more likely to continue to have recurrent or reservoir is the white tail deer. Ehrlichiosis is characterized by an TO ABORT A SEVERE MIGRAINE! Rather, combine with an
persistent pain. In patients with chronic LBP, exercise has proven acute febrile illness with malaise and altered mental status. NSAID for greater efficacy. Don't use sumatriptan if patient has
to be beneficial in reducing pain and improving function. Patients Ehrlichiosis is not often associated with a rash (<30% in adults) CAD.
often start with a supervised exercise program that emphasizes and is described as "Rocky Mountain spotted fever (RMSF)
stretching and strengthening of the back muscles. Aerobic exercise without the spots." Neurologic symptoms may include confusion,
is also helpful. Subsequently, patients can transition to a home mental status changes, clonus, and neck stiffness. Doxycycline is
exercise program, which should be continued on a long-term basis. the appropriate treatment and is often initiated empirically when
Short courses of acetaminophen or NSAIDs can be used the diagnosis is suspected while confirmatory testing is pending.
intermittently. Some patients also may benefit from tricyclic
antidepressants or duloxetine, but opioids, benzodiazepines, and In 90% of RMSF patients, a maculopapular rash (involving the
muscle relaxants are not advised for routine use. Prolonged activity soles and palms) occurs a few days after the onset of fever.
restriction in patients with uncomplicated LBP is associated with Doxycycline is the first-line treatment. Chloramphenicol is a
increased long-term pain and stiffness. Patients should be second-line treatment option for ehrlichiosis and RMSF. It is
counseled to maintain normal activity. Spinal manipulation can associated with blood dyscrasias and is not commonly used.
provide small benefits for LBP that has failed other interventions.
However, the benefits are transient, and patients should first
optimize their exercise regimen.

file:///C:/Users/vikas/OneDrive/Desktop/print.html 69/207
5/10/2021 print.html

How do you treat the following headaches? How does distractibility (eg, performing mental tasks) affect the How does herpes encephalitis affect reflexes?
intensity of Parkinsonian tremors?
Tension headache: NSAIDs Hyperreflexia
Cluster headache, acute: 100% O2 (by facemask) +/- sumatriptan Increase
(or ergotamine) The majority of acute viral encephalitis cases are due to unknown
Cluster headache, prophylaxis: prednisone → verapamil Parkinsonian tremors are more pronounced with distractibility (eg, causes. However, herpes viruses (eg, herpes simplex, varicella,
Migraine, acute, mild: NSAIDs performing mental tasks) and re-emergence (tremor goes away Epstein-Barr) most commonly cause encephalitis in

file:///C:/Users/vikas/OneDrive/Desktop/print.html 70/207
5/10/2021 print.html
Migraine, acute, severe: ergotamine (+/- NSAID) or sumatriptan with movement and then re-emerges when the movement is immunocompetent adults. Patients typically present with fever,
(+/- NSAID) stopped). The tremor of Parkinson's disease characteristically altered mental status, agitation, headaches, and seizures that can
Migraine, acute, refractory: assess for rebound analgesic occurs during rest and improves with activity (essential tremor has rapidly progress to complete unresponsiveness or coma.
headaches (likely not a migraine) the opposite pattern.) The Parkinson's tremor usually has a Examination can show neurologic abnormalities, including
Migraine, prophylaxis: lifestyle changes + β-blocker (or TCA) frequency of 4-5 Hz, and is often the first manifestation of hemiparesis, cranial nerve palsies, and exaggerated deep-
Parkinson's disease. The tremor often involves one hand before the tendon reflexes. Clinical signs of meningeal irritation (eg,
DO NOT COMBINE SUMATRIPTAN AND ERGOTAMINE other or presents asymmetrically. Rigidity is also characteristic of photophobia, nuchal rigidity) are usually absent in patients with
TO ABORT A SEVERE MIGRAINE! Rather, combine with an Parkinson's disease. The absence of additional symptoms such as pure encephalitis.
NSAID for greater efficacy. Don't use sumatriptan if patient has postural instability or gait abnormality should not exclude the
CAD. diagnosis of Parkinson's. Lumbar puncture usually shows cerebrospinal fluid (CSF) findings
of elevated white blood cell count with lymphocytic
predominance, normal glucose, and normal to slightly elevated
protein concentration (generally 100 mg/dL). Most patients have
an increased red blood cell count as well. Diagnosis is usually
confirmed by detecting viral DNA by polymerase chain reaction
(PCR) in the CSF. Empiric treatment with intravenous acyclovir
should be started while awaiting PCR results as encephalitis is
often associated with significant morbidity and mortality.

How is Leprosy diagnosed? How is Lyme disease diagnosed and confirmed? How is proprioception evaluated?

file:///C:/Users/vikas/OneDrive/Desktop/print.html 71/207
5/10/2021 print.html
Full thickness biopsy of the skin lesion edge Early localized Lyme: clinically with erythema migrans Passively moving the distal phalange of a digit up and down and
Early disseminated Lyme: ELISA detection of IgM or IgG having patients identify the direction of movement with their eyes
Leprosy is a chronic granulomatous disease of the skin and followed by confirmation with Western blotting closed.
peripheral nerves caused by the acid-fast bacillus Mycobacterium Late Lyme: ELISA detection of IgM or IgG followed by
leprae. Transmission is thought to occur via respiratory droplets, confirmation with Western blotting Also assessed with the Romberg test in which patients are
although cases are occasionally linked to close contact with a nine- Lyme meningitis: lumbar puncture observed for unsteadiness as they stand with their feet together,
banded armadillo. Infections are rare in the United States and arms to the sides, and eyes closed. Patients with impaired
occur primarily in immigrants or travelers to endemic regions (eg, Diagnosis of early localized Lyme: based solely on the presence proprioception (eg, vitamin B12 deficiency, tabes dorsalis) have
Asia, Africa, South America). of the trademark rash in the context of recent travel to Lyme- difficulty determining joint position and may lose balance during
endemic areas. Primary EM usually presents within 1 month of the Romberg testing.
Manifestations include >1 chronic, anesthetic, macular (often tick bite, appearing on the arms and moist areas of the body such
hypopigmented) skin lesions with raised, well-demarcated borders. as the axillae, groin, or trunk. The round or oval macule is initially
Nearby nerves often become nodular and tender, and segmental uniformly red and can develop a zone of central clearing as it
demyelination may result in loss of sensation and motor function. expands, giving it the classic bull's eye appearance. It is not
Diagnosis is clinical in endemic regions, but in the United States particularly painful or itchy and can become as large as 20 cm in
patients usually require a full-thickness biopsy of the skin lesion diameter. Blood cultures for B burgdorferi are not available in
edge (as M leprae is not culturable). Patients with minimal most clinical laboratories and are not recommended. Serology is
lesions ("paucibacillary") are treated with dapsone and rifampin; not recommended in early localized disease as it is too insensitive
those with extensive lesions ("multibacillary") require the addition and many patients would be seronegative.
of clofazimine. Lesions often take months or years to heal
completely. Diagnosis of early disseminated or late Lyme: after the onset of
EM, IgM antibodies to B burgdorferi usually appear within 1-2
weeks, and IgG antibodies typically appear within 2-6 weeks.
However, serology (Western blot, ELISA) should be performed in
patients with signs of early disseminated or late Lyme disease.

Treatment of early localized Lyme: Clinical trials have shown


that oral doxycycline, amoxicillin, and cefuroxime have equivalent
efficacy for treating early localized Lyme disease. Doxycycline is
often used because it is effective in treating potential coexisting
Anaplasma phagocytophilum infection, which is also transmitted
by the Ixodes tick. However, doxycycline is contraindicated in
children age <8 years and pregnant women. It can slow bone
growth in exposed fetuses and cause enamel hypoplasia and
permanent teeth stains during tooth development in young
children. Therefore, oral amoxicillin or cefuroxime is the treatment
of choice for children age <8 years and pregnant women.

Treatment of early or late disseminated Lyme: intravenous


ceftriaxone is reserved for Lyme meningitis and heart block,
manifestations of early disseminated Lyme disease.

file:///C:/Users/vikas/OneDrive/Desktop/print.html 72/207
5/10/2021 print.html

file:///C:/Users/vikas/OneDrive/Desktop/print.html 73/207
5/10/2021 print.html

How soon should you administer intravenous heparin in the early


How long should aspirin be avoided following thrombolytic phase of ischemic stroke.
therapy?
Intravenous heparin is not recommended in the early acute phase
24 hours of ischemic stroke due to its limited efficacy and greater risk of
bleeding complications. ( ͡~ ͜ʖ ͡°)
Aspirin and statins are commonly used for primary and secondary
ischemic stroke prevention. Aspirin may help reduce clot Aspirin and statins are commonly used for primary and secondary
propagation and risk of stroke recurrence if administered within 48 ischemic stroke prevention. Aspirin may help reduce clot
hours of stroke onset; however, intravenous alteplase is more propagation and risk of stroke recurrence if administered within 48
effective than antiplatelet therapy at achieving clot recanalization hours of stroke onset; however, intravenous alteplase is more
and neurologic recovery. Aspirin should be held for 24 hours effective than antiplatelet therapy at achieving clot recanalization
following thrombolytic therapy due to the increased risk of and neurologic recovery. Aspirin should be held for 24 hours
intracranial hemorrhage. Intravenous heparin is not recommended following thrombolytic therapy due to the increased risk of
in the early acute phase of ischemic stroke due to its limited intracranial hemorrhage. Intravenous heparin is not recommended
efficacy and greater risk of bleeding complications. in the early acute phase of ischemic stroke due to its limited
efficacy and greater risk of bleeding complications.
How long can postconcussive syndrome last for?

Weeks to months

TBI of any severity can lead to (a few hours or days later)


postconcussive syndrome, which is clinically described by the
following constellation of symptoms: headache, confusion,
amnesia, difficulty concentrating or with multitasking, vertigo,
mood alteration, sleep disturbance, and anxiety. Typically, these
symptoms resolve with symptomatic treatment within a few weeks
to months following TBI; however, some patients may have
persistent symptoms lasting >6 months.

Huntington chorea is a(n) autosomal dominant genetic disease Huntington's chorea is most likely to present with behavioral Huntington's presents with chorea and leads to dementia,
with a(n) terrible prognosis that exhibits anticipation and is caused (judgment, executive function, awareness and attention) changes. psychosis, and often death by suicide.
by trinucleotide repeats.
file:///C:/Users/vikas/OneDrive/Desktop/print.html 74/207
5/10/2021 print.html
Huntington's chorea is an autosomal dominant neurodegenerative Chorea: purposeless ballistic movements.
Anticipation: occurring earlier and earlier in subsequent
disorder. Patients usually present in their forties or fifties with
generations as the number of trinucleotide repeats expands. chorea and/or behavioral disturbance. Chorea is characterized by Gait is usually unsteady and irregular.
sudden, jerky and irregular movements of the extremities.
Onset is between 30 and 50 years of age. Symptoms worsen
Impaired judgment, executive function, awareness and attention
steadily, with 15 years being the typical duration from onset to
occur at an early stage. Memory impairment is a late finding.
death. Depression, irritability and social withdrawal are also common.
Atrophy of the caudate nucleus and putamen is a characteristic
CAG repeats on chromosome 4 → loss of GABA-producing
feature, and is seen on neuroimaging as enlargement of lateral
neurons in the striatum
ventricles.

file:///C:/Users/vikas/OneDrive/Desktop/print.html 75/207
5/10/2021 print.html

Idiopathic intracranial hypertension presents with pulsatile If a patient has a TIA, what is the risk that they will have a stroke If t-PA is contraindicated in an ischemic stroke patient, give them
tinnitus and most often affects the abducens nerve. in the next five years? aspirin. Next line is clopidogrel, followed by ticlopidine.

Although the abducens nerve is the most commonly affected in 30% Anticoagulants have not been proven to have efficacy in acute
IIH, elevated intracranial pressure in the setting of trauma will stroke. Aspirin and statins are commonly used for primary and
likely affect the oculomotor nerve (CNIII) secondary to uncal Once a patient has a TIA, there is a high risk of stroke in secondary ischemic stroke prevention. Aspirin may help reduce
(transtentorial) herniation. subsequent months. clot propagation and risk of stroke recurrence if administered
within 48 hours of stroke onset; however, intravenous alteplase is
Patients with IIH typically present with holocranial headache, more effective than antiplatelet therapy at achieving clot
vision changes (blurry vision and diplopia), and pulsatile tinnitus recanalization and neurologic recovery. Aspirin should be held for
("whooshing" sound in the ears). Although IIH is frequently seen 24 hours following thrombolytic therapy due to the increased risk
in young obese women, it has also been associated with certain of intracranial hemorrhage. Intravenous heparin is not
medications (eg, isotretinoin, tetracyclines, growth hormone, recommended in the early acute phase of ischemic stroke due to its
excessive vitamin A). limited efficacy and greater risk of bleeding complications.

Initial evaluation of IIH includes complete ocular examination and


neuroimaging to exclude secondary causes of intracranial
hypertension (eg, mass, hemorrhage, cerebral vein thrombosis).
Magnetic resonance imaging possibly with magnetic resonance
venography (to rule out cerebral vein thrombosis) is the preferred
imaging modality. Empty sella is present in about 70% of patients
but is not diagnostic. Lumbar puncture (LP) is then indicated to
document elevated opening pressure.

Papilledema is not a contraindication to LP unless the patient has


evidence of obstructive or noncommunicating hydrocephalus
and/or a space-occupying lesion with/without mass effect or
midline shift. Pseudotumor cerebri causes a communicating
hydrocephalus (ie, pressures in the ventricular and subarachnoid
spaces are equilibrated with the lumbar cistern) and therefore LP is
considered safe. LP is performed with the patient in the lateral
decubitus position with legs extended. An opening pressure of
>250 mm H2O is considered abnormal and in the appropriate
clinical setting is diagnostic of IIH. Cerebrospinal fluid studies are
normal in IIH patients.

file:///C:/Users/vikas/OneDrive/Desktop/print.html 76/207
5/10/2021 print.html

file:///C:/Users/vikas/OneDrive/Desktop/print.html 77/207
5/10/2021 print.html

If the CT scan is negative, but clinical suspicion is high, diagnose


a subarachnoid hemorrhage with a(n) lumbar puncture, which
should reveal either ↑RBCs or xanthochromia. To pinpoint the
bleeding site for surgical clamping, order a(n) cerebral angiogram. In a hemorrhagic cerebellar stroke, where will there be
weakness?
Xanthochromia: yellow color of CSF; results from RBC lysis.
Face (there is usually no hemiparesis)
Patients with a confirmed diagnosis should be evaluated further
Cerebellar hemorrhage presents with occipital headache,
with cerebral angiography and treated with surgery (craniotomy
nausea/vomiting, dizziness, and cerebellar signs (eg, ataxia,
with aneurysm clipping) or endovascular methods (coiling/stenting
of the aneurysm). dysmetria). There is usually no hemiparesis. Early diagnosis is
essential as emergency decompression may be life-saving.

In a patient with recent tonic-clonic seizure, the presence of blood


on urinalysis suggests what?

Rhabdomyolysis

NOTE: This is not actually blood in the urine, but a standard


urinalysis is not able to distinguish between hemoglobin and
myoglobin due to chemical similarities, but microscopic exam of
the urine for RBCs can add clarity. Large amounts of myoglobin
in the urinary system can result in tubular injury and acute renal
failure.

In acute angle-closure glaucoma, the lens is located anatomically In patients with ischemic stroke, antihypertensives should be In shingles, the first manifestation is pain.
more forward. avoided unless their BP exceeds 220/120, they have a significant
indication, or they are receiving t-PA. Pain from shingles may precede the onset of the classic vesicular
Acute angle-closure glaucoma (ACG) is due to sudden narrowing rash by several days, during which the diagnosis may not be
or closure of the anterior chamber angle. In patients with ACG, the Antihypertensive therapy indicated for acute MI, aortic dissection, obvious. The possibility of shingles should be considered in
lens is located anatomically more forward and rests against the iris. severe heart disease, and hypertensive encephalopathy patients with regional pain who have no conclusive evidence of
This partially covers the anterior chamber angle and prevents
normal flow of aqueous humor (through the pupil and into the
anterior chamber), thereby increasing intraocular pressure. Sudden
angle closure typically occurs as a response to pupillary dilation
from medications (eg, anticholinergics such as tolterodine,
sympathomimetics, and decongestants) or other stimuli (eg, dim
light).

Acute ACG is more common in women (especially age >40),


Asian and Inuit populations, and individuals with farsightedness.
Patients typically develop rapid onset of severe eye pain and may
see halos around lights. The affected eye will appear injected and

file:///C:/Users/vikas/OneDrive/Desktop/print.html 78/207
5/10/2021 print.html
the pupil will be dilated and poorly responsive to light. Patients disease in the local internal organs.
can also develop tearing and severe headache ("never had a
headache like this before") with subsequent nausea and vomiting
as the intraocular pressure increases. Untreated acute ACG can
lead to severe and permanent vision loss within 2-5 hours of
symptom onset.

In spinal cord compression, early signs include hypoactive In which neurofibromatosis are disfiguring cutaneous Indications for urgent dialysis
reflexes, while late signs include hyperactive reflexes. neurofibromas more common?
AEIOU

file:///C:/Users/vikas/OneDrive/Desktop/print.html 79/207
5/10/2021 print.html
Spinal cord compression (SCC) is characterized by injury to the Type I
following:
1. Descending corticospinal tracts (lower-extremity weakness and
loss of rectal tone)
2. Ascending sensory spinothalamic tracts (the sensory level is
often 2 spinal cord segments below the level of lesion)
3. Descending autonomics in the reticulospinal tract (urinary
retention/bladder flaccidity/bladder shock)

By contrast, lesions affecting the conus medullaris will typically


cause back pain with bladder and rectal dysfunction, but weakness
and sensory loss are less common. Cauda equina (spinal roots)
syndrome may cause radicular pain, lower-extremity weakness,
saddle anesthesia, and bowel/bladder dysfunction, but this
typically affects the sacral roots. A sensory level at the umbilicus
excludes cauda equina syndrome.

SCC is most commonly caused by disk herniation, compression


fractures, or malignancy. Immediate neurosurgical evaluation is
warranted to avoid permanent neurologic dysfunction. Typically,
neuroimaging (ie, MRI of the spine) is performed while awaiting
surgical consultation. High-dose glucocorticoids are often used,
especially in cases of suspected malignant compression.

Holy shit...

file:///C:/Users/vikas/OneDrive/Desktop/print.html 80/207
5/10/2021 print.html

file:///C:/Users/vikas/OneDrive/Desktop/print.html 81/207
5/10/2021 print.html

Inherited causes of ataxia include Friedreich ataxia and ataxia


telangiectasia, which are both autosomal recessive. Intracerebral hemorrhage is most commonly caused by a(n)
sudden increase in BP.
Friedreich ataxia = ataxia + nystagmus + impaired vibratory sense
+ impaired proprioception HTN causes a rupture of small vessels deep within the brain
Ataxia telangiectasia = Friedreich ataxia + telangiectasia parenchyma. Chronic HTN causes degeneration of small arteries,
leading to microaneurysms, which can rupture easily.

Other causes: ischemic stroke, amyloid angiopathy, anticoagulant


use, brain tumors, and AV malformations

Inflammation of basal brain in tb meningitis can cause

Cranial nerve palsies

Intracerebral hemorrhage most commonly occurs in the basal Intracranial hypertension presents with headaches that are Is a dark red glow on funduscopy more associated with retinal
ganglia and cerebellar nuclei (second most common). worst at night (likely due to lying supine) and the Cushing reflex, detachment or vitreous hemorrhage?
which consists of hypertension, bradycardia, and respiratory
The most common sites for hypertensive hemorrhage in depression. Vitreous hemorrhage
descending order include the basal ganglia (putamen), cerebellar
nuclei, thalamus, pons, and cerebral cortex. Patients with Patients with intracranial hypertension typically present with Fundoscopy in patients with retinal detachment typically
intraparenchymal hemorrhage usually present with gradual onset headache (worse at night), nausea/vomiting, and mental status demonstrates an elevated retina with folds and/or a tear.
of symptoms (minutes to a few hours) in contrast to brain changes (eg, decreased level of consciousness, cognitive
embolism or subarachnoid hemorrhage, in which symptoms are dysfunction). Patients may also have focal neurologic symptoms Vitreous hemorrhage typically presents as a sudden loss of vision
abrupt. Typical features of cerebellar hemorrhage include occipital (eg, vision changes, unsteady gait) and seizure. Symptoms can and onset of floaters. The most common cause is diabetic
headache (may radiate to neck/shoulders), neck stiffness (due to worsen with maneuvers that further increase intracranial pressure retinopathy. Ophthalmoscopy reveals loss of fundus details,
extension of blood into the 4th ventricle), nausea/vomiting, and (eg, leaning forward, Valsalva, cough). Cushing reflex floating debris and a dark red glow. An important diagnostic clue
nystagmus. Patients may also have ipsilateral hemiataxia of the (hypertension, bradycardia, respiratory depression) is a worrisome is that the fundus is hard to visualize, and even if it is visualized,
trunk (cerebellar vermis) and/or limbs (cerebellar hemispheres) as finding suggestive of brainstem compression. details may be obscured. Immediate ophthalmologic consultation
the corticopontocerebellar fibers decussate twice. Hemiparesis and is required. For patients with underlying medical conditions,
sensory loss are usually absent, but extension of the bleed to the conservative treatment (i.e., upright position during sleep, which
brainstem can cause cranial neuropathies (eg, ipsilateral facial enhances settling of the hemorrhage) is
nerve palsy). Further hematoma expansion may lead to brainstem recommended. Proliferative diabetic retinopathy in the initial stage
compression resulting in stupor, coma, and ultimately death. Early is asymptomatic. Patients may later complain of decreased visual
diagnosis with noncontrast head CT scan is crucial as emergency acuity. Neovascularization is the hallmark of proliferative diabetic
decompression may be life-saving, especially if the hemorrhage is retinopathy. The other findings are vitreous hemorrhage and
>3 cm. macular edema. These changes may lead to retinal detachment (the
separation of the inner layers of the retina), which is also
associated with metabolic disorders (e.g., diabetes mellitus),
trauma (including ocular surgery), vascular disease, myopia, or
degeneration. Patients with retinal detachment typically complain
of vision loss, photopsia with showers of floaters.

file:///C:/Users/vikas/OneDrive/Desktop/print.html 82/207
5/10/2021 print.html

file:///C:/Users/vikas/OneDrive/Desktop/print.html 83/207
5/10/2021 print.html

Ischemic stroke is caused by either emboli from the heart, Lacunar strokes present with contralateral pure motor deficits, if Lacunar strokes present with contralateral pure motor deficits, if
thrombi due to atherosclerotic plaques in large- and medium-sized the lesion involves the internal capsule; or contralateral pure the lesion involves the internal capsule; or contralateral pure
arteries in the brain, or lacunar strokes which result from sensory deficits, if the lesion involves the thalamus. sensory deficits, if the lesion involves the thalamus.
thickening of small vessel walls in association with hypertension.
Lacunar infarcts typically produce neurologic deficits over minutes Lacunar infarcts typically produce neurologic deficits over minutes
Non-vascular causes of stroke: low CO, anoxia to hours and symptoms may follow a stuttering course. The to hours and symptoms may follow a stuttering course. The
Other sources of emboli: ICA, aorta, peripheral veins → septal absence of cortical signs (eg, aphasia, agnosia, neglect, apraxia, absence of cortical signs (eg, aphasia, agnosia, neglect, apraxia,
defects hemianopia), seizures, and mental status changes (eg, stupor, hemianopia), seizures, and mental status changes (eg, stupor,
Lacunar stokes often affect the subcortical stuctures. Diabetes is coma) supports a deep/subcortical localization. Lacunar strokes are coma) supports a deep/subcortical localization. Lacunar strokes are
another important risk factor for lacunar strokes. small (<15 mm in diameter) subcortical infarcts resulting from small (<15 mm in diameter) subcortical infarcts resulting from
occlusion of deep penetrating branches of the major cerebral occlusion of deep penetrating branches of the major cerebral

file:///C:/Users/vikas/OneDrive/Desktop/print.html 84/207
5/10/2021 print.html
arteries (eg, anterior cerebral, middle cerebral, basilar). Affected arteries (eg, anterior cerebral, middle cerebral, basilar). Affected
areas typically include the basal ganglia, subcortical white matter areas typically include the basal ganglia, subcortical white matter
(internal capsule, corona radiata), and pons. Lacunar infarcts are (internal capsule, corona radiata), and pons. Lacunar infarcts are
most commonly associated with chronic hypertension, which leads most commonly associated with chronic hypertension, which leads
to arteriolar sclerosis and vessel occlusion (hypertensive to arteriolar sclerosis and vessel occlusion (hypertensive
vasculopathy). Other risk factors include diabetes, smoking, vasculopathy). Other risk factors include diabetes, smoking,
advanced age, and increased LDL cholesterol. advanced age, and increased LDL cholesterol.

file:///C:/Users/vikas/OneDrive/Desktop/print.html 85/207
5/10/2021 print.html

Lambert-eaton myasthenic syndrome is caused by Lhermitte sign aka barber chair phenomenon is a(n) electrical Lyme disease is caused by Borrelia burgdorferi transmission from
autoantibodies directed against presynaptic calcium channels and sensation that runs down the back and into the limbs with neck Ixodidae scapularis ticks.
presents with hyporeflexia and proximal muscle weakness that flexion in patients with multiple sclerosis.
improves with repeated muscle stimulation.

file:///C:/Users/vikas/OneDrive/Desktop/print.html 86/207
5/10/2021 print.html
Lhermitte phenomenon, sometimes called the barber chair Tick is hosted by white-footed mice, white-tailed deer, and,
Lambert-Eaton syndrome (LE) is a neuromuscular junction
phenomenon, is an electrical sensation that runs down the back and unfortunately, nasty-ass humans.
disorder that involves autoantibodies to presynaptic Ca-channels.
into the limbs. The sensation can feel like it goes up or down the
In LE syndrome, weakness progressively improves with spine. It is generally considered uncomfortable. In many patients,
activity. LE also has autonomic symptoms like dry mouth or it is elicited by bending the head forward. It can also be evoked
impotence. LE syndrome starts weak but gets prgoressively
when a practitioner pounds on the posterior cervical spine while
stronger, thus illiciting initial reflexes is diffilcut. In contrast, MG the neck is flexed; this is caused by involvement of the posterior
has muscle weakness that worsens with repetitive motions, but can columns.
illicit strong initial reflexes.

file:///C:/Users/vikas/OneDrive/Desktop/print.html 87/207
5/10/2021 print.html

Marked imrovement of gait after spinal fluid removal is seen in Meningitis is diagnosed with CSF analysis when a lumbar Meningitis presents with the classic triad of fever, nuchal rigidity,
puncture is safe, and a(n) blood culture + CT scan when a lumbar and headache.
NPH puncture is unsafe.
Other clinical features: headache, nausea, vomiting, malaise,
If CT scan is negative, then do a LP photophobia, ↑ICP, cranial nerve palsies, Kernig sign,
and Brudzinski sign

file:///C:/Users/vikas/OneDrive/Desktop/print.html 88/207
5/10/2021 print.html

Migraines are classically associated with a(n) aura and a(n) Migraines are classically associated with a(n) aura and a(n) Multi-infarct dementia is a(n) step-wise decline due to serial
prodromal phase. The clinical features can be remembered with the prodromal phase. The clinical features can be remembered with the strokes.
mneumonic "POUND": mneumonic "POUND":

Pulsatile Pulsatile
One day in duration (4-72 hours) One day in duration (4-72 hours)
Unilateral Unilateral
Nausea and vomiting Nausea and vomiting
Disabling Disabling

Aura: usually visual (bilateral homonymous scotoma: bright, Aura: usually visual (bilateral homonymous scotoma: bright,
flashing, crescent-shaped images with jagged edges often appear flashing, crescent-shaped images with jagged edges often appear
on a page, obscuring the underlying print) or neurologic (sensory, on a page, obscuring the underlying print) or neurologic (sensory,
hemiparesis, dysphasia) lasting 10 to 20 minutes hemiparesis, dysphasia) lasting 10 to 20 minutes

Prodrome consists of symptoms of excitation or inhibition of the Prodrome consists of symptoms of excitation or inhibition of the
CNS: elation, excitability, increased appetite and craving for CNS: elation, excitability, increased appetite and craving for
certain foods (especially sweets); alternatively, depression, certain foods (especially sweets); alternatively, depression,
irritability, sleepiness, and fatigue may be manifested. irritability, sleepiness, and fatigue may be manifested.

file:///C:/Users/vikas/OneDrive/Desktop/print.html 89/207
5/10/2021 print.html

file:///C:/Users/vikas/OneDrive/Desktop/print.html 90/207
5/10/2021 print.html

file:///C:/Users/vikas/OneDrive/Desktop/print.html 91/207
5/10/2021 print.html

Multiple sclerosis is diagnosed with a demyelinating


lesions/plaques on MRI; and oligoclonal bands of IgG and T-
lymphocyte predominance in CSF via lumbar puncture.

Multiple sclerosis (MS), an autoimmune inflammatory


Multiple sclerosis presents with neurologic symptoms separated
demyelinating disorder of the central nervous system (CNS),
by time and space; the primary complaint is often blurry vision due
typically presents with neurologic deficits disseminated in space
to optic neuritis or internuclear ophthalmoplegia due to medial
and time (eg, sensory loss/paresthesias of the extremities,
longitudinal fasciculus lesion.
dizziness, optic neuritis) in women age 15-50. Symptoms may
Multiple sclerosis is a(n) autoimmune disease that leads to
worsen during exposure to high temperatures, such as moving to
demyelination of nerve nuclei, primarily involving white matter, Also affects pyramidal pathway, leading to motor weakness or
Arizona in June (Uhthoff phenomenon).
preferentially near the lateral ventricles or corpus callosum. spasticity; cerebellar pathway, leading to ataxia, intention tremor,
and dysarthria; and posterior columns, leading to decreased
When MS is suspected, magnetic resonance imaging (MRI) of the
sensation in the limbs.
brain and spine is typically performed to support the diagnosis. T2-
weighted imaging usually demonstrates multifocal, ovoid,
Fatigue is a common complaint. Other symptoms: loss of bladder
subcortical hypo-/hyperintense lesions located in the white matter
control, autonomic involvement, late cerebral involvement,
of the CNS, particularly the periventricular and subpial white
neuropathic pain.
matter of the cerebrum (corpus callosum), optic nerves, brainstem,
and spinal cord. Lumbar puncture can be performed after an MRI
to document oligoclonal IgG bands in the cerebrospinal fluid
(found in 85%-95% of patients with MS), especially in cases in
which the diagnosis is not clear.

Multiple sclerosis presents with neurologic symptoms separated Myastenia gravis is a(n) autoimmune disease that presents Myastenia gravis is a(n) autoimmune disease that presents
by time and space; the primary complaint is often blurry vision due in old men and young women. It targets post-synaptic in old men and young women. It targets post-synaptic
to optic neuritis or internuclear ophthalmoplegia due to medial acetylcholine receptors, typically causing fatigability in muscles acetylcholine receptors, typically causing fatigability in muscles
longitudinal fasciculus lesion. that are stimulated repeatedly, such as the eyes and throat. that are stimulated repeatedly, such as the eyes and throat.

Also affects pyramidal pathway, leading to motor weakness or Fatigability because Ach-blockade requires increased Ach Fatigability because Ach-blockade requires increased Ach
spasticity; cerebellar pathway, leading to ataxia, intention tremor, concentrations to make the muscles work, depleting concentrations to make the muscles work, depleting
and dysarthria; and posterior columns, leading to decreased reserves. Myasthenia gravis is a neuromuscular disorder caused by reserves. Myasthenia gravis is a neuromuscular disorder caused by
sensation in the limbs. autoantibodies (originating in the thymus) directed against autoantibodies (originating in the thymus) directed against
nicotinic acetylcholine receptors at the neuromuscular junction. nicotinic acetylcholine receptors at the neuromuscular junction.
Fatigue is a common complaint. Other symptoms: loss of bladder Patients classically experience fluctuating and fatigable weakness Patients classically experience fluctuating and fatigable weakness
control, autonomic involvement, late cerebral involvement, involving the ocular (eg, diplopia, ptosis) and/or bulbar (eg, involving the ocular (eg, diplopia, ptosis) and/or bulbar (eg,
neuropathic pain. dysphagia, dysarthria) muscles. Individuals with typical clinical dysphagia, dysarthria) muscles. Individuals with typical clinical
features of myasthenia gravis should undergo confirmatory features of myasthenia gravis should undergo confirmatory
immunologic testing for acetycholine receptor antibodies (highly immunologic testing for acetycholine receptor antibodies (highly
file:///C:/Users/vikas/OneDrive/Desktop/print.html 92/207
5/10/2021 print.html
specific). Those with negative acetycholine receptor antibodies specific). Those with negative acetycholine receptor antibodies
should subsequently be checked for muscle-specific tyrosine should subsequently be checked for muscle-specific tyrosine
kinase antibodies. If the diagnosis remains unclear, kinase antibodies. If the diagnosis remains unclear,
electrophysiologic studies (eg, repetitive nerve stimulation, single- electrophysiologic studies (eg, repetitive nerve stimulation, single-
fiber electromyography) may be helpful. Patients with established fiber electromyography) may be helpful. Patients with established
myasthenia gravis should receive chest imaging (eg, CT scan or myasthenia gravis should receive chest imaging (eg, CT scan or
MRI) to evaluate for thymoma as thymectomy can lead to long- MRI) to evaluate for thymoma as thymectomy can lead to long-
term improvement. The edrophonium (Tensilon) test may be used term improvement. The edrophonium (Tensilon) test may be used
to support the diagnosis of myasthenia gravis. However, it is less to support the diagnosis of myasthenia gravis. However, it is less
specific than the acetylcholine receptor antibody assay and is specific than the acetylcholine receptor antibody assay and is
difficult to interpret in patients with only minor symptoms. In this difficult to interpret in patients with only minor symptoms. In this
bedside study, the infusion of edrophonium (a short-acting bedside study, the infusion of edrophonium (a short-acting
acetylcholinesterase inhibitor) prolongs the presence of acetylcholinesterase inhibitor) prolongs the presence of
acetylcholine in the neuromuscular junction, resulting in an acetylcholine in the neuromuscular junction, resulting in an
immediate improvement in muscle strength. immediate improvement in muscle strength.

file:///C:/Users/vikas/OneDrive/Desktop/print.html 93/207
5/10/2021 print.html

file:///C:/Users/vikas/OneDrive/Desktop/print.html 94/207
5/10/2021 print.html

Myastenia gravis is diagnosed initially with anti-Ach-R Myastenia gravis presents with progressive weakness with intact Myotonic discharges on EMG are seen in which muscular
antibodies or, if negative (with high clinical suspicion), anti- reflexes that is relieved by rest. It can involve the diaphragm and dystrophy?
muscle specific kinase antibodies; and confirmed with a(n) EMG intercostal muscles leading to myasthenic crisis, which is a
showing ↓amplitude on repeated stimulation. Once diagnosed, medical emergency. Myotonic dystrophy
order a chest CT to rule out thymoma.
MG has muscle weakness that worsens with repetitive motions, Myotonic dystrophy is an autosomal dominant disease that
If AChR antibodies are negative, consider testing for anti-MUSK but can illicit strong initial reflexes. Contrast this with Lambert- generally presents in the teenage years with muscle weakness,
antibodies (muscle specific kinase). CPK levels are typically Eaton syndrome (LE), a neuromuscular junction disorder that myotonia, cataracts, and cardiac conduction abnormalities. The
normal in patients with myasthenia gravis. If the diagnosis remains involves autoantibodies to presynaptic Ca-channels. In LE muscular groups that are most affected include the facial muscles,
unclear, electrophysiologic studies (eg, repetitive nerve syndrome, weakness progressively improves with activity. LE also intrinsic hand muscles, and ankle dorsiflexors. Myotonia, or
stimulation, single-fiber electromyography) may be helpful. has autonomic symptoms like dry mouth or impotence. LE delayed muscle relaxation, is a prominent feature of the disease
Patients with established myasthenia gravis should receive chest syndrome starts weak but gets prgoressively stronger, thus and manifests as a myotonic pattern on electromyography. In
imaging (eg, CT scan or MRI) to evaluate for thymoma as illiciting initial reflexes is difficult. contrast, a myopathic pattern would be seen in DMD and
thymectomy can lead to long-term improvement. The BMD.
edrophonium (Tensilon) test may be used to support the diagnosis
of myasthenia gravis. However, it is less specific than the
acetylcholine receptor antibody assay and is difficult to interpret in
patients with only minor symptoms. In this bedside study, the
infusion of edrophonium (a short-acting acetylcholinesterase
inhibitor) prolongs the presence of acetylcholine in the
neuromuscular junction, resulting in an immediate improvement in
muscle strength.

file:///C:/Users/vikas/OneDrive/Desktop/print.html 95/207
5/10/2021 print.html

Narcolepsy is an inherited disorder of REM sleep regulation that Normal pressure hydrocephalus presents with the triad of Normal pressure hydrocephalus presents with the triad of

file:///C:/Users/vikas/OneDrive/Desktop/print.html 96/207
5/10/2021 print.html
results in excessive daytime sleepiness. urge urinary incontinence, ataxic gait, and dementia, that improves urge urinary incontinence, ataxic gait, and dementia, that improves
with serial lumbar punctures and is treated with a(n) VP shunt. with serial lumbar punctures and is treated with a(n) VP shunt.
Characterized by the following features:
1. Involuntary sleep attacks at any time of the day that last several "Wet, wobbly, and wacky" "Wet, wobbly, and wacky"
minutes. The problem is ↑CSF, not ↑ICP The problem is ↑CSF, not ↑ICP
2. Cataplexy (loss of muscle tone w/ intense emotional stimulus) Damage to the cortico-cortical white matter fibers of the frontal Damage to the cortico-cortical white matter fibers of the frontal
3. Sleep paralysis, in which patient cannot move when waking up lobe, as seen in normal pressure hydrocephalus, can lead to gait lobe, as seen in normal pressure hydrocephalus, can lead to gait
4. Hypnagogic hallucinations that are vivid hallucinations; apraxia (Bruns ataxia). Strength, coordination, and sensory apraxia (Bruns ataxia). Strength, coordination, and sensory
"dreams" while awake functions are intact, but there is difficulty in initiation of forward functions are intact, but there is difficulty in initiation of forward
movement of the feet when they are in contact with the ground movement of the feet when they are in contact with the ground
("magnetic gait"). ("magnetic gait").

NPH is a clinical syndrome accompanied by increased ventricular NPH is a clinical syndrome accompanied by increased ventricular
size without persistent elevations in intracranial pressure. size without persistent elevations in intracranial pressure.
Symptoms result from distortion of the periventricular brain Symptoms result from distortion of the periventricular brain
matter. Affected patients typically have a slow, broad-based, matter. Affected patients typically have a slow, broad-based,
shuffling gait. Gait difficulties appear early, consist of slow, short shuffling gait. Gait difficulties appear early, consist of slow, short
steps with decreased height, and appear as though the patient's feet steps with decreased height, and appear as though the patient's feet
are stuck to the ground (magnetic gait). Upper motor neuron signs are stuck to the ground (magnetic gait). Upper motor neuron signs
(eg, spasticity, hyperreflexia) may also be evident. As the disease (eg, spasticity, hyperreflexia) may also be evident. As the disease
advances, cognitive disturbances (eg, difficulty concentrating, lack advances, cognitive disturbances (eg, difficulty concentrating, lack
of attention) arise. The dementia in NPH is characterized by of attention) arise. The dementia in NPH is characterized by
memory loss but no focal neurologic changes. Urinary memory loss but no focal neurologic changes. Urinary
incontinence is classic, but fecal incontinence may develop at incontinence is classic, but fecal incontinence may develop at
advanced stages. advanced stages.

The prevailing theory for the pathogenesis of NPH is that patients The prevailing theory for the pathogenesis of NPH is that patients
have a transient increase in intracranial pressure that causes have a transient increase in intracranial pressure that causes
ventricular enlargement. After the ventricles enlarge, the pressure ventricular enlargement. After the ventricles enlarge, the pressure
returns to normal. The initial increase in ventricular size may be returns to normal. The initial increase in ventricular size may be
due to either diminished CSF absorption at the arachnoid villi or due to either diminished CSF absorption at the arachnoid villi or
obstructive hydrocephalus. The diagnosis is made by obstructive hydrocephalus. The diagnosis is made by
demonstrating enlarged ventricles on CT or MRI, plus normal demonstrating enlarged ventricles on CT or MRI, plus normal
opening pressure on lumbar puncture. Treatment consists of opening pressure on lumbar puncture. Treatment consists of
sequential CSF removal. This is initially done by performing sequential CSF removal. This is initially done by performing
serial large volume lumbar punctures. A ventriculoperitoneal serial large volume lumbar punctures. A ventriculoperitoneal
shunt may be placed if the patient's symptoms respond to shunt may be placed if the patient's symptoms respond to
lumbar punctures. lumbar punctures.

Urinary incontinence is a late manifestation and is commonly of Urinary incontinence is a late manifestation and is commonly of
the urgency type, reflecting decreased inhibitory control of the urgency type, reflecting decreased inhibitory control of
bladder contractions by the frontal lobe. Expanded ventriculi bladder contractions by the frontal lobe. Expanded ventriculi
place traction on cortical efferent and afferent fibers (corona place traction on cortical efferent and afferent fibers (corona
radiata). This traction disrupts the periventricular pathways that radiata). This traction disrupts the periventricular pathways that
transmit impulses from the cortex to the sacral micturition transmit impulses from the cortex to the sacral micturition
center. Later in the disease process, lack of inhibition from the center. Later in the disease process, lack of inhibition from the
cerebral cortex leads to frequent and uncontrolled micturition, or cerebral cortex leads to frequent and uncontrolled micturition, or
urge incontinence. The bladder fills with urine and empties urge incontinence. The bladder fills with urine and empties
reflexively when full. The patient has no sensation of bladder reflexively when full. The patient has no sensation of bladder
fullness and no control over bladder function. Voluntary relaxation fullness and no control over bladder function. Voluntary relaxation
of the urethral sphincter remains intact. of the urethral sphincter remains intact.

file:///C:/Users/vikas/OneDrive/Desktop/print.html 97/207
5/10/2021 print.html

file:///C:/Users/vikas/OneDrive/Desktop/print.html 98/207
5/10/2021 print.html

file:///C:/Users/vikas/OneDrive/Desktop/print.html 99/207
5/10/2021 print.html

file:///C:/Users/vikas/OneDrive/Desktop/print.html 100/207
5/10/2021 print.html

On CT scan, ischemic stroke typically appears as an area Open-angle glaucoma is characterized by a(n) gradual increase in Optic neuritis is caused by immune mediated demyelination of
of hypodensity. intraocular pressure that compresses vascular supply to the optic the optic nerve, diagnosed with a(n) MRI of the orbits and brain
nerve and presents with no symptoms early and loss of bilateral and fundoscopy revealing nothing ( ͡~ ͜ʖ ͡°), and is treated with IV
Rather than a hyperdensity affecting a vascular distribution. peripheral vision late. corticosteroids.

Absence of symptoms early in the course can lead to delay in Optic neuritis is an inflammatory demyelination of the optic nerve.
diagnosis and "silent" progression. Progressive and insidious Optic neuritis is thought to be immune-mediated and is most
visual field loss usually sparing central vision until end-stage commonly seen in women age 20-40. Symptoms develop acutely
disease. and usually include monocular vision loss, pain with eye
movement, and "washed-out" color vision. Examination typically
Secondary clogging of the trabecular meshwork or reduced reveals an afferent pupillary defect (paradoxical pupillary dilation
drainage → gradual ↑ in IOP → vascular compression → ischemia of the affected eye with the swinging-flashlight test) and central
to the optic nerve → progressive visual impairment. scotoma. Funduscopy is usually normal as inflammation occurs
behind the optic nerve head.
Open angle glaucoma is usually asymptomatic in the earlier stages.
It is more common in African Americans, and has an increased Optic neuritis is strongly associated with multiple sclerosis and is
prevalence in those with a family history of glaucoma and frequently the heralding symptom of this condition. As such,
diabetes. There is a gradual loss of peripheral vision over a period patients are typically evaluated with MRI of the orbits and the
of years, and eventual tunnel vision. On examination, the brain to look for other areas of inflammation.
intraocular pressure is high. There may be cupping of the optic
disc with loss of peripheral vision. Annual examination in high-
risk populations is of great benefit in prevention of the condition.
Beta-blockers such as Timolol eye drops are effective in the initial
management of the patient. Laser trabeculoplasty is used as an
adjunctive measure. If there is a continuous increase in intraocular
pressure, surgical trabeculectomy is done.

Angle closure glaucoma is characterized by a sudden onset of


symptoms such as blurred vision, severe eye pain, nausea, and
vomiting. Examination reveals a red eye with a hazy cornea and a
fixed, dilated pupil. Macular degeneration affects central
vision. Cataract is a vision-impairing disease characterized by
progressive thickening of the lens. Oxidative damage of the lens
occurs with aging and leads to cataract formation. Patients usually
complain of blurred vision, problems with nighttime driving, and
glare. Definitive treatment is lens extraction.

file:///C:/Users/vikas/OneDrive/Desktop/print.html 101/207
5/10/2021 print.html

Orthostatic hypotension presents in individuals with autonomic


Orthostatic hypotension is defined as systolic BP ↓20, diastolic
dysfunction, hypovolemia, or abundant inflammatory cytokines.
BP ↓10, or HR ↑10 when going from laying down to standing up.
Orbital cellulitis is typically unilateral (can rapidly progress to Sepsis/anaphylaxis have abundant inflammatory cytokines, which
bilateral). Treat with volume + pressors
fight against the sympathetic tone. Treat with volume + pressors
It is manifested by an abrupt onset of fever, proptosis, restriction of
extraocular movements and swollen, red eyelids. Cranial nerves
III, IV, V, and VI pass through the cavernous sinus, which has
anastomoses crossing midline. As a result, unilateral symptoms
(eg, headache, binocular palsies, periorbital edema, hypoesthesia,
or hyperesthesia in V1/V2 distribution) can rapidly
become bilateral.

Other than improperly canned foods, what other food is highly Parkinson disease results from a loss of dopaminergic neurons in Patients with Bell's palsy who have sparing of the forehead

file:///C:/Users/vikas/OneDrive/Desktop/print.html 102/207
5/10/2021 print.html
associated with foodborne botulism? the substantia nigra and locus ceruleus in the midbrain. The muscles require brain imaging to evaluate an intracranial lesion.
diagnosis is made with the clinical presence of two out of three
Cured fish cardinal signs (eg, rest tremor, rigidity and bradykinesia). The rapid onset of unilateral upper and lower facial weakness is
consistent with Bell's palsy, an acute peripheral neuropathy of
Foodborne botulism, a potentially life-threatening illness, is caused Onset is usually 50+ years old. Lab studies play no role in cranial nerve VII (lesion below the pons). Common findings of the
by the ingestion of food contaminated with botulinum toxin diagnosis. Parkinsonism refers to symptoms and signs of affected side include inability to raise the eyebrow or close the eye,
formed through germination of Clostridium botulinum spores. Parkinson disease and can result from many conditions (e.g., drooping of the mouth corner (with the mouth drawn to the
Improperly canned foods and cured fish provide the appropriate medications). unaffected side), and disappearance of the nasolabial fold. Patients
environment for spore germination. Ingestion results in absorption afflicted with Bell's palsy may also have decreased tearing,
of a preformed toxin that inhibits presynaptic acetylcholine release The three cardinal signs of PD are rest tremor, rigidity and hyperacusis, and/or loss of taste sensation over the anterior two-
at the neuromuscular junction. bradykinesia. The presence of two of these three signs suggests a thirds of the tongue.
clinical diagnosis of PD, which can be further confirmed by
Symptoms develop acutely (within 36 hours) following toxin physical examination. To date, there are no imaging or laboratory It is very important that patients be assessed for symmetry by
ingestion. Prodromal symptoms can include gastrointestinal tests that can be used to confirm this diagnosis with any greater raising their eyebrows due to bilateral upper-motor-neuron
discomfort, dry mouth, and sore throat. Patients classically develop accuracy than physical examination. The most common presenting innervation to the forehead. Forehead muscle sparing is suggestive
bilateral cranial neuropathies often leading to blurred vision (due sign of PD is an asymmetric resting tremor in the distal part of an of an intracranial lesion and would warrant brain imaging to
to fixed pupillary dilation), diplopia, facial weakness, dysarthria, upper extremity. evaluate for ischemia or tumors.
and dysphagia. Symmetric descending muscle weakness usually
follows, initially affecting the upper extremities and trunk and then Physical examination findings that contribute to a clinical Patients with Bell's palsy present with acute onset of symptoms
the lower extremities. Respiratory failure requiring mechanical diagnosis of PD include: Tremor (a resting 4 to 6 Hz tremor with a (<2 days) with maximum clinical paralysis apparent within 3
ventilation can occur secondary to diaphragmatic paralysis. "pill rolling" quality that frequently first manifests in one hand, weeks. For most patients, partial or complete recovery occurs
and may then slowly generalize to involve the other side of the within 6 months.
Patients with suspected foodborne botulism should be checked for body and the lower extremities), rigidity [baseline increased
toxin in the blood. In addition to supportive care, treatment for resistance to passive movement about a joint which may be
adults and children age >1 involves passive immunity through uniform (lead pipe) or oscillating
administration of horse-derived antitoxin. (cogwheel)], bradykinesia [difficulty initiating movements, as
when starting to walk or rising from a chair; narrow-based,
shuffling gait with short strides and without arm swing (festinating
gait); micrographia (small handwriting); hypomimia (decreased
facial expression); and hypophonia (soft speech)] and postural
instability (flexed axial posture, loss of balance during turning or
stopping, loss of balance when pushed slightly from a stationary
bipedal stance, and frequent falls)

file:///C:/Users/vikas/OneDrive/Desktop/print.html 103/207
5/10/2021 print.html

file:///C:/Users/vikas/OneDrive/Desktop/print.html 104/207
5/10/2021 print.html

Patients with hypertensive retinopathy typically have gradual


vision loss.

Patients with first time seizures donot require anti epileptic Patients with first time seizures donot require anti epileptic
medications unless there is reccurent seizure activity medications unless there is reccurent seizure activity

Pharmacologic treatment
of narcolepsy involves daytime modafinil and nighttime sodium
oxybate (GHB).

Modafinil is a stimulant with fewer adverse effects compared to


amphetamines; believed to enhance dopaminergic
signaling. Traditional stimulants like amphetamines and
methylphenidate are second-line. Physiological tremors are worsened by emotional and physical ( ͡~
͜ʖ ͡°) stress, caffeine, toxic/metabolic derangements, and drug
withdrawal.
Peripheral neuropathy diagnosis is made by
Physiologic tremor is a benign tremor with low amplitude and high
EMG and Nerve conduction studies
frequency (12-14 Hz) that is not visible under ordinary
conditions. Physiologic tremors are postural tremors (eg, holding
arms outstretched) and infrequently occur with action.

file:///C:/Users/vikas/OneDrive/Desktop/print.html 105/207
5/10/2021 print.html

Pick's disease is a dementia characterized by frontal-temporal


degeneration that takes personality and language first, then comes
back for memory. Diagnosis is clinical and there's no treatment. Pick's disease is more frequently seen in females.

Serotonergic medications (e.g., SSRIs, trazodone) may help reduce Pick's disease (frontotemporal dementia) is similar in presentation
disinhibition, anxiety, impulsivity, repetitive behaviors, and eating to Alzheimer's disease, except that it presents at an earlier age. It is
disorders. Poliomyelits presents with asymmetric muscle weakness, absent
seen more frequently in females, and frequently causes personality
DTRs, and NORMAL ( ͡~ ͜ʖ ͡°) sensation due to invasion of anterior
changes due to involvement of the frontal lobes; however, it does
horn cells by the poliovirus.
not cause any extrapyramidal symptoms, and is not genetically
acquired.
Bulbar involvement can lead to respiratory and CV impairment.
No treatment → vaccinate your damn kids.
Patients with this condition exhibit primitive reflexes associated
with decreased inhibition from the impaired frontal lobe. Although
these reflexes can be seen in the advanced stages of many other
types of dementia, they occur relatively early in the course of FTD.

Prevention of thrombotic strokes includes reduction of


atherosclerotic risk factors and administration of antiplatelet (eg,
aspirin) therapy, as well as performing carotid endarterectomy in
patients that are symptomatic

Embolic strokes: reduction of atherosclerotic risk factors and


aspirin
Lacunar strokes: control of HTN
Primary CNS lymphoma is caused by EBV and will be found
Anticoagulation with heparin should be considered in those with near the ventricles in the brain.
acute cardioembolism from atrial fibrillation. Secondary
prevention with aspirin is indicated in most patients with acute The presence of EBV DNA in the CSF is specific for this Primitive reflexes occuring early in the disease is characteristic of
ischemic stroke, and anticoagulation with heparin can be condition. MRI reveals a weakly ring-enhancing mass that is what type of dementia?
considered in those with acute cardioembolism (eg, due to atrial usually solitary and periventricular. The majority of primary
fibrillation). CT should be performed prior to initiating either of CNS lymphomas arise from B-cells. Diffuse large B-cell Frontotemporal dementia (Pick's disease) and vascular dementia
these agents to rule out hemorrhage. Carotid Doppler and lymphoma is the most common subtype. The cells of CNS
echocardiogram may be considered after head CT to evaluate for lymphomas are commonly positive for the B-cell markers CD20 Pick's disease is similar in presentation to Alzheimer's disease,
possible embolic sources in patients with ischemic stroke. and CD79a. Primary CNS lymphomas are universally associated except that it presents at an earlier age. It is seen more frequently
with the Epstein-Barr virus (EBV). in females, and frequently causes personality changes due to
involvement of the frontal lobes; however, it does not cause any
extrapyramidal symptoms, and is not genetically acquired.

Patients with this condition exhibit primitive reflexes associated


with decreased inhibition from the impaired frontal lobe. Although
these reflexes can be seen in the advanced stages of many other
types of dementia, they occur relatively early in the course of FTD.

file:///C:/Users/vikas/OneDrive/Desktop/print.html 106/207
5/10/2021 print.html

Rabies presents, in progresive order, with pain at bite site,


Rabies is diagnosed by histological identification of Negri bodies. Rabies is prophylaxed after exposure with human rabies prodromal symptoms, encephalitis, hydrophobia, and ascending
immunoglobulin and the antirabies vaccine. paralysis (if it doesn't kill you first).
Other diagnostic tools: ID in infected tissue or saliva, fourfold
increase in serum antibody titers, PCR detection of virus RNA. Devastating, deadly viral encephalitis contracted from a bite or
scratch by an infected animal or a corneal transplant

Restless leg syndrome can be caused secondarily by iron Reversible causes of dementia that must be ruled out include Risk factors for Alzheimer disease include age, family history,
deficiency anemia or chronic kidney disease via uremia and is metabolic disorders, such as hypothyroidism or Vitamin and Down syndrome.
treated with dopamine agonists (pramipexole, ropinirole when B12 deficiency; subdural hematoma; syphillis; uremia; cirrhosis;
persistent. and psuedo-dementia. All Down syndrome >40 have Alzheimer disease

Restless legs syndrome (RLS) is a neurologic disorder affecting


nearly 5%-10% of the population. RLS can be primary (idiopathic)
or secondary to causes such as iron deficiency anemia, pregnancy,
uremia (chronic kidney disease or end-stage renal disease),
diabetes mellitus, Parkinson disease, multiple sclerosis, or
medications (eg, antidepressants, metoclopramide). Patients
typically develop an irresistible urge to move the legs along with
dysesthesias (eg, crawling and itching feelings) that are partially
relieved by movement and worsened by inactivity or rest.
Symptoms usually worsen in the evening or night and frequently
interfere with duration and quality of sleep. Patients typically do
not have pain or symptoms of peripheral neuropathy (eg,
numbness/tingling). Evaluation should exclude secondary causes
and treat other medical conditions that may present
similarly. Patients with intermittent or mild symptoms usually
improve with nonpharmacologic therapy, including leg massage,
heating pads, regular exercise, and avoiding aggravating factors
(eg, sleep deprivation). Oral iron supplementation is recommended
for serum ferritin levels ≤75 µg/L. Some patients may require
intermittent pharmacotherapy; those with persistent/moderate-to-
severe symptoms usually require pharmacotherapy. Dopamine-
agonists (eg, pramipexole, ropinirole) are preferred. RLS patients
with comorbid insomnia, chronic pain syndrome, or anxiety may
benefit from alpha-2-delta calcium channel ligands (eg,
gabapentin) over dopaminergic agents. Although gabapentin is
structurally similar to γ-aminobutyric acid (GABA), it binds to
voltage-gated calcium channels instead of GABA receptors.
Patients with refractory symptoms with these treatments may
respond to opioids.

file:///C:/Users/vikas/OneDrive/Desktop/print.html 107/207
5/10/2021 print.html

file:///C:/Users/vikas/OneDrive/Desktop/print.html 108/207
5/10/2021 print.html

Spinal stenosis is worsened by extension of the spine.

Lumbar spinal stenosis, a common cause of pain in the low back


and legs, may result from any condition that narrows the spinal
canal and compresses nerve roots. The most common cause is
degenerative joint disease (DJD). In DJD, disc herniation and facet
osteophytes impinge upon the spinal cord. This is most commonly
seen in middle-aged and elderly patients. Flexion of the spine
causes widening of the spinal canal, while extension causes
narrowing of the canal. Therefore, the symptoms of lumbar spinal
stenosis are exacerbated by extension of the spine (e.g. standing
and downhill walking), and improved by flexion of the spine (e.g.
sitting and uphill walking). Weakness, sensory loss,
Spared eye injury is also known as sympathetic ophthalmia. numbness/tingling, and discomfort of the legs may all be seen.
Neurologic examination may be entirely normal and only 10% of
Sympathetic ophthalmia is also known as "spared eye injury." It is patients have a positive straight leg test. The diagnosis is
characterized by an immune-mediated inflammation of one eye confirmed radiologically. MRI demonstrates bony anatomy, neural
Rocky mountain spotted fever is caused by Rickettsia structures, and soft tissue, making it the study of choice for
(the sympathetic eye) after a penetrating injury to the other eye.
rickettsii infection during a(n) tick bite and presents with a rash suspected lumbar spinal stenosis.
The typical manifestation is anterior uveitis, but panuveitis,
that starts peripherally and spreads centrally, in addition to sudden
papillary edema, and blindness may develop. The
constitutional symptoms. In lumbar spinal stenosis, the ligamentum flavum undergoes
pathophysiological mechanism is believed to be the uncovering of
'hidden' antigens. Some antigens contained within the eye are hypertrophy. This is loss of height places a disproportionate load
May lead to interstitial pneumonitis. on the posterior aspect of the spinal column, leading to formation
protected from immunologic recognition by natural barriers.
Breaking these barriers results in the uncovering of 'hidden' of facet joint osteophytes and hypertrophy of the ligamentum
antigens. An immune response against these antigens can involve flavum
autoantibodies as well as a cell-mediated reaction.

Spinal stenosis presents with neuropathic claudication in the legs Spontaneous lobar hemorrhages are most commonly caused Sturge-Weber syndrome presents with the presence of capillary
and butt that is present when upright. by amyloid angiopathy and affect the parietal and occipital lobes angiomatoses of the pia mater and facial vascular nevi.
of the brain.
The pain of spinal stenosis is often associated with activity, as Acquired disease, port wine stain.
lumbar extension during walking worsens the narrowing of the Hemorrhage usually occurs during routine activity and most often Epilepsy and retardation usually present.
canal. This is termed neurogenic claudication and can resemble the involves the occipital and parietal lobes.
lower extremity vascular claudication seen in obstructive
atherosclerotic arterial disease. Both spinal stenosis/neurogenic Progressive confusion + lethargy over several hours and
claudication and vascular claudication are generally seen in older hyperdensity (red arrow) on head CT scan are consistent with an
patients with multiple atherosclerotic risk factors. acute right parietal lobe hemorrhage, most likely due to cerebral
amyloid angiopathy. Amyloid angiopathy is the most common
Neuropathic claudication is thought to be the result of increased cause of spontaneous lobar hemorrhage, particularly in adults age
metabolic demands of a compressed and ischemic spinal cord. This >60. It occurs as a consequence of β-amyloid deposition in the
ischemia compromises the ability to walk longer distances. This walls of small- to medium-size cerebral arteries, resulting in vessel
differs from vascular claudication which is a result of arterial wall weakening and predisposition to rupture. The disease is not
vessel narrowing and impaired metabolic demands of muscle associated with systemic amyloidoses; rather, the amyloidogenic
tissue. proteins are usually the same as those seen in Alzheimer dementia.

file:///C:/Users/vikas/OneDrive/Desktop/print.html 109/207
5/10/2021 print.html
Parietal hemorrhages can cause contralateral hemisensory loss
(due to primary somatosensory cortex injury) and contralateral
hemineglect if the parietal association cortex (particularly in the
nondominant hemisphere) is affected. Hematoma expansion can
lead to elevated intracranial pressure, resulting in impaired
consciousness, confusion, headache, and nausea/vomiting.

file:///C:/Users/vikas/OneDrive/Desktop/print.html 110/207
5/10/2021 print.html

Subclavian steal syndrome is caused by stenosis of the


subclavian artery proximal to the origin of the vertebral artery and
results in the distal subclavian artery stealing blood flow from the
basilar system via the ipsilateral vertebral artery during times of
exercise, decreasing cerebral blood flow.

Subarachnoid hemorrhage is most commonly caused by a(n) Subarachnoid hemorrhage presents with a(n) thunderclap
ruptured saccular aneurysm at the bifurcations of the arteries of the headache, loss of consciousness, vomiting, meningeal irritation,
circle of Willis. nuchal rigidity, photophobia, retinal hemorrhages, and death.

Other causes: trauma, AV malformation "The worst headache of my life"

file:///C:/Users/vikas/OneDrive/Desktop/print.html 111/207
5/10/2021 print.html

Syringomyelia is the central cavitation of the cervical cord due to


abnormal collection of fluid in the parenchyma and is most
commonly associated with Arnold-Chiari malforation.

A syrinx can be asymptomatic and discovered incidentally on


spinal cord imaging. Other patients present with progressive
central cord deficits that can include a prominent central pain
syndrome in a segmental distribution. Surgical decompression with
fenestration and/or shunt placement is recommended for patients
with neurologic deterioration or intractable central pain.
Neurologic deficits usually stabilize after intervention and
sometimes improve.

Chiari I malformation (CM-I) is characterized by abnormally


shaped cerebellar tonsils that are displaced below the level of the
Sulfur containing medications (sumatriptan) have what affect foramen magnum. For patients with CM-I who are clearly
on mucous membranes? symptomatic with lower cranial nerve palsies, syringomyelia,
Sympathetic ophthalmitis also known as spared eye injury most myelopathy, cerebellar symptoms, severe neck pain or occipital
Blue-green discoloration commonly due to headache related to the Chiari malformation, UTD recommends
surgical decompression of the foramen magnum.
Sulfhemoglobinemia occurs after exposure to an oxidative sulfur- uncovering of hidden antigens due to breaking of immunologic
containing medication (eg, sumatriptan, sulfasalazine). Patients barriers in the eye. Auto antibodies are produced
with this condition typically have a blue-green discoloration of
blood and mucocutaneous surfaces.

Tarsal tunnel syndrome is caused by compression of which Tetanus is caused by Clostridium tetani, which produces a(n) Tetanus is treated with benzodiazepines (for muscle spasms),
nerve? exotoxin that blocks inhibitory NMJ transmitters. human tetanus immunoglobulin (to neutralize free toxin), Td
toxoid (to immunize), and IV metronidazole.
Tibial nerve Tetanus occurs when Clostridium tetani, which is normally found
in soil, enters the body through a break in the skin (eg, puncture Thoroughly clean and debride any wounds with tissue necrosis.
Tarsal tunnel syndrome is due to compression of the tibial nerve as wound, lower extremity abrasions/lacerations). This bacterium
it passes through the ankle and is usually caused by a fracture of then secretes a toxin that binds to the peripheral nerve
the bones around the ankle. Patients have burning, numbness, and terminals and, using retrograde axonal transport, arrives at a
aching of the distal plantar surface of the foot or toes that central nervous system synapse, where it blocks the release of
sometimes radiate up to the calf. inhibitory neurotransmitters glycine and GABA across the
synaptic cleft. Because of this mechanism, symptoms usually
occur within a few days to several weeks following inoculation
and can include sweating, dysphagia, labile blood pressure, and
tachycardia.

Tetanus occurs almost exclusively in those with poor access to


health care as they are more likely to be unvaccinated or

file:///C:/Users/vikas/OneDrive/Desktop/print.html 112/207
5/10/2021 print.html
inadequately immunized. For those who become ill, the infected
wound should be debrided, antibiotics should be started, and
human tetanus immunoglobulin (to bind residual circulating toxin)
and tetanus toxoid should be administered.

Contrast this with Guillain-Barré syndrome, an acute immune-


mediated, demyelinating polyneuropathy, usually postinfectious,
that classically presents with ascending weakness, minor sensory
symptoms (acral paresthesias), and areflexia. Also contrast with
botulism, which is caused by Clostridium botulinum toxin, leading
to impairment of presynaptic acetylcholine release. It usually
presents with symmetric descending flaccid paralysis, blurred
vision, and areflexia and can include diaphragmatic paralysis.

file:///C:/Users/vikas/OneDrive/Desktop/print.html 113/207
5/10/2021 print.html

file:///C:/Users/vikas/OneDrive/Desktop/print.html 114/207
5/10/2021 print.html

The classic symptoms of parkinson disease stem from


Thalamic pain syndrome is also known as Dejerine-Roussy
bradykinesia, which manifests itself as a(n) resting pill-rolling
syndrome.
tremor that decreases with voluntary movement; cog-wheel
rigidity; postural instability, which presents as shuffling steps; and
The ventral posterolateral and ventral posteromedial nuclei of the
Tetanus presents with lockjaw and progresses to severe, a(n) mask-like expressionless face.
thalamus transmit sensory information from the contralateral side
generalized muscle contractions and sympathetic hyperactivity. of the body and face, respectively. Consequently, patients with
Tremor worsens with emotional stress and goes away when
thalamic stroke in this region often present with sudden-onset
performing routine tasks. Cogwheel rigidity refers to a rachet-like
contralateral sensory loss involving all sensory modalities (ie, pure
jerking, which can be elicited by testing the tone in one limb while
sensory stroke). Symptoms can be accompanied by transient
the patient clenches the opposing fist. Patient will have difficulty
hemiparesis, athetosis, or ballistic movements due to disruption of
initiating the first step.
neighboring basal ganglia structures and corticospinal fibers in the
posterior limb of the internal capsule. Several weeks to months
Other features: Dysarthria, dysphagia,
following the stroke, sensory deficits can improve; however, some
micrographia, dementia, orthostatic hypotension,
patients develop thalamic pain syndrome (Dejerine-Roussy
constipation, increased sweating, oily skin, personality changes,
syndrome). This condition is characterized by severe paroxysmal
and depression
burning pain over the affected area and is classically exacerbated
by light touch (allodynia).

file:///C:/Users/vikas/OneDrive/Desktop/print.html 115/207
5/10/2021 print.html

The early, localized stage of Lyme disease is characterized by


erythema migrans at the site of the tick bite (occurs approximately
one month following tick bite) and multiple lesions if spread has The grid test is used diagnose what kind of vision loss?
occured.
Macular degeneration
Large, painless, well-demarcated target-shaped lesion, commonly
seen on the thigh, groin, or axilla. Macular degeneration is the leading cause of blindness in
industrialized countries. Macular degeneration is a painless,
Primary EM usually presents within 1 month of the tick bite, progressive loss of central vision that typically occurs in older
appearing on the arms and moist areas of the body such as the patients, especially those with vascular risk factors. Both eyes are
axillae, groin, or trunk. The round or oval macule is initially often affected, and funduscopic abnormalities (eg, drusen [yellow
uniformly red and can develop a zone of central clearing as it deposits]) are prominent. One of the earliest findings in macular
expands, giving it the classic bull's eye appearance. It is not degeneration is distortion of straight lines such that they
particularly painful or itchy and can become as large as 20 cm in appear wavy. The grid test described above is frequently used to
diameter. screen for patients with macular degeneration. The primary risk
factor for macular degeneration is increasing age, although
Blood cultures for B burgdorferi are not available in most clinical smoking can increase the risk as well. Patients may be
laboratories and are not recommended. Serology is not asymptomatic, but others complain of visual problems in either
recommended in early localized disease as it is too insensitive and one or both eyes. Driving and reading are often some of the first
The early, disseminated stage of Lyme disease spreads via
many patients would be seronegative. After the onset of EM, IgM activities that are affected since they require fine visual acuity,
lymphatics and blood to cause intermittent constitutional
antibodies to B burgdorferi usually appear within 1-2 weeks, and which is provided primarily by the macula. In addition to the grid
symptoms, potentially resuting in early neuroborreliosis or Lyme
IgG antibodies typically appear within 2-6 weeks. However, test described above, ophthalmologic examination can identify
carditis.
serology should be performed in patients with signs of early drusen deposits in the macula, which are common lesions seen
disseminated or late Lyme disease. with this disorder. Visualization of straight lines is a task that
Early neuroborreliosis presents with cranial nerve palsies (most
requires fine visual acuity, which is controlled primarily by the
commonly peripheral facial nerve palsy); nocturnal radicular pain,
centrally-located macula as opposed to the more peripheral retina.
paresthesia, and paresis; meningitis; and/or polyneuropathy
(mononeuritis multiplex, asymmetrical). Lyme carditis carries a
risk of cardiac arrhythmias and myopericarditis.

file:///C:/Users/vikas/OneDrive/Desktop/print.html 116/207
5/10/2021 print.html

The most common cause of encephalitis is HSV-1.

The late, persistent stage of Lyme disease presents with arthritis


(10% of all cases, 60% of untreated patients), chronic CNS
disease, and/or acrodermatitis chronica atrophicans.
The presence of personality changes and strange behavior would
Arthritis lasts over a year and may be intermittent or persistent and
suggest a brain tumor in which location?
typically affects the large joints (especially knee or elbow).
Chronic CNS disease presents as subacute, mild encephalitis,
Frontal lobe
transverse myelitis, or axonal polyneuropathy. Late
neuroborreliosis manifestations include aseptic (lymphocytic)
The CT/MRI findings of a butterfly appearance with central
meningitis, progressive encephalomyelitis, cognitive impairment,
necrosis is classic for GBM, and the heterogenous, serpiginous
gait abnormality, bladder dysfunction, psychiatric abnormalities
contrast enhancement is typical of high-grade astrocytoma.
(e.g., depression, anxiety, bipolar disorder etc.), and/or peripheral
polyneuropathy. Acrodermatitis chronica atrophicans presents with
reddish-purple plaques and nodules on the extensor surfaces of the
legs, and hasn't been described convincingly in the United States

The secondary causes of a headache can be ruled out by checking The triad of parkinsonism, autonomic dysfunction (eg, Thrombolytic therapy should be administered in ischemic stroke
for red flags and remembered with the mneumonic: "VOMIT" hypotension), and widespread neurologic signs is suggestive of patients only within the first 3 - 4.5 hours and if the patient has no
what disease? risk of intracranial hemorrhage, including BP >185/110 mm Hg,
Vascular glucose <50 mg/dL, and INR >1.7
Other causes: mainly psuedotumor cerebri Shy-Drager syndrome
Medications/Drugs Technically, you're concerned about all hemorrhage, not just
Infection Always consider Shy-Drager syndrome (multi system atrophy) intracranial, so watch out for recent surgery and head
Tumor when a patient with Parkinsonism experiences orthostatic trauma. Aspirin and statins are commonly used for primary and
hypotension, impotence, incontinence, or other autonomic secondary ischemic stroke prevention. Aspirin may help reduce
symptoms. The accompanying bulbar dysfunction and laryngeal clot propagation and risk of stroke recurrence if administered
stridor may be fatal. Anti-Parkinsonism drugs are generally within 48 hours of stroke onset; however, intravenous alteplase is
ineffective, and treatment is aimed at intravascular volume more effective than antiplatelet therapy at achieving clot
expansion with fludrocortisone, salt supplementation, alpha- recanalization and neurologic recovery. Aspirin should be held for
adrenergic agonists, and application of constrictive garments to the 24 hours following thrombolytic therapy due to the increased risk
lower body. of intracranial hemorrhage. Intravenous heparin is not
recommended in the early acute phase of ischemic stroke due to its
limited efficacy and greater risk of bleeding complications.

file:///C:/Users/vikas/OneDrive/Desktop/print.html 117/207
5/10/2021 print.html

Thrombolytic therapy should be administered in ischemic stroke Tourette syndrome is associated with OCD and presents before Tourette syndrome is treated with patient
patients only within the first 3 - 4.5 hours and if the patient has no age 18 with multiple motor tics and at least one phonic tic. education and guanfacine and high-potency antipsychotics.
risk of intracranial hemorrhage, including BP >185/110 mm Hg,
glucose <50 mg/dL, and INR >1.7 Not all tics are Tourette's. Not all Tourette's involuntarily swear. Treat if symptoms are affecting quality of life.

Technically, you're concerned about all hemorrhage, not just Antipsychotics: haloperidol, fluphenazine, pimozide; other
intracranial, so watch out for recent surgery and head treatment options include α2 agonists, tetrabenazine, and atypical
trauma. Aspirin and statins are commonly used for primary and antipsychotics
secondary ischemic stroke prevention. Aspirin may help reduce
clot propagation and risk of stroke recurrence if administered α2-adrenergic agonists: guanfacine, clonidine; other treatment
within 48 hours of stroke onset; however, intravenous alteplase is
options include antipsychotics and tetrabenazine
more effective than antiplatelet therapy at achieving clot
recanalization and neurologic recovery. Aspirin should be held for
file:///C:/Users/vikas/OneDrive/Desktop/print.html 118/207
5/10/2021 print.html
24 hours following thrombolytic therapy due to the increased risk
of intracranial hemorrhage. Intravenous heparin is not
recommended in the early acute phase of ischemic stroke due to its
limited efficacy and greater risk of bleeding complications.

Transverse myelitis presents with lower extemity weakness or Treatment of insomnia involves addressing the underlying cause, Trigeminal neuralgia is one of the most painful conditions known
plegia, back pain, sensory deficits below the level of the lesion, psych eval, and sparingly sedative-hypnotics with caution for to mankind. Treat it with carbamazepine or oxcarbazepine.
and sphincter disturbance. symptomatic relief.
Trigeminal neuralgia (TN) presents with unilateral, intermittent
This is a rare condition that specifically affects the tracts across the Use the smallest dose possible, and avoid use for longer than 2 to 3 sharp pain of the right cheek and lips that lasts several seconds and
horizontal aspect of the spinal cord at a given level. The thoracic weeks. is triggered by minor stimuli (eg, brushing teeth, drinking cold
spine is the most commonly involved. High-dose steroid therapy is water). This disorder usually causes severe neuropathic pain
often used, but evidence supporting its use is equivocal. (burning, electric shock-like) along the V2 (maxillary) and V3
file:///C:/Users/vikas/OneDrive/Desktop/print.html 119/207
5/10/2021 print.html
(mandibular) branches of the trigeminal nerve (CN V). Very rarely,
Neuro-inflammatory spinal cord disorder that presents with the it can involve the V1 (ophthalmic) branch, where it can be
rapid onset of weakness, sensory alterations, and bowel and associated with ipsilateral autonomic findings (eg, tearing,
bladder dysfunction. rhinorrhea). An episode can last a few weeks to months and occur
again later. TN affects women more commonly than men and is
clinically diagnosed based on the characteristic pain. The
pathophysiology is thought to be related to demyelination along
the trigeminal nerve root (as it enters the pons), likely from
localized compression (eg, vascular structure). TN is usually
treated with medications first (eg, carbamazepine, oxcarbazepine).
If treatment is unsuccessful, surgery (eg, nerve decompression,
radiosurgery) can be considered.

file:///C:/Users/vikas/OneDrive/Desktop/print.html 120/207
5/10/2021 print.html

True or false: Meningiomas may present with calcifications on


neuroimaging.

True

A tumor originating from the arachnoid cap cells is called a


meningioma. These extra-axial, well-circumscribed, dural-based
masses can be partially calcified on neuroimaging. Patients may
present with headache, seizure, and focal neurologic deficits due to
mass effect. It is more common in women, and it is
estrogen/progesterone sensitive. Histology demonstrates
psammoma body formation.
True or false: Cluster headaches have an association with family
True or false: Aspirin is indicated for acute cardioembolic stroke history.
due to bacterial endocarditis.
False
False - only use antibiotics

Patients with bacterial endocarditis are at high risk for


complications, including transient ischemic attack or stroke due to
septic embolism. This commonly occurs in the distribution of the
middle cerebral artery. Treatment of ischemic stroke should be
aimed at the underlying cause. In patients with infective
endocarditis, intravenous antibiotics significantly reduce the risk of
septic cardioembolism within weeks of initiating therapy;
therefore, the most appropriate next step in management of this
patient is to continue current care with observation. Aortic valve
surgery can be considered if there is significant valvular
dysfunction resulting in heart failure, if infection is persistent or
difficult to treat medically, or if septic embolization is recurrent.

Early use of aspirin (eg, within 48 hours) in acute ischemic


stroke due to atherosclerotic thrombosis or embolism is
associated with a lower risk of recurrent stroke and decreased
mortality. Conversely, early use of intravenous heparin is
generally not recommended as it is associated with an
increased risk of symptomatic intracranial hemorrhage.

True or false: Neurocysticercosis brain lesions have contrast Tuberous sclerosis presents with cognitive impairment, epilepsy, Upper motor neuron lesions cause more weakness in the
enhancement. and skin lesions. supinator muscles of the upper limb.

Sometimes ( ͡~ ͜ʖ ͡°) Retianl hamartomas, renal angiomyolipomas, and rhabdomyomas Compared to the pronator muscles of the upper limb. Hence why
of the heart may also be present. Pronator drift is a useful diagnostic test.
NCC tends to cause seizures, and MRI of the brain usually reveals
a mixture of enhancing, nonenhancing, and calcified lesions Usually autosomal dominant.
file:///C:/Users/vikas/OneDrive/Desktop/print.html 121/207
5/10/2021 print.html
(depending on the stage of the cyst). Contrast this with
toxoplasmosis, which always has enhancement.

Taenia solium is a tapeworm transmitted to humans by the


ingestion of undercooked, contaminated pork. Tapeworms
typically remain localized in the gastrointestinal tract, but shed
eggs may pass to other individuals via the fecal-oral route.
Ingested eggshatch in the small intestine, invade the bowel wall,
and spread hematogenously (brain, muscle, liver), causing
cysticercosis. Most patients are initially asymptomatic, but cysts
may degenerate over months or years causing inflammation,
edema, and clinical illness.

Neurocysticercosis typically manifests as an adult-onset seizure.


Less commonly, patients develop signs of increased intracranial
pressure (eg, vomiting, headache, papilledema) due to obstruction
of cerebral spinal fluid outflow. Systemic symptoms are usually
absent, and laboratory studies are typically normal. Brain imaging
(CT, MRI) usually reveals >1 cystic lesion with surrounding
contrast enhancement and edema (without displacement of
adjacent tissue). Cysts at various stages of development
(hypodense lesions without enhancement, calcified nodules) are
also sometimes found. Diagnosis is typically made based on
clinical and radiologic findings; brain biopsy is rarely needed.
Patients are treated with antiepileptics (eg, phenytoin),
antiparasitics (eg,albendazole), and corticosteroids (for brain
inflammation).

file:///C:/Users/vikas/OneDrive/Desktop/print.html 122/207
5/10/2021 print.html

Uveitis (aka iritis or iridocyclitis) presents with constrictied pupils.

Uveitis presents as blurred vision with moderate pain, conjunctival


injection, and constricted pupils. Hypopyon is seen in severe Vasospasm occurs 3 to 10 days following a subarachnoid
anterior uveitis. Keratic precipitates ("mutton fat") and iris nodules hemorrhage and is detected with CT angiography.
may be seen. It is associated with HLA B27-related conditions.
Although rebleeding risk can be high in untreated SAH, surgical
Anterior uveitis presents with a painful, red eye associated with repair of the aneurysm (eg, endovascular coiling in this patient)
photophobia, tearing, and diminished visual acuity. The uvea is the practically eliminates the rebleeding risk. Rebleeding is the major
tissue layer between the cornea/sclera and the retina; the anterior cause of death within the first 24 hours of presentation, especially
uvea consists of the iris and ciliary body and the posterior uvea within the first 6 hours of untreated SAH. Vasospasm can occur in
consists of the choroid. Anterior uveitis is inflammation of the up to 30% of SAH patients from days 3-10 after presentation and
anterior uveal tract and is sometimes termed "iritis" (or is the major cause of delayed morbidity and death. It is likely
"iridocyclitis" if the ciliary body is involved). It is much more caused by arterial narrowing at the base of the brain due to
common than posterior uveitis, which involves the choroid. degradation of the blood and its metabolites and can lead to
Examination findings in anterior uveitis can include hyperemia cerebral infarction. CT angiography is preferred for detecting
concentrated at the junction of the sclera and cornea (ciliary flush), vasospasm, which can best be preventedwith initiation of
pupillary constriction, a hazy "flare" in the aqueous humor, and nimodipine. Vasovagal syncope can be caused by specific triggers such
layering of white cells in the anterior chamber (hypopyon). as micturition, defecation, cough.

Anterior uveitis is most often idiopathic or traumatic, but it can be Transient loss of consciousness along with loss of postural or
associated with systemic inflammatory diseases such as certain motor tone during urination is consistent with situational
infections (eg, herpesviruses, toxoplasmosis), sarcoidosis, (postmicturition) syncope, a form of reflex (neurally mediated)
spondyloarthritis (eg, ankylosing spondylitis, reactive arthritis), syncope associated with specific triggers (eg,micturition,
and inflammatory bowel disease. defecation, cough). The diagnosis is typically made by clinical
history. The specific triggers for reflex syncope cause an alteration
in the autonomic response and can lead to a cardioinhibitory,
vasodepressor, or mixed response: increased parasympathetic
stimulation can manifest as profound bradycardia, varying degrees
of atrioventricular block, or asystole; and decreased sympathetic
output can lead to vasodilation, hypotension, or syncope.

Vasovagal syncope is diagnosed with a(n) tilt-table study and Vasovagal syncope is due to an overactive vagus nerve leading to Vasovagal syncope is due to an overactive vagus nerve leading to
treated with reassurance and advising patients on physical counter hypotension or bradycardia. Vagus nerve stimulation is due to hypotension or bradycardia. Vagus nerve stimulation is due to
pressure maneuvers during the prodromal phase. visceral stimulation, overactive carotid sinus, or psychotropic visceral stimulation, overactive carotid sinus, or psychotropic
causes. causes.
Neurocardiogenic (vasovagal) syncope can be triggered by
emotional or painful stimuli and is frequently associated with
prodromal symptoms (eg, dizziness, nausea, pallor, diaphoresis,
abdominal pain, generalized sense of warmth) prior to the syncopal

file:///C:/Users/vikas/OneDrive/Desktop/print.html 123/207
5/10/2021 print.html
episode. The prognosis is excellent.

For patients with recurrent syncope, management consists of


advising patients to avoid triggers and to assume a supine position
with leg raising at the onset of symptoms. Physical
counterpressure maneuvers (eg, leg crossing with tensing of
muscles, handgrip and tensing of arm muscles with clenched
fists) during the prodromal phase can improve venous return and
cardiac output, sometimes aborting syncopal episodes. Some
patients in high-risk settings associated with a potential risk of
physical injury (eg, airline pilots, commercial drivers) require
temporary activity restriction until these maneuvers are proven to
be effective.

Lying down and raising legs may also be beneficial if it can be


undertaken safely in the circumstance. These should be initiated
upon first recognition of premonitory symptoms. No therapy has
been proven consistently effective for preventing vasovagal
syncope recurrences. Patients need reassurance, trigger avoidance,
and physical counterpressure maneuvers at the onset of symptoms.
Although beta blockers have been the most commonly used drug
therapy for vasovagal syncope, available evidence does not
support their efficacy. Older individuals (>42 years) may benefit
from beta blockers, and this observation has been included as a IIb
recommendation in the 2017 ACC/AHA/HRS guidelines.
However, the evidence is weak.

Tilt-table study reproduces the symptoms.

file:///C:/Users/vikas/OneDrive/Desktop/print.html 124/207
5/10/2021 print.html

file:///C:/Users/vikas/OneDrive/Desktop/print.html 125/207
5/10/2021 print.html

Vertebral osteophytes present similarly to herniated discs, but are Von Hippel-Lindau disease presents with cavernous Wallenberg syndrome is a lateral medulla stroke that results from
not associated with heavy lifting. hemangiomas of the brain, renal angiomas, and cysts in multiple occlusion to the posterior inferior cerebellar artery (PICA).
organs; and is associated with renal cell carcinoma and
pheochromocytomas. Vestibulocerebellar findings are almost always present and include
nystagmus (both horizontal and rotational), vertigo with falling to
Autosomal dominant the side of the lesion, and difficulty sitting upright without support.
Sensory findings include loss of pain and temperature in the
Cavernous malformations (cavernous angiomas, cavernomas) may ipsilateral face (spinal trigeminal nucleus and tract) and
occur as a sporadic or familial condition. These are thin walled contralateral trunk/limbs (spinothalamic tract). Other findings
dilated capillaries with a simple endothelial lining. They can occur include ipsilateral bulbar muscle weakness (eg, dysphagia,
throughout the brain but are most common in the subcortical dysarthria) due to involvement of the nucleus ambiguus and
rolandic and temporal areas. Clinical presentation can include autonomic dysfunction (eg, ipsilateralHorner's syndrome). Patients
hemorrhage, seizures, and/or progressive neurologic deficits. typically have sparing of voluntary motor function in the face and
Annual bleeding rates are up to 1 percent per year for body. The diagnosis is made via magnetic resonance imaging.
supratentorial lesions and up to 3 percent per year for brainstem Acute treatment usually involves intravenous thrombolytics (eg,
lesions. Recurrent hemorrhage is more common after an initial tissue plasminogen activator).
bleed and may be as high as 5 percent per year for supratentorial
lesions and 21 percent for brainstem lesions. The pattern of cranial nerve involvement helps distinguish between
lateral mid-pontine lesions and those in the medulla. Lateral mid-
pontine lesions affect the motor and principal sensory nuclei of the
ipsilateral trigeminal nerve, causing weakness of the muscles of
mastication, diminished jaw jerk reflex, and impaired tactile and
position sensation over the face. In contrast, dysphagia,
hoarseness, and diminished gag reflex are more typical of a lateral
medullary infarct due to cranial nerve IX and X involvement.

Medial medullary syndrome (alternating hypoglossal hemiplegia)

file:///C:/Users/vikas/OneDrive/Desktop/print.html 126/207
5/10/2021 print.html
is typically due to branch occlusion of the vertebral or anterior
spinal artery. Patients develop contralateral paralysis of the arm
and leg and tongue deviation toward the lesion. Contralateral loss
of tactile and position sense can also occur with infarcts that
extend dorsally.

Medial mid-pontine infarction presents with contralateral ataxia


and hemiparesis of the face, trunk, and limbs (ie, ataxic
hemiparesis). There is also variable loss of contralateral tactile and
position sense.

file:///C:/Users/vikas/OneDrive/Desktop/print.html 127/207
5/10/2021 print.html

file:///C:/Users/vikas/OneDrive/Desktop/print.html 128/207
5/10/2021 print.html

Wallenberg syndrome presents with ataxia where patients tend to Washed out color vision suggests what pathology? Washed out colour vision
fall toward to the ipsilateral side of lesion; loss of pain and
temperature in the ipsilateral face and contralateral body; Optic neuritis Optic neuritis
dysphagia and hoarseness due to involvement of the nucleus
ambiguus; and intractable hiccups due to autonomic dysfunction. Optic neuritis is thought to be immune-mediated and is most
commonly seen in women age 20-40. Symptoms develop acutely
Vestibulocerebellar findings are almost always present and include and usually include monocular vision loss, pain with eye
nystagmus (both horizontal and rotational), vertigo with falling to movement, and "washed-out" color vision. Examination typically
the side of the lesion, and difficulty sitting upright without support reveals an afferent pupillary defect (paradoxical pupillary dilation
(due to injury of the inferior cerebellar peduncle from PICA of the affected eye with the swinging-flashlight test) and central
stroke). Sensory findings include loss of pain and temperature in scotoma. Funduscopy is usually normal as inflammation occurs
the ipsilateral face (spinal trigeminal nucleus and tract) and behind the optic nerve head.
contralateral trunk/limbs (spinothalamic tract). Other findings
include ipsilateral bulbar muscle weakness (eg, dysphagia, Optic neuritis is strongly associated with multiple sclerosis and is
dysarthria) due to involvement of the nucleus ambiguus and frequently the heralding symptom of this condition. As such,
autonomic dysfunction (eg, ipsilateral Horner's syndrome). patients are typically evaluated with MRI of the orbits and the
Patients typically have sparing of voluntary motor function in the brain to look for other areas of inflammation.
face and body. The diagnosis is made via magnetic resonance

file:///C:/Users/vikas/OneDrive/Desktop/print.html 129/207
5/10/2021 print.html
imaging. Acute treatment usually involves intravenous
thrombolytics (eg, tissue plasminogen activator).

The pattern of cranial nerve involvement helps distinguish between


lateral mid-pontine lesions and those in the medulla. Lateral mid-
pontine lesions affect the motor and principal sensory nuclei of the
ipsilateral trigeminal nerve, causing weakness of the muscles of
mastication, diminished jaw jerk reflex, and impaired tactile and
position sensation over the face. In contrast, dysphagia,
hoarseness, and diminished gag reflex are more typical of a lateral
medullary infarct due to cranial nerve IX and X involvement.

Medial medullary syndrome (alternating hypoglossal hemiplegia)


is typically due to branch occlusion of the vertebral or anterior
spinal artery. Patients develop contralateral paralysis of the arm
and leg and tongue deviation toward the lesion. Contralateral loss
of tactile and position sense can also occur with infarcts that
extend dorsally.

Medial mid-pontine infarction presents with contralateral ataxia


and hemiparesis of the face, trunk, and limbs (ie, ataxic
hemiparesis). There is also variable loss of contralateral tactile and
position sense.

file:///C:/Users/vikas/OneDrive/Desktop/print.html 130/207
5/10/2021 print.html

file:///C:/Users/vikas/OneDrive/Desktop/print.html 131/207
5/10/2021 print.html

Wernicke's area is located in the superior temporal gyrus and What Parkinson drug is used in younger patients where tremor is What are Td/Tdap vaccine recommendations for adults?
when lesioned can result in Wernicke's aphasia and the predominant symptom?
a(n) contralateral superior homonymous quadrantanopsia Tdap once as an adult instead of Td booster; Td every 10 years
Trihexyphenidyl thereafter
Nearly 95% of right-handed and up to 70% of left-handed people
are left-hemisphere dominant for verbal and written language Parkinsonian tremors are more pronounced with distractibility (eg, Adult patients should receive the tetanus-diphtheria-pertussis
function. Patients with lesions affecting the posterior inferior performing mental tasks) and re-emergence (tremor goes away (Tdap) vaccine as a one-time dose in place of Td. If the patient has
frontal gyrus (Broca's area) can comprehend and follow commands with movement and then re-emerges when the movement is not received Tdap as an adult, or if the patient's prior vaccine
but are unable to verbalize or write properly (expressive aphasia). stopped). The tremor of Parkinson's disease characteristically history is unknown, the Centers for Disease Control and
Broca's lesions can also affect the neurologic pathway from that occurs during rest and improves with activity (essential tremor has Prevention (CDC) recommends that Tdap be given followed by Td
brain region to the motor function of the larynx and mouth. the opposite pattern). The Parkinson's tremor usually has a every 10 years thereafter. This recommendation is especially true
frequency of 4-5 Hz, and is often the first manifestation of for pregnant women and other adults who are expected to be in
In rare instances, the lesions can extend and affect the arcuate Parkinson's disease. The tremor often involves one hand before the close contact with small children. The tetanus-diphtheria toxoid
fasciculus to impair the patient's ability to repeat phrases other or presents asymmetrically. Rigidity is also characteristic of booster (Td) vaccine should be given every 10 years to all adults
(conduction aphasia). An easy way to remember is that Broca's Parkinson's disease. The absence of additional symptoms such as after age 18.
aphasia often represents a broken speech system. Patients with postural instability or gait abnormality should not exclude the
injury to the lateral frontal lobe can also develop contralateral diagnosis of Parkinson's. There are a variety of medications that
weakness of the face and extremities (motor cortex) and conjugate can be used in different combinations for the treatment of

file:///C:/Users/vikas/OneDrive/Desktop/print.html 132/207
5/10/2021 print.html
gaze deviation to the side of the lesion (contraversive frontal eye Parkinson's disease. Trihexyphenidyl is typically used in younger
fields). In contrast, patients with Wernicke's aphasia have difficulty patients where tremor is the predominant symptom.
comprehending and following commands but are able to speak
fluently. However, their speech tends to be rambling without Clonazepam is a second-line treatment for essential tremor, but
concrete meaning. other medications are preferred due to the abuse potential of
benzodiazepines. Primidone can be used as a treatment for
Dominant temporal lobe lesions can affect comprehension essential tremor, sometimes in combination with a beta-
(receptive aphasia), ability to speak nouns (anomic aphasia), blocker. The beta-blocker propranolol is considered first-line for
repetition (conductive aphasia) due to arcuate fasciculus the treatment of essential tremor.
involvement, and contralateral superior homonymous
quadrantanopsia due to inferior optic radiations (Meyer's loop)
involvement. However, patients usually have intact expressive
speech, motor, and primary sensory functions.

Dominant parietal lobe lesions can present with contralateral


sensory loss (eg, pain, vibration, agraphesthesia, astereognosis)
and contralateralinferior homonymous quadrantanopsia due to
superior optic radiations involvement. Nondominant parietal lobe
lesions typically cause anosognosia (denial of one's disabilities)
and contralateral apraxia (inability to carry out learned purposeful
movements).

file:///C:/Users/vikas/OneDrive/Desktop/print.html 133/207
5/10/2021 print.html

file:///C:/Users/vikas/OneDrive/Desktop/print.html 134/207
5/10/2021 print.html

What are normal protein levels in the CNS? What are the first tests to perform in suspected corneal abrasion? What are the two most common bacteria that cause brain abscess?

<40 1. Penlight test 1. Viridans streptococci (eg, Streptoccocus mutas, Streptococcus


2. Visual acuity sangus)
2. Staphylococcus aureus
Perform these before fluorescein examination.
A single brain abscess is usually the result of direct extension from
an adjacent tissue infection (eg, otitis media, sinusitis, dental
infection). The 2 most commonly isolated organisms are viridans
streptococci and Staphylococcus aureus. Symptoms are often
nonspecific, which may delay diagnosis. Patients frequently have
headaches that are severe, unilateral, and resistant to analgesics.
Fevers are an unreliable predictor, occurring in ~50% of patients.
If diagnosis is delayed, focal neurologic deficits orseizures may
emerge.

Brain imaging (CT scan, MRI) typically reveals cerebritis initially,


but within 1-2 weeks the infection consolidates to a ring-enhancing
lesion with central necrosis. Diagnosis requires CT-guided
aspiration or surgical biopsy to obtain tissue for Gram stain and
culture (bacterial, fungal, mycobacterial). Most patients are treated
with empiric, intravenous antibiotics (eg, metronidazole,
ceftriaxone, and vancomycin) and aspiration of the lesion.

Nocardia may cause a brain abscess with ring-enhancing lesions


on CT scan. Most infections occur in patients with significant
immunocompromise (eg, HIV with CD4 count <100/mm3).

file:///C:/Users/vikas/OneDrive/Desktop/print.html 135/207
5/10/2021 print.html

file:///C:/Users/vikas/OneDrive/Desktop/print.html 136/207
5/10/2021 print.html

What caused this fundoscopic image?

Central retinal vein occlusion

What are the two most common malignancies to spread to the


brain?

1. Metastatic lung cancer


2. Metastatic breast cancer

Pathophysiology: neoplastic cells travel through the vasculature


and lodge in the small-caliber vessels at the gray and white matter
junction. Multiple lesions typically form and may cause mass
effect from tumor growth and edema.

Diagnosis: although pulmonary symptoms arise in the majority of


patients with lung cancer, up to 30% of cases present with
manifestations of metastatic brain disease, including headaches, What are the two most important risk factors for developing a
focal deficits, cognitive changes, and seizures. MRI of the brain stroke?
with contrast usually reveals multiple well-circumscribed lesions
with vasogenic edema at the gray and white matter junction. Age and hypertension

Treatment: glucocorticoids are often prescribed to reduce swelling Hypertension has the strongest association with both ischemic and
hemorrhagic stroke due to elevated shearing force on the Funduscopic examination of the left eye reveals swelling of the
and palliate symptoms, but the primary tumor site must be optic disk, retinal hemorrhages, dilated and tortuous veins, and
identified (usually with chest x-ray and/or CT scans) and biopsied intracerebral vascular endothelium, which accelerates the
atherosclerotic process and promotes thrombi formation. Patients cotton wool spots.
for definitive diagnosis.
with hypertension have approximately 4 times the risk of CVA
compared to normotensive individuals. Even mild reductions of
blood pressure with antihypertensive medication can result in a
diminished CVA risk. Other significant, but less potent, risk factors
for CVA: hypercholesterolemia, diabetes mellitus, smoking, and
sedentary lifestyle.

Others include atrial fibrillation, coronary artery disease (CAD),


family history of stroke, previous stroke/TIA, and carotid bruits. In
younger patients risk factors include oral contraceptive use,
hypercoagulable states, vasoconstrictive drug use, polycythemia
vera, and sickle cell disease.

What classes of medications can cause acute angle-closure What cutaneous manifestations are associated with amantadine? What diagnostic tests should be ordered for idiopathic
glaucoma? intracranial hypertension?

file:///C:/Users/vikas/OneDrive/Desktop/print.html 137/207
5/10/2021 print.html
Livedo reticularis
1. Sympathomimetics 1. MRI with or without MRV (to exclude other causes)
2. Anticholinergics When parkinson symptoms compromise daily life, symptomatic 2. Lumbar puncture (safe despite presence of communicating
3. Decongestants (antihistamines have anticholinergic activity) therapy should be started. Agents available for this purpose are hydrocephalus)
levodopa/carbidopa, dopamine agonists, anticholinergics, and
Mydriasis increases angle between iris and lens, inhibiting amantadine. Patients with IIH typically present with holocranial headache,
aqueous drainage. vision changes (blurry vision and diplopia), and pulsatile tinnitus
Levodopa/carbidopa is the most effective symptomatic therapy and ("whooshing" sound in the ears). Although IIH is frequently seen
Pathophysiology: β2 increases aqueous humor production, has beneficial effects on all the features of parkinsonism, although in young obese women, it has also been associated with certain
α1 increases pupillary dilator muscle contraction it does not stop progression of the disorder. The most common medications (eg, isotretinoin, tetracyclines, growth hormone,
early side effects include hallucinations, confusion, agitation, excessive vitamin A).
(mydriasis). M3 increases pupillary sphincter muscle contraction
dizziness, somnolence, and nausea. Involuntary movements (eg,
(miosis), ciliary muscle contraction (accommodation). dyskinesia, dystonia) typically occur after 5–10 years of therapy in Initial evaluation of IIH includes complete ocular examination and
nearly 50% of patients taking levodopa/carbidopa. As a result, it is neuroimaging to exclude secondary causes of intracranial
Sudden angle closure typically occurs as a response to pupillary important to find the lowest therapeutic dose and closely monitor hypertension (eg, mass, hemorrhage, cerebral vein thrombosis).
dilation from medications (eg, anticholinergics such as tolterodine, patients. Choreiform dyskinesia is a later side effect of Magnetic resonance imaging possibly with magnetic resonance
sympathomimetics, and decongestants) or other stimuli (eg, dim levodopa/carbidopa but is more commonly seen earlier with venography (to rule out cerebral vein thrombosis) is the preferred
light). catechol-O-methyltransferase inhibitors (eg, entacapone and imaging modality. Empty sella is present in about 70% of patients
tolcapone). but is not diagnostic. Lumbar puncture (LP) is then indicated to
document elevated opening pressure.
Livedo reticularis, a mottled vascular pattern on the lower
extremities, is a side effect more commonly seen with amantadine Papilledema is not a contraindication to LP unless the patient has
and is not usually seen with levodopa/carbidopa. evidence of obstructive or noncommunicating hydrocephalus
and/or a space-occupying lesion with/without mass effect or
Urinary retention is more commonly seen with anticholinergics midline shift. Pseudotumor cerebri causes a communicating
(eg, benztropine), which improve tremor and rigidity but do not hydrocephalus (ie, pressures in the ventricular and subarachnoid
have much effect on bradykinesia. spaces are equilibrated with the lumbar cistern) and therefore LP is
considered safe. LP is performed with the patient in the lateral
decubitus position with legs extended. An opening pressure of
>250 mm H2O is considered abnormal and in the appropriate
clinical setting is diagnostic of IIH. Cerebrospinal fluid studies are
normal in IIH patients.

The treatment includes weight reduction and acetazolamide (if


weight reduction fails). When medical measures fail or visual field
defects are progressive, shunting or optic nerve sheath fenestration
is done to prevent blindness, which is the most significant
complication of this otherwise benign disorder

file:///C:/Users/vikas/OneDrive/Desktop/print.html 138/207
5/10/2021 print.html

file:///C:/Users/vikas/OneDrive/Desktop/print.html 139/207
5/10/2021 print.html

What disease should be suspected if trigeminal neuralgia presents


bilaterally?

Multiple sclerosis (herpes zoster presents unilaterally)

Trigeminal neuralgia is a disorder characterized by recurrent and


sudden-onset severe, stabbing pain along the V2 (maxillary) and
V3 (mandibular) branches of the trigeminal nerve, which usually
occurs unilaterally. The episodes last for a few seconds to minutes
and can be triggered by minor stimuli (eg, touch, wind, chewing).
Multiple sclerosis should be suspected when trigeminal neuralgia
presents bilaterally. Multiple sclerosis (MS), an autoimmune
demyelinating central nervous system disorder, is one of the few
conditions that may present with trigeminal neuralgia bilaterally.
This occurs due to demyelination of the nucleus of the trigeminal
nerve or the nerve root, which leads to improper signaling of the
nerve and paroxysms of severe pain.
What differentiates persistent vegetative state from brain death?
What differentiates coma from persistent vegetative state? Inflammation and edema of the facial nerve (CN VII) (Bell's
Persistent vegetative state has active brainstem function, whereas palsy) often occur due to herpes simplex reactivation and usually
Coma has depressed cerebral function, whereas persistent brain death has absent brainstem function. do not cause pain but rather unilateral facial paralysis. Reactivation
vegetative state has absent cerebral function. of a virus (eg, herpes zoster) in the sensory ganglia can cause
As determined by brainstem reflexes: corneal, cold water calorics,
severe pain. However, this usually occurs along a dermatomal
doll's eyes
distribution and often is associated with vesicles or a macular rash.
When the trigeminal nerve is involved, it is often the V1 branch
(herpes zoster ophthalmicus), which can lead to blindness.

What disorder(s) or lesion(s) result in each of the following types What disorder(s) or lesion(s) result in each of the following types What does the swinging flashlight test demonstrate in optic
of gait disturbances? of gait disturbances? neuritis?

Ataxic gait: cerebellar lesion (weak booty) Ataxic gait: cerebellar lesion (weak booty) Afferent pupillary defect
En-bloc gait: vertigo and nystagmus (staggering gait w/ minimal En-bloc gait: vertigo and nystagmus (staggering gait w/ minimal
head movement head movement
file:///C:/Users/vikas/OneDrive/Desktop/print.html 140/207
5/10/2021 print.html
Gait disequilibrium: frontal lobe or multiple sensory system Gait disequilibrium: frontal lobe or multiple sensory system Optic neuritis is an inflammatory demyelination of the optic nerve.
lesions lesions Optic neuritis is thought to be immune-mediated and is most
Hypokinetic gait: parkinsonism (typically narrow-based) Hypokinetic gait: parkinsonism (typically narrow-based) commonly seen in women age 20-40. Symptoms develop acutely
Spastic gait: UMN lesion (spinal cord injury, cerebral palsy) Spastic gait: UMN lesion (spinal cord injury, cerebral palsy) and usually include monocular vision loss, pain with eye
Waddling gait: muscular dystrophy Waddling gait: muscular dystrophy movement, and "washed-out" color vision. Examination typically
Wide-based, high-stepping gait: sensory ataxia (lesions in Wide-based, high-stepping gait: sensory ataxia (lesions in reveals an afferent pupillary defect (paradoxical pupillary dilation
peripheral nerves, dorsal roots, posterior columns) peripheral nerves, dorsal roots, posterior columns) of the affected eye with the swinging-flashlight test) and central
scotoma. Funduscopy is usually normal as inflammation occurs
In spastic gait, movements of affected extremities are slow, stiff In spastic gait, movements of affected extremities are slow, stiff behind the optic nerve head.
effortful. Patients with loss of proprioception due to sensory effortful. Patients with loss of proprioception due to sensory
neuronopathy may have postural or gait instability and a wide- neuronopathy may have postural or gait instability and a wide- Optic neuritis is strongly associated with multiple sclerosis and is
based gait. They may stomp their feet against the floor (slap gait) based gait. They may stomp their feet against the floor (slap gait) frequently the heralding symptom of this condition. As such,
to help them know where their lower limbs are relative to the to help them know where their lower limbs are relative to the patients are typically evaluated with MRI of the orbits and the
ground. ground. brain to look for other areas of inflammation.

file:///C:/Users/vikas/OneDrive/Desktop/print.html 141/207
5/10/2021 print.html
Gait disequilibrium: Gait disequilibrium:

What electrolyte imbalance is a complication of subarachnoid What endocrine disorders are associated with carpal tunnel What eye disease is characterized by a sudden onset of pain and
hemorrhage? syndrome? redness in the medial canthal region?

Hyponatremia (SIADH) Diabetes and hypothyroidism Dacryocystitis

Compression of the median nerve causes carpal tunnel syndrome Not the same as dacryostenosis, which is a neonatal disease
(CTS), characterized by pain and paresthesias in the first 3 digits
and the radial half of the fourth. These symptoms are often worse Dacryocystitis is an infection of the lacrimal sac. It usually occurs
at night. Motor involvement in severe cases can cause weakness of in infants and adults over the age of 40. Acute dacryocystitis is
thumb abduction and opposition, and atrophy of the thenar characterized by the sudden onset of pain and redness in the
eminence. The diagnosis is usually made on clinical grounds. medial canthal region. Sometimes, a purulent discharge is noted
During examination, tapping over the median nerve at the wrist from the punctum. A few patients present with fever, prostration,
(Tinel sign) or holding the wrists in extreme flexion with the and an elevated leukocyte count. Staphylococcus aureus and beta-
dorsum of the hands pressed together (Phalen test) can often hemolytic Streptococcus are the usual infecting organisms. It
reproduce the symptoms. However, if the diagnosis is uncertain (or usually responds to systemic antibiotic therapy.
severe symptoms prompt consideration for surgery), CTS can be
confirmed with nerve conduction studies, which will show slowing
in the median nerve at the wrist. X-ray is not routinely advised in
the evaluation of CTS. It may be considered in those with a history
of significant trauma or evidence of advanced osteoarthritis (eg,
decreased range of motion, Heberden and Bouchard nodes).

file:///C:/Users/vikas/OneDrive/Desktop/print.html 142/207
5/10/2021 print.html

file:///C:/Users/vikas/OneDrive/Desktop/print.html 143/207
5/10/2021 print.html

What frequency of migraine headaches should you start to What is Ramsay Hunt syndrome? What is Todd paralysis?
consider prophylactic medication?
Herpes zoster infection of the ear (can cause facial never palsy) Transient unilateral weakness following a tonic-clonic seizure that
>4/month usually spontaneously resolves
The most typical manifestation of herpes zoster infection in the ear
Abortive medications (eg, sumatriptan) can be taken as needed at is Ramsay Hunt syndrome, which presents with facial nerve palsy
the onset of an attack, but prophylactic medication should be and vesicles in the auditory canal and auricle.
considered in patients who:
1. Have frequent (eg, >4/month) or long-lasting (eg, >12 hours)
episodes
2. Experience disabling symptoms that prevent regular activities
despite abortive treatment
3. Are unable to take or have had no relief with abortive
medications
4. Overuse abortive medication (eg, nonsteroidal anti-
inflammatory drugs [NSAIDs]) and have rebound headache

file:///C:/Users/vikas/OneDrive/Desktop/print.html 144/207
5/10/2021 print.html

What is chemoprophylaxis for all respiratory contacts of a patient


with meningococcal meningitis?
What is an orthostatic tremor?
Rifampin, ciprofloxacin, ceftriaxone (one or the other)
What is a chalazion? Tremor in the legs that occurs immediately on standing and is
relieved by sitting. Used for patients with >8 hours of contact. Not indicated for
Chronic, granulomatous inflammation of the meibomian gland the majority of health care workers unless exposure tor
Orthostatic tremor is a postural tremor considered to be a variant espiratory secretions.
Following resolution of infection of a hordeolum (stye), some of essential tremor. It is usually high frequency (14-18 Hz) without
patients have a residual granulomatous nodule (chalazion) that other clinical signs or symptoms. Gram stain is often able to identify diplococci. The empiric
regresses slowly over several months. Chalazion presents as lid treatment of choice is a third-generation cephalosporin (eg,
discomfort. It is a chronic, granulomatous inflammation of the ceftriaxone) and vancomycin. Dexamethasone is not beneficial in
meibomian gland. It appears as a hard, painless lid nodule. For meningococcal meningitis but is often administered prior to
patients with a persistent hordeolum (eg, >1-2 weeks) or a large organism isolation (once isolated, dexamethasone can be
chalazion, additional management options include incision and discontinued).
curettage. Topical antibiotics are often prescribed for hordeolum
but are usually unnecessary. Empiric treatment of unidentified adult meningitis: most patients
receive a third-generation cephalosporin (eg, ceftriaxone),
vancomycin, and dexamethasone (to reduce inflammatory
morbidity). Ampicillin is added for patients age >50 or those who
are immunocompromised due to increased risk of Listeria
monocytogenes.

file:///C:/Users/vikas/OneDrive/Desktop/print.html 145/207
5/10/2021 print.html

What is initial pharmacotherapy for Lewy-Body dementia? What is management for a patient bitten by the following What is management of acute warfarin-associated intracerebral
animals? hemorrhage?
Cholinesterase inhibitors
High-risk wild animal that is unavailable for testing: Postexposure Prothrombin complex concentrate plus IV vitamin K
If psychotic symptoms persist, a low-dose, second-generation prophylaxis
antipsychotic is preferred due to the severe neuroleptic sensitivity Pet animal that is unavailable for testing: Postexposure Patients with warfarin-associated intracerebral hemorrhage should
seen in these patients. prophylaxis have their anticoagulation reversed immediately to reduce the risk
Pet animal that is available for testing: Observation for 10 days for of death and permanent disability. Initial therapy should include
signs of rabies intravenous vitamin K, which has a sustained response but takes
High-risk wild animal that is available for testing: Animal is approximately 12-24 hours to be effective (promotes clotting
euthanized and its brain is tested for rabies factor synthesis in the liver). Prothrombin complex concentrate
Low-risk wild animal: Nothing (PCC) should also be provided as it contains vitamin K-dependent
clotting factors (eg, II, VII, IX, X). Fresh frozen plasma can be
Rabies usually presents with motor weakness, paresthesias, and considered if PCC is not available; however, it takes longer to
encephalitis that progress to coma and death. Rabies is transmitted prepare and administer and requires more volume infusion
following a bite from an infected mammal. In developing compared to PCC.
countries, dogs account for >90% of transmission due to
inadequate rabies control programs. In contrast, rabies in
domesticated animals is very rare in the United States due to
effective rabies vaccination. Most cases of rabies in the United
States are due to bites from wild animals (eg, bats, raccoons, foxes,
skunks).

Postexposure prophylaxis (PEP) consists of a series of rabies


immunizations as well as rabies immune globulin. However, not
all patients require PEP:
1. Patients exposed to high-risk wild animals (eg, bats, raccoons,
foxes, skunks) should receive PEP for rabies if the animal is
unavailable for testing. In particular, bat bites can go
unrecognized, so PEP is recommended following direct exposure
to bats (unless the patient is constantly aware of the bats and is
certain a bite was not inflicted). In the rare situation that the high-
risk wild animal is available for testing, PEP can be withheld until
the animal is euthanized, its brain has been tested for rabies, and
the test results are available.
2. Patients bitten by domestic animals (ie, pets) in the United
States do not require PEP if the pet is available for testing. The
incubation period for rabies is usually 1-3 months, but animals that
are contagious (ie, have rabies virus in their saliva) will be
symptomatic 5-10 days after becoming contagious. Therefore,
pets available for quarantine can be observed for 10 days for signs
of rabies. However, if the pet is unavailable for quarantine (or is
symptomatic), PEP should be administered.
3. Patients bitten by low-risk animals (eg, squirrels, chipmunks) do
not need PEP.

If the dog is available, it can be observed for 10 days for signs of


rabies. There is no need to euthanize it for testing. PEP is painful,
costly, and intense (4 visits). Rabies incubation lasts several
months and PEP is effective at any point during that time;
therefore, immediate PEP is not indicated when the animal is
available for observation or testing. Patients with bites from
high-risk wild animals or symptomatic domestic animals (eg,
dogs) should start PEP if the animal is unavailable. However, if the
animal is available, either observation (pets) or testing (wild

file:///C:/Users/vikas/OneDrive/Desktop/print.html 146/207
5/10/2021 print.html
animals) should be performed before deciding on PEP.

What is management of myasthenic crisis? What is management of postictal lactic acidosis? What is meralgia paresthetica?

1. Intubation Observation and a repeat chemistry panel after approximately 2 Burning pain and paresthesia of the lateral thigh caused by
2. Plasmapheresis or IVIG hours compression of the lateral femoral cutaneous nerve at the waist
3. Corticosteroids
It is a transient anion gap metabolic acidosis that resolves without Meralgia paresthetica is caused by compression of the lateral
Myasthenic crisis is characterized by severe respiratory muscle treatment within 90 minutes following resolution of seizure femoral cutaneous nerve at the waist. It causes burning pain and
weakness leading to respiratory failure. Individuals often have activity. Seizure activity, especially a tonic-clonic seizure, can paresthesias at the lateral thigh. Symptoms are unaffected by
increasing generalized and bulbar muscle weakness (eg, difficulty significantly raise serum lactic acid levels due to skeletal muscle motion.
coughing up sputum) prior to the onset of crisis. The condition hypoxia and impaired hepatic lactic acid uptake. If the metabolic
may be precipitated by infection (eg, pneumonia), surgery, or acidosis has not resolved after two hours, other potential causes of Trochanteric bursitis is caused by friction of the tendons of the
various medications (eg, azithromycin). anion gap metabolic acidosis (eg, intoxication, sepsis) should be gluteus medius and tensor fascia lata over the greater trochanter of
investigated. the femur. Pain is localized to the lateral hip and is worsened by
Patients with deteriorating respiratory status (eg, declining oxygen
saturation and vital capacity) should first be intubated for airway The risks and benefits of treating acute metabolic acidosis with
protection. Following intubation, acetylcholinesterase inhibitors sodium bicarbonate are not entirely clear; however, it is generally
(eg, pyridostigmine) used in the daily management of MG are recommended in patients with severe acute metabolic acidosis with
temporarily held to reduce excess airway secretions and the risk of pH <7.1. Administration of sodium bicarbonate may cause
aspiration. myocardial depression and increased lactic acid production;
therefore, in patients with pH ≥7.1, the relatively small benefits of
Once stabilized, patients in myasthenic crisis require treatment sodium bicarbonate do not typically outweigh the risks.
with intravenous immunoglobulins or plasmapheresis (preferred)
in addition to corticosteroids. Other immunomodulatory therapy
(eg, mycophenolate mofetil or azathioprine) can be considered,
especially if corticosteroid therapy is ineffective, but usually
requires several weeks to reach clinical efficacy.

file:///C:/Users/vikas/OneDrive/Desktop/print.html 147/207
5/10/2021 print.html
direct pressure.

What is pharmacological management of a transient ischemic What is pharmacological management of amyotrophic lateral What is seen on peripheral blood smear in ehrlichiosis?
attack caused by cardiovascular disease? sclerosis?
Intracytoplasmic morulae in monocytes

file:///C:/Users/vikas/OneDrive/Desktop/print.html 148/207
5/10/2021 print.html
Aspirin, statin, reduction of blood pressure Riluzole Ehrlichiosis is a tick-borne infection caused by Ehrlichia
chaffeensis and E ewingii. These bacteria are carried by ticks,
TIAs are treated by modifying identifiable risk factors to prevent Riluzole is a glutamate inhibitor that is currently approved for use including the lone star tick (Amblyomma americanum), found in
recurrent TIA or stroke, including discontinuing tobacco use, in patients with amyotrophic lateral sclerosis. Modestly increases the southeastern and south central United States. The principal
initiating aspirin, starting a statin for hyperlipidemia, and reducing survival by decreasing glutamate excitotoxicity via an unclear reservoir is the white tail deer. Ehrlichiosis is characterized by an
blood pressure. mechanism. Although it cannot arrest the underlying pathological acute febrile illness with malaise and altered mental status.
process, it may prolong survival and the time to tracheostomy. Its Ehrlichiosis is not often associated with a rash (<30% in adults)
Administration of tissue plasminogen activator is used for the side effects are dizziness, nausea, weight loss, elevated liver and is described as "Rocky Mountain spotted fever (RMSF)
treatment of acute ischemic stroke that causes a significant enzymes and skeletal weakness. without the spots." Neurologic symptoms may include confusion,
neurological deficit (eg, hemiparesis) and presents within 4.5 hours mental status changes, clonus, and neck stiffness. Laboratory
from onset of symptoms. Alteplase is not indicated in TIA since studies often show leukopenia and/or thrombocytopenia, along
it's not a stroke. with elevated aminotransferases and lactate dehydrogenase. The
diagnosis is often clinical, although definitive diagnosis can be
made through visualization of intracytoplasmic morulae in white
blood cells or through polymerase chain reaction testing.
Doxycycline is the appropriate treatment and is often initiated
empirically when the diagnosis is suspected while confirmatory
testing is pending.

Ixodes scapularis, the tick that carries Borrelia burgdorferi, is not


commonly found in the southern United States. The majority
(80%) of patients with early localized (primary) Lyme disease
(within days of the tick bite) have the classic erythema migrans
rash with or without constitutional symptoms. Lyme disease
typically presents with a rash and the presence of a high fever and
hematologic and liver enzyme abnormalities Early disseminated
(secondary) Lyme disease exhibits prominent neurologic (eg,
encephalitis, cranial nerve palsy, radiculopathy) and cardiac (eg,
heart block, pericarditis) manifestations several weeks to months
after the initial tick bite. Doxycylcine is sufficient to treat Lyme
disease. Ceftriaxone is used for the treatment of neurologic and
cardiac manifestations of Lyme disease.

In 90% of RMSF patients, a maculopapular rash (involving the


soles and palms) occurs a few days after the onset of fever.
Doxycycline is the first-line treatment. Chloramphenicol is a
second-line treatment option for ehrlichiosis and RMSF. It is
associated with blood dyscrasias and is not commonly used.

file:///C:/Users/vikas/OneDrive/Desktop/print.html 149/207
5/10/2021 print.html

What is tetanus prophylaxis for the following situations? What is the Uhthoff phenomenon? What is the bedside ice pack test used for?

Minor wound, ≥3 tetanus toxoid doses, last dose <10 years Worsening symptoms of multiple sclerosis during exposure to high Diagnosis of myasthenia gravis
ago: Nothing temperatures
Minor wound, ≥3 tetanus toxoid doses, last dose ≥10 years The diagnosis of MG can be supported with the bedside ice pack
ago: Tetanus toxoid-containing vaccine test. In this test, an ice pack is applied over the eyelids for several
Minor wound, <3 tetanus toxoid doses or unknown vaccine minutes, leading to an improvement in ptosis. The cold
status: Tetanus toxoid-containing vaccine temperature improves muscle strength by inhibiting the breakdown
Dirty/severe wound, ≥3 tetanus toxoid doses, last dose <5 years of acetylcholine at the neuromuscular junction, thereby increasing
ago: Nothing acetylcholine availability to the nicotinic receptor. Patients with
Dirty/severe wound, ≥3 tetanus toxoid doses, last dose ≥5 years positive test results should undergo confirmatory testing for MG
ago: Tetanus toxoid-containing vaccine with acetylcholine receptor antibodies (highly specific).
Dirty/severe wound, <3 tetanus toxoid doses or unknown vaccine
status: Tetanus toxoid-containing vaccine + tetanus immune
globulin

Patients with significant or dirty puncture wounds who have


received ≥3 tetanus toxoid doses but have not received
revaccination for tetanus within 5 years should be vaccinated.
Tetanus immune globulin (TIG), which provides immediate
passive (but temporary) immunity, is used to treat symptomatic
file:///C:/Users/vikas/OneDrive/Desktop/print.html 150/207
5/10/2021 print.html
tetanus. It is also recommended (in addition to tetanus vaccination)
for prophylaxis following severe or contaminated wounds in
patients who have not received at least 3 doses of tetanus vaccine
or whose vaccine status is uncertain. TIG is never administered
without tetanus vaccination.

What is the cause of choroidal rupture? What is the cause of clasp-knife rigidity? What is the cause of intention tremor?

Blunt ocular trauma Pyramidal tract disease (UMN) Cerebellar dysfunction (lateral hemisphere)

Choroidal rupture occurs due to blunt ocular trauma. Examination Typical features of cerebellar degeneration include progressive gait Typical features of cerebellar degeneration include progressive gait
reveals central scotoma, retinal edema, hemorrhagic detachment of dysfunction, truncal ataxia, nystagmus, intention tremor or dysfunction, truncal ataxia, nystagmus, intention tremor or
the macula, subretinal hemorrhage, and crescent-shaped streak dysmetria (limb-kinetic tremor when attempting to touch a target), dysmetria (limb-kinetic tremor when attempting to touch a target),
concentric to the optic nerve. The usual complaint is blurred vision and impaired rapid alternating movements (dysdiadochokinesia). and impaired rapid alternating movements (dysdiadochokinesia).
following blunt trauma. Muscle hypotonia can also be present, leading to a pendular knee Muscle hypotonia can also be present, leading to a pendular knee
reflex - persistent swinging movements of the limb after eliciting reflex - persistent swinging movements of the limb after eliciting
the deep-tendon reflex (more than 4 swings is considered the deep-tendon reflex (more than 4 swings is considered
abnormal). Pendular reflexes are not brisk, unlike clonus which abnormal). Pendular reflexes are not brisk, unlike clonus which
would suggest pyramidal tract disease. would suggest pyramidal tract disease.

The "clasp knife" phenomenon refers to a velocity-dependent The "clasp knife" phenomenon refers to a velocity-dependent
resistance to passive movement of the limb. It is seen in patients resistance to passive movement of the limb. It is seen in patients

file:///C:/Users/vikas/OneDrive/Desktop/print.html 151/207
5/10/2021 print.html
with hypertonia due to pyramidal tract disease and is not with hypertonia due to pyramidal tract disease and is not
characteristic of cerebellar disease. characteristic of cerebellar disease.

file:///C:/Users/vikas/OneDrive/Desktop/print.html 152/207
5/10/2021 print.html

What is the diagnostic study of choice for cavernous sinus


thrombosis?

MRI with magnetic resonance venography

1. Symptoms
1. Hyperesthesia or hypoesthesia in V1/V2 distribution
1. The bilateral forehead and mid-face may be
extremely tender to light palpation.
2. Binocular palsy
3. Low-grade fever
4. Periorbital edema usually occur several days later
secondary to impaired venous flow in the orbital
veins.
What is the definition of status epilepticus? 5. Vomiting is also a result of intracranial hypertension
6. Fundoscopy may reveal papilledema.
What is the cause of pendular knee reflex? 2. Pathogenesis
Single seizure lasting >5 minutes OR
A cluster of seizures with the patient not recovering a normal 1. Cranial nerves III, IV, V, and VI pass through the
Cerebellar dysfunction cavernous sinus, which has anastomoses crossing
mental status in between
midline. As a result, unilateral symptoms (eg,
Typical features of cerebellar degeneration include progressive gait headache, binocular palsies, periorbital edema,
Approximately 30% of patients with epilepsy will have status
dysfunction, truncal ataxia, nystagmus, intention tremor or hypoesthesia, or hyperesthesia in V1/V2 distribution)
epilepticus, especially those who are noncompliant with medical
dysmetria (limb-kinetic tremor when attempting to touch a target), can rapidly become bilateral
therapy. Status epilepticus has been defined historically as a single
and impaired rapid alternating movements (dysdiadochokinesia). 3. Diagnosis
seizure lasting >30 minutes. However, recent studies suggest that a
Muscle hypotonia can also be present, leading to a pendular knee 1. Magnetic resonance imaging with magnetic resonance
brain that has seized for >5 minutes is at increased risk of
reflex - persistent swinging movements of the limb after venography is the imaging modality of choice for
developing permanent injury due to excitatory cytotoxicity.
eliciting the deep-tendon reflex (more than 4 swings is diagnosis of CST
considered abnormal). Pendular reflexes are not brisk, unlike 4. Treatment
Cortical laminar necrosis is the hallmark of prolonged seizures and
clonus which would suggest pyramidal tract disease. 1. includes broad-spectrum intravenous antibiotics and
can lead to persistent neurologic deficits and recurrent seizures.
Magnetic resonance imaging of the brain will generally show prevention or reversal of cerebral herniation.
The "clasp knife" phenomenon refers to a velocity-dependent
evidence of cortical hyperintensity on diffusion-weighted imaging
resistance to passive movement of the limb. It is seen in patients
suggesting infarction. The definition of status epilepticus has
with hypertonia due to pyramidal tract disease and is not
therefore been amended to be any single seizure lasting >5 minutes
characteristic of cerebellar disease.
or a cluster of seizures with the patient not recovering a normal
mental status in between.

file:///C:/Users/vikas/OneDrive/Desktop/print.html 153/207
5/10/2021 print.html

What is the inital treatment regimen for subarachnoid What is the inital treatment regimen for subarachnoid
hemorrhage? hemorrhage?

Strict bedrest, acetaminophen for pain, IV fluids for hydration and Strict bedrest, acetaminophen for pain, IV fluids for hydration and
sodium control, anticoagulation reversal, BP control with β- sodium control, anticoagulation reversal, BP control with β-
blockers to prevent rebleeding, vasospasm prevention blockers to prevent rebleeding, vasospasm prevention
with nimodipine and hyperdynamic therapy, ICP control with nimodipine and hyperdynamic therapy, ICP control
with mannitol, and potential seizure prophylaxis with phenytoin. with mannitol, and potential seizure prophylaxis with phenytoin.

Complications of subarachnoid hemorrhage include rebleeding, Complications of subarachnoid hemorrhage include rebleeding,
vasospasm, communicating hydrocephalus, seizures, SIADH vasospasm, communicating hydrocephalus, seizures, SIADH

Hyperdynamic therapy is also referred to as triple-H therapy, this Hyperdynamic therapy is also referred to as triple-H therapy, this
strategy includes the use of hypertension, hypervolemia, and strategy includes the use of hypertension, hypervolemia, and
What is the first imaging study that you should obtain for a hemodilution to optimize cerebral perfusion. hemodilution to optimize cerebral perfusion.
stroke?
Although rebleeding risk can be high in untreated SAH, surgical Although rebleeding risk can be high in untreated SAH, surgical
repair of the aneurysm (eg, endovascular coiling in this patient) repair of the aneurysm (eg, endovascular coiling in this patient)
Non-contrast CT scan
practically eliminates the rebleeding risk. Rebleeding is the major practically eliminates the rebleeding risk. Rebleeding is the major
No contrast because hemorrhagic stroke hasn't been ruled out. CT cause of death within the first 24 hours of presentation, especially cause of death within the first 24 hours of presentation, especially
will reveal whether or not a bleed is present. MRI takes too long. within the first 6 hours of untreated SAH. Vasospasm can occur in within the first 6 hours of untreated SAH. Vasospasm can occur in
up to 30% of SAH patients from days 3-10 after presentation and up to 30% of SAH patients from days 3-10 after presentation and
is the major cause of delayed morbidity and death. It is likely is the major cause of delayed morbidity and death. It is likely
caused by arterial narrowing at the base of the brain due to caused by arterial narrowing at the base of the brain due to
degradation of the blood and its metabolites and can lead to degradation of the blood and its metabolites and can lead to
cerebral infarction. CT angiography is preferred for detecting cerebral infarction. CT angiography is preferred for detecting
vasospasm, which can best be prevented with initiation of vasospasm, which can best be prevented with initiation of
nimodipine. nimodipine.

file:///C:/Users/vikas/OneDrive/Desktop/print.html 154/207
5/10/2021 print.html

What is the major cause of death in subarachnoid hemorrhage?

Rebleeding (within first 24 hours)

While vasospasm can occur in up to 30% of SAH patients from


days 3-10 after presentation and is the major cause of delayed
morbidity and death. }}

Although rebleeding risk can be high in untreated SAH, surgical


repair of the aneurysm (eg, endovascular coiling in this patient)
practically eliminates the rebleeding risk. Rebleeding is the major
cause of death within the first 24 hours of presentation, especially
within the first 6 hours of untreated SAH. Vasospasm can occur in
up to 30% of SAH patients from days 3-10 after presentation and
is the major cause of delayed morbidity and death. It is likely
What is the initial step of management of hemorrhagic caused by arterial narrowing at the base of the brain due to
transformation following an ischemic stroke? degradation of the blood and its metabolites and can lead to
cerebral infarction. CT angiography is preferred for detecting
Repeat emergent noncontrast CT scan of the head vasospasm, which can best be prevented with initiation of
nimodipine.
Hemorrhagic transformation (HT) is a common complication of
ischemic stroke, especially when the stroke affects a large area (as What is the leading cause of neurologic disability?
in this patient), is due to an embolic cause, or has been treated with
thrombolytics. Manifestations typically arise <48 hours after the Stroke
initial event and are characterized by deterioration in the patient's
neurologic status. Emergent noncontrast CT scan of the head is Third most common cause of death in the US.
required to evaluate for HT as urgent surgical decompression is
often needed.

Intravenous mannitol is often used to reduce intracranial pressure.


Ischemic strokes may cause elevated intracranial pressure due to
HT or malignant cerebral edema.

file:///C:/Users/vikas/OneDrive/Desktop/print.html 155/207
5/10/2021 print.html

What is the most common affected CNS structure in diffuse


axonal injury?
What is the mechanism of action of the following anesthetics?
Corpus Callosum
Lidocaine: decreased depolarization of neurons in peripheral
nerves
Diffuse axonal injury is the result of traumatic
Capsaicin: loss of membrane potential in nociceptive fibers
acceleration/deceleration shearing forces that diffusely damage
axons in the brain. Patients typically have coma, and head CT scan
Capsaicin causes release and subsequent depletion of substance P,
may show diffuse small bleeds at the grey-white matter junction.
What is the management of multiple system atrophy? a polypeptide neurotransmitter involved in transmission of pain
signals. It causes excessive activation of TRPV1 (a transmembrane
Intravascular fluid expansion cation channel), causing a buildup of intracellular calcium that
results in long-lasting dysfunction of nociceptive nerve fibers
Anti-parkinson drugs are ineffective. (defunctionalization). On initial application, topical capsaicin
causes burning, stinging, and erythema, but persistent exposure
leads to a moderate reduction in pain over time.
Multiple system atrophy (Shy-Drager syndrome) is a degenerative
disease characterized by the following: parkinsonism; autonomic
dysfunction (postural hypotension, abnormal sweating, disturbance
of bowel or bladder control, abnormal salivation or lacrimation,
impotence, gastroparesis, etc.); and widespread neurological signs
(cerebellar, pyramidal or lower motor neuron).

Always consider Shy-Drager syndrome when a patient with


Parkinsonism experiences orthostatic hypotension, impotence,
incontinence, or other autonomic symptoms. The accompanying
bulbar dysfunction and laryngeal stridor may be fatal. Anti-
Parkinsonism drugs are generally ineffective, and treatment is
aimed at intravascular volume expansion with fludrocortisone, salt
supplementation, alpha-adrenergic agonists, and application of
constrictive garments to the lower body.

What is the most common cause of CNIII palsy in adults? What is the most common cause of acute neck pain? What is the most common cause of blindness worldwide?

Ischemic neuropathy due to poorly controlled diabetes mellitus Cervical strain Trachoma from Chlamydia trachomatis

CN III has 2 major components as follows: Usually self-limiting and resolves with time. Caused by serotypes A, B, C. Inclusion conjunctivitis results from
1. Inner somatic fibers - innervate the levator muscle of the eyelid serotypes D to K (caused by penis in the eye).
and 4 of the extraocular muscles (EOMs) (superior rectus, medial
rectus, inferior rectus, inferior oblique)
2. Superficial parasympathetic fibers - innervate the sphincter of
the iris and the ciliary muscles (controlling pupil dilation).

Because the inner somatic fibers are farther from the blood supply,
they are more susceptible to ischemic injury. Therefore, patients
with ischemic CN III palsy typically have paralysis of the levator
muscle (ptosis) and 4 EOMs ("down-and-out-gaze") with
preserved pupillary response.

Nerve compression (eg, aneurysm, tumor) of CN III can cause


file:///C:/Users/vikas/OneDrive/Desktop/print.html 156/207
5/10/2021 print.html
similar manifestations to ischemic palsy. However, nerve
compression usually also affects the parasympathetic fibers,
leading to an abnormal pupillary response (eg, mydriasis).

Isolated nerve inflammation (eg, vasculitis, autoimmune disorders)


of CN III is much less common than an ischemic pathology,
especially in patients with extensive vascular risk factors.

file:///C:/Users/vikas/OneDrive/Desktop/print.html 157/207
5/10/2021 print.html

What is the most common cause of corneal blindness in the


United States?

Herpes simplex virus

Ophthalmologic problems occur in approximately half of patients


with advanced HIV infection. Retinitis can occur as a complication
of opportunistic infections in AIDS patients. Both varicella-zoster
(VZV) and herpes simplex virus (HSV) can cause severe,
devastating intraocular inflammation. Most cases of retinal disease
are believed to be a reactivation of a previously acquired infection.

In an immunocompromised individual, HSV retinitis may be


characterized by rapidly progressing bilateral necrotizing retinitis
(referred to as the "acute retinal necrosis syndrome"). The initial
symptoms are keratitis and conjunctivitis with eye pain, followed
by rapidly progressive visual loss. Funduscopy reveals widespread,
What is the most common cause of intracranial hemorrhage in What is the most common cause of irreversible blindness in
pale, peripheral lesions and central necrosis of the retina.
children? African-Americans?
HSV infection of the eye is the most common cause of corneal
Arteriovenous malformation Glaucoma
blindness in the United States.

file:///C:/Users/vikas/OneDrive/Desktop/print.html 158/207
5/10/2021 print.html

What is the most common cause of lower back pain?

Muscle spasm

Since musculoskeletal pain is the most common cause of back


pain, finding reasons to look for deeper problems becomes critical.
Most patients do not require additional testing. Diagnostic testing
should be reserved for those with progressive neurologic deficits,
failure to improve after 6 weeks, and in those who you strongly
suspect an underlying diagnosis. The diagnostic step of choice
often begins with X-ray, though the MRI is always best. A CT scan
may be used in cases where the MRI is contraindicated
(Pacemaker).

What is the most common cause of organic dementia?

Alzheimer disease What is the most common cause of red eye?

Fourth most common cause of death in the US. Conjunctivitis

The key to ID more serious causes of red eye that require a referral
to an ophthalmologist, which are all intuitive: treatment-refractory
eye pain, sudden decrease in visual acuity, recent eye surgery,
trauma, corenal opacification/ulcer/foreign body that can't be
removed, chemical exposure, orbital cellulitis,
asymmetric/unreactive pupils

What is the most common cause of syncope? What is the most common cause of vision loss in people over 65 What is the most common cause of vitreous hemorrhage?
years of age in developed countries?
Vagovasal syncope aka neurocardiogenic syncope Diabetic retinopathy
Age-related macular degeneration
Neurocardiogenic syncope is the most common type of syncope Vitreous hemorrhage typically presents as a sudden loss of vision
and occurs due to excessive vagal tone. The excess tone causes a Macular degeneration is the leading cause of blindness in and onset of floaters. The most common cause is diabetic
transient profound hypotensive reaction and bradycardia due to industrialized countries. Macular degeneration is a painless, retinopathy. Ophthalmoscopy reveals loss of fundus details,
activation of the autonomic reflex. It is benign and self-limited in progressive loss of central vision that typically occurs in older floating debris and a dark red glow. An important diagnostic clue

file:///C:/Users/vikas/OneDrive/Desktop/print.html 159/207
5/10/2021 print.html
most cases, lasting about 10 seconds to a few minutes. patients, especially those with vascular risk factors. Both eyes are is that the fundus is hard to visualize, and even if it is visualized,
Neurocardiogenic syncope can be triggered by prolonged standing, often affected, and funduscopic abnormalities (eg, drusen [yellow details may be obscured. Immediate ophthalmologic consultation
emotional distress, or painful stimuli. Patients frequently deposits]) are prominent. One of the earliest findings in macular is required. For patients with underlying medical conditions,
experience dizziness, nausea, pallor, diaphoresis, abdominal pain, degeneration is distortion of straight lines such that they appear conservative treatment (i.e., upright position during sleep, which
and/or a generalized sense of warmth prior to the episodes. wavy. The grid test described above is frequently used to screen enhances settling of the hemorrhage) is
Patients with neurocardiogenic syncope usually have an excellent for patients with macular degeneration. The primary risk factor for recommended. Proliferative diabetic retinopathy in the initial stage
prognosis without any increase in morbidity or mortality. macular degeneration is increasing age, although smoking can is asymptomatic. Patients may later complain of decreased visual
increase the risk as well. Patients may be asymptomatic, but others acuity. Neovascularization is the hallmark of proliferative diabetic
complain of visual problems in either one or both eyes. Driving retinopathy. The other findings are vitreous hemorrhage and
and reading are often some of the first activities that are affected macular edema. These changes may lead to retinal detachment (the
since they require fine visual acuity, which is provided primarily separation of the inner layers of the retina), which is also
by the macula. In addition to the grid test described above, associated with metabolic disorders (e.g., diabetes mellitus),
ophthalmologic examination can identify drusen deposits in the trauma (including ocular surgery), vascular disease, myopia, or
macula, which are common lesions seen with this disorder. degeneration. Patients with retinal detachment typically complain
of vision loss, photopsia with showers of floaters. Fundoscopy in
Visualization of straight lines is a task that requires fine visual patients with retinal detachment typically demonstrates an elevated
acuity, which is controlled primarily by the centrally-located retina with folds and/or a tear.
macula as opposed to the more peripheral retina.

file:///C:/Users/vikas/OneDrive/Desktop/print.html 160/207
5/10/2021 print.html

file:///C:/Users/vikas/OneDrive/Desktop/print.html 161/207
5/10/2021 print.html

What is the most common form of conjunctivitis? What is the most common form of multiple sclerosis? What is the most common presentation of neurocysticercosis?

Viral conjunctivitis Relapsing-remitting Adult-onset seizure

Typically adenovirus; associated w/ recent history of URI. Multiple sclerosis (MS), an autoimmune inflammatory Systemic symptoms are usually absent, and laboratory studies are
Hyperemia of one or both eyes followed by spread to the other demyelinating disorder of the central nervous system (CNS), typically normal.
eye. Palpable preauricular lymph node may be present. typically presents with neurologic deficits disseminated in space
and time in women age 15-50. MS should be suspected in patients Taenia solium is a tapeworm transmitted to humans by the
with >2 distinctive episodes of CNS dysfunction with at least some ingestion of undercooked, contaminated pork. Tapeworms
resolution that cannot be explained by a single lesion. Symptoms typically remain localized in the gastrointestinal tract, but shed
occur over hours to days and then improve over weeks to months, eggs may pass to other individuals via the fecal-oral route.
although some may be permanent. Common initial symptoms may Ingested eggshatch in the small intestine, invade the bowel wall,
include: and spread hematogenously (brain, muscle, liver), causing
1. Fatigue: the most common non-specific symptom of MS. cysticercosis. Most patients are initially asymptomatic, but cysts
Patients may feel particularly fatigued after taking a hot shower or may degenerate over months or years causing inflammation,
after strenuous activity in heated environments. This is due to edema, and clinical illness.
decreased axonal transmission associated with increased heat. Heat
exposure also may lead to episodes of worsening neurological Neurocysticercosis typically manifests as an adult-onset seizure.
deficits (heat sensitivity) such as optic neuritis. Less commonly, patients develop signs of increased intracranial
2. Optic neuritis: monocular visual loss, painful eye movements, pressure (eg, vomiting, headache, papilledema) due to obstruction
and afferent pupillary defect of cerebral spinal fluid outflow. Systemic symptoms are usually
3. Transverse myelitis: motor and sensory loss below the level of absent, and laboratory studies are typically normal. Brain imaging
the lesion with bowel and bladder dysfunction. Patients initially (CT, MRI) usually reveals >1 cystic lesion with surrounding
have flaccid paralysis (spinal shock), followed by spastic paralysis contrast enhancement and edema (without displacement of
with hyperreflexia. adjacent tissue). Cysts at various stages of development
4. Internuclear ophthalmoplegia: demyelination of the medial (hypodense lesions without enhancement, calcified nodules) are
longitudinal fasciculus resulting in impaired conjugate horizontal also sometimes found. Diagnosis is typically made based on
gaze in which the affected eye (ipsilateral to the lesion) is unable clinical and radiologic findings; brain biopsy is rarely needed.
to adduct and the contralateral eye abducts with nystagmus Patients are treated with antiepileptics (eg, phenytoin),
5. Cerebellar dysfunction: intention tremor, ataxia, and nystagmus antiparasitics (eg,albendazole), and corticosteroids (for brain

Lumbar puncture is typically performed in patients with suspected


MS when the diagnosis is not completely clear. Cerebrospinal fluid
typically shows oligoclonal IgG bands. Once the diagnosis is
established, patients with an acute attack and disabling symptoms

file:///C:/Users/vikas/OneDrive/Desktop/print.html 162/207
5/10/2021 print.html
are typically treated with high-dose intravenous (IV) inflammation).
glucocorticoids (eg, methylprednisolone). Plasma exchange is
indicated in those who do not respond to glucocorticoids.

Interferon beta, natalizumab and glatiramer acetate are disease-


modifying agents used for chronic maintenance therapy as they
decrease the frequency of exacerbations in patients with relapsing-
remitting or secondary, progressive forms of MS. These agents
have no proven role in the treatment of acute exacerbations.

What is the most common site of ulnar nerve entrapment? What is the most common symptom of cavernous sinus What is the most common symptom of subclavian steal
thrombosis? syndrome?
The elbow where the ulnar nerve lies at the medial epicondylar
groove Headache Upper extremity claudication (pain, fatigue, paresthesias)

Ulnar nerve syndrome: decreased sensations over the 4th and 5th 1. Symptoms There is often a lower (>15 mg difference) systolic blood pressure
fingers and weak grip due to involvement of interosseous muscles 1. Low-grade fever in the affected arm with bruit.
of the hand. Prolonged, inadvertent compression of the nerve by
2. Periorbital edema usually occur several days later
leaning on the elbows while working at a desk or table is the secondary to impaired venous flow in the orbital In subclavian steal syndrome, significant stenosis or occlusion
typical scenario. Ulnar nerve compression can occur at the wrist, veins. leads to decreased pressure in the distal subclavian artery and
but is less common. The forearm is a rare site for the ulnar nerve 3. Vomiting is also a result of intracranial hypertension reversal ("steal") of blood flow in the ipsilateral vertebral artery.
involvement; it occurs mostly in diabetic patients. 4. Fundoscopy may reveal papilledema. The left subclavian artery is more commonly affected than the
2. Pathogenesis right, likely due to sharper curvature and more turbulent blood
Extremely high yield fact for USMLE!!! 1. Cranial nerves III, IV, V, and VI pass through the flow (leading to atherosclerosis).
cavernous sinus, which has anastomoses crossing
Most patients with subclavian artery stenosis are asymptomatic.
midline. As a result, unilateral symptoms (eg,
headache, binocular palsies, periorbital edema, When symptomatic, patients most commonly have symptoms of
hypoesthesia, or hyperesthesia in V1/V2 distribution) ischemia in the affected upper extremity (eg, pain, fatigue,
can rapidly become bilateral paresthesias). Less commonly, patients with concurrent
3. Diagnosis atherosclerosis of the circle of Willis may develop symptoms of
1. Magnetic resonance imaging with magnetic resonance vertebrobasilar ischemia (eg, dizziness, ataxia, dysequilibrium).
venography is the imaging modality of choice for Exercising the affected upper extremity causes arterial vasodilation
diagnosis of CST and a further lowering of distal pressure, which may exacerbate
vertebrobasilar symptoms. Physical examination often
4. Treatment
1. includes broad-spectrum intravenous antibiotics and demonstrates a significantly lower brachial systolic blood pressure
prevention or reversal of cerebral herniation. (eg, >15 mm Hg) in the affected arm and a systolic bruit in the
5. Vs Periorbital cellulitis: Not bilateral, inflammation does not supraclavicular fossa on the affected side. A fourth heart sound
extend beyond the oribtal septum. may be present due to left ventricular hypertrophy from systemic
hypertension.

file:///C:/Users/vikas/OneDrive/Desktop/print.html 163/207
5/10/2021 print.html

Diagnosis is typically made by Doppler ultrasound or magnetic


resonance angiography. Treatment involves lifestyle management
(eg, lipid-lowering interventions, smoking cessation) and
sometimes stent placement.

file:///C:/Users/vikas/OneDrive/Desktop/print.html 164/207
5/10/2021 print.html

What is the most common type of ischemic stroke?


What is the most commonly affected artery in an embolic stroke?
Thrombotic stroke
Middle cerebral artery
Ischemic strokes can be thrombotic or embolic in etiology, with
atherosclerotic-related thrombotic strokes the most common. Affects contralateral arms (and torso) and causes aphasia
However, the presence of an irregularly irregular rhythm in a What is the most common vector-borne illness in the US?
patient with a history of structural heart disease (severe left atrial
enlargement likely due to poorly controlled hypertension) is Lyme disease
strongly suggestive of a cardioembolic etiology in paroxysmal
atrial fibrillation. Embolic strokes present with a very rapid onset Peak incidence is in summer months in association with wooded
of severe focal neurologic deficits. In contrast, an unstable areas in Northeastern seaboard, midwest, and west coast.
ruptured atherosclerotic plaque with a propagating thrombus on its Incubation period is 3 to 32 days.
surface cause the development of focal neurologic symptoms at a
relatively slower rate.

file:///C:/Users/vikas/OneDrive/Desktop/print.html 165/207
5/10/2021 print.html

What is the most significant complication of pseudotumor


cerebri?

Blindness

Suspect benign intracranial hypertension (pseudotumor cerebri) in


a young obese female with a headache that is suggestive of a brain
tumor, but with normal neuroimaging and elevated CSF pressure.
Neurologic signs are usually absent, except for papilledema, visual
field defects and sometimes sixth (VI) nerve palsy. There may be a
history of exposure to provoking agents such as glucocorticoids or
vitamin A. Oral contraceptive pills have also been associated with
What is the preferred first-line management of acute sciatica?
this disorder. The pathology involves impaired absorption of CSF
by the arachnoid villi. The treatment includes weight reduction and
NSAIDs and acetaminophen
acetazolamide (if weight reduction fails). When medical
measures fail or visual field defects are progressive, shunting
What is the next step in management for acute pain in a patient Acute lumbosacral radiculopathy (sciatica) is most likely due to
or optic nerve sheath fenestration is done to prevent blindness,
with substance abuse history who has failed NSAIDs? nerve root compression by a herniated disc Older patients or those
which is the most significant complication of this otherwise
with prior traumatic injury can also have nerve root compression
benign disorder.
Intravenous morphine due to lumbar spondylosis. Less common causes include infectious
or inflammatory disorders, mass lesions, vascular disorders, and
Transdermal fentanyl and methadone are too slow in their developmental anomalies. Patients with uncomplicated lumbar
onset for acute pain. strain can have radiation of pain to the buttocks or posterior thigh,
but radiation to the calf and foot is more consistent with sciatica.
In patients with current or recovering opioid addiction, studies Traction on the nerve root during the straight leg raise test causes
suggest that providers may undertreat pain in such circumstances worsening or reproduction of pain.
due to fear of exacerbating the addiction. Initial management
options for acute pain (including opioids) are generally similar for Most patients with acute sciatica will experience spontaneous
all patients, regardless of substance abuse history. However, resolution; therefore, initial management is primarily focused on
patients with current or recovering opioid addiction who are given acute relief of symptoms. Nonsteroidal anti-inflammatory drugs
opioid analgesics may need close follow-up care to monitor pain and acetaminophen are the preferred first-line drugs. Short-term
relief and avoid addiction relapse. use of opioids or muscle relaxants can be considered in patients
with persistent pain but is associated with significant sedation.
Activity modification is often advisable, but patients should be
encouraged to maintain moderate physical activity. An x-ray
would not change initial management.

file:///C:/Users/vikas/OneDrive/Desktop/print.html 166/207
5/10/2021 print.html

What is the treatment of choice for Parkinson patients in which


levodopa-carbidopa has begun to wear off?

Selegiline, a(n) MAO-B inhibitor or capone, a(n) COMT inhibitor

Inhibit MAO-B and COMT, respectively, which prevents


breakdown of levadopa.

Other medical therapies: trihexypenidyl and benztropine are


useful in young patients with tremor only; amitripyline is useful
What is the presentation of tick-born paralysis? to oppose ACh and treat depression.
Progressive ascending paralysis over hours to days (similar to
GBS)

Patients usually present with progressive ascending paralysis over


hours to days. Paralysis may be localized or more pronounced in 1
What is the prognosis of diffuse axonal injury?
leg or arm. Fever is typically not present. Sensation is usually
normal. There is no autonomic dysfunction noted in these
Poor (often leads to persistent vegetative state)
patients, unlike that seen in the majority of patients with
Guillain Barré syndrome (GBS). The CSF examination is
Diffuse axonal injury is the most significant cause of morbidity in
typically normal. The etiology of the paralysis is neurotoxin
patients with traumatic brain injuries. It is frequently due to
release; the tick needs to feed for 4-7 days for the release of
traumatic deceleration injury and results in vegetative state.
neurotoxin. Meticulous search for ticks in these patients usually
Sudden acceleration-deceleration impact produces rotational forces
reveals a tick, and removal usually results in improvement within
that affect the brain areas where the density difference is the
an hour and complete recovery after several days.
maximum, thus most of the diffuse axonal injury occur at gray
white matter junction. Clinical features of patients with diffuse
GBS presents with an ascending symmetrical paralysis over days
axonal injury are out of proportion with the CT scan findings.
to weeks, but not usually hours. Sensation is usually normal to
Patient loses consciousness instantaneously and later develops
mildly abnormal. Autonomic dysfunction (eg, tachycardia, urinary
persistent vegetative state. CT scan characteristically shows
retention, and arrhythmias) occurs in 70% of patients. The CSF
numerous minute punctate hemorrhages with blurring of grey
examination is typically abnormal and may show
white interface. However, MRI is more sensitive than CT scan for
albuminocytologic dissociation (high protein with few cells). This
diagnosing diffuse axonal injury.
finding may not be present early in the course of the disease but is
present in 80-90% of patients at 1 week. Treatment includes IV
immunoglobulin or plasmapheresis. GBS can be difficult to
differentiate from tick paralysis in certain cases; however,
meticulous search for a tick is very easy to perform. If a tick is
found, extensive workup and unnecessary treatment can be
avoided.

What is the treatment of choice for Parkinson patients that are What is the treatment of choice for Parkinson patients that are What is the treatment of choice for Parkinson patients that don't
functional AND under 70 years old? non-functional OR over 70 years old? respond to medical therapy or develop severe disease before age
40?
Pramipexole, a(n) dopamine agonist Levodopa with carbidopa, a(n) dopamine precursor
Deep brain stimulation
Dopamine receptor agonist; ropinirole is an alternative. Delays Carbidopa inhibits peripheral degradation of levodopa. It decreases
need for L-dopa for several years. When symptoms compromise peripheral side effects but increases neuropsychiatric side effects.

file:///C:/Users/vikas/OneDrive/Desktop/print.html 167/207
5/10/2021 print.html
daily life, symptomatic therapy should be started. Agents available
for this purpose are levodopa/carbidopa, dopamine agonists, Chronic use (5-7 yrs) carries side effects of dyskinesias which
anticholinergics, and amantadine. Levodopa/carbidopa is the most delays drug initiation for as long as possible. Advanced
effective symptomatic therapy and has beneficial effects on all the disease show an “on–off” phenomenon (over the course of the
features of parkinsonism, although it does not stop progression of day) during treatment, which leads to fluctuations in
the disorder. The most common early side effects include symptoms. When symptoms compromise daily life, symptomatic
hallucinations, confusion, agitation, dizziness, somnolence, and therapy should be started. Agents available for this purpose are
nausea. Involuntary movements (eg, dyskinesia, dystonia) levodopa/carbidopa, dopamine agonists, anticholinergics, and
typically occur after 5–10 years of therapy in nearly 50% of amantadine. Levodopa/carbidopa is the most effective
patients taking levodopa/carbidopa. As a result, it is important to symptomatic therapy and has beneficial effects on all the features
find the lowest therapeutic dose and closely monitor of parkinsonism, although it does not stop progression of the
patients. Choreiform dyskinesia is a later side effect of disorder. The most common early side effects include
levodopa/carbidopa but is more commonly seen earlier with hallucinations, confusion, agitation, dizziness, somnolence, and
catechol-O-methyltransferase inhibitors (eg, entacapone and nausea. Involuntary movements (eg, dyskinesia, dystonia)
tolcapone). Livedo reticularis, a mottled vascular pattern on the typically occur after 5–10 years of therapy in nearly 50% of
lower extremities, is a side effect more commonly seen with patients taking levodopa/carbidopa. As a result, it is important to
amantadine and is not usually seen with find the lowest therapeutic dose and closely monitor
levodopa/carbidopa. Urinary retention is more commonly seen patients. Choreiform dyskinesia is a later side effect of
with anticholinergics (eg, benztropine), which improve tremor and levodopa/carbidopa but is more commonly seen earlier with
rigidity but do not have much effect on bradykinesia. catechol-O-methyltransferase inhibitors (eg, entacapone and
tolcapone). Livedo reticularis, a mottled vascular pattern on the
lower extremities, is a side effect more commonly seen with
amantadine and is not usually seen with
levodopa/carbidopa. Urinary retention is more commonly seen
with anticholinergics (eg, benztropine), which improve tremor and
rigidity but do not have much effect on bradykinesia.

file:///C:/Users/vikas/OneDrive/Desktop/print.html 168/207
5/10/2021 print.html

file:///C:/Users/vikas/OneDrive/Desktop/print.html 169/207
5/10/2021 print.html

What is the treatment plan for multiple sclerosis? What is the treatment plan for multiple sclerosis? What is the treatment plan for myasthenia gravis?

Acute flares: high-dose IV corticosteroids (first-line) Acute flares: high-dose IV corticosteroids (first-line) Symptomatic: pyridostigmine, which ↑[Ach]
or plasmapheresis (second-line) or plasmapheresis (second-line) Acute: IVIG plasma
Chronic management: interferon β (first-line), glatiramer (first- Chronic management: interferon β (first-line), glatiramer (first- Refractory: prednisone
line), or natalizumab (second-line) line), or natalizumab (second-line) Surgery: thymectomy
Symptom - spasticity: baclofen, dantrolene, or physiotherapy Symptom - spasticity: baclofen, dantrolene, or physiotherapy
Symptom - painful parasthesias: carbamazepine or amitriptyline Symptom - painful parasthesias: carbamazepine or amitriptyline

file:///C:/Users/vikas/OneDrive/Desktop/print.html 170/207
5/10/2021 print.html
Symptom - urinary retention: intermittent catheterization and Symptom - urinary retention: intermittent catheterization and
parsympathomimetics parsympathomimetics
Symptom - urinary incontinence: parasympatholytics Symptom - urinary incontinence: parasympatholytics

Dantrolene can also be used for muscle spasticity. Treat depression Dantrolene can also be used for muscle spasticity. Treat depression
if indicated. MS typically presents in women age 15-50 with if indicated. MS typically presents in women age 15-50 with
neurologic deficits (eg, ataxia, diplopia, vertigo, pronator drift) neurologic deficits (eg, ataxia, diplopia, vertigo, pronator drift)
disseminated in space and time. MRI of the brain often disseminated in space and time. MRI of the brain often
demonstrates multifocal, ovoid hyperintense white matter lesions demonstrates multifocal, ovoid hyperintense white matter lesions
(arrows) in the CNS, particularly the periventricular and subpial (arrows) in the CNS, particularly the periventricular and subpial
white matter of the cerebrum (corpus callosum) and the optic white matter of the cerebrum (corpus callosum) and the optic
nerves, brainstem, and spinal cord. Lumbar puncture is typically nerves, brainstem, and spinal cord. Lumbar puncture is typically
performed in patients with suspected MS when the diagnosis is not performed in patients with suspected MS when the diagnosis is not
completely clear. Cerebrospinal fluid typically shows oligoclonal completely clear. Cerebrospinal fluid typically shows oligoclonal
IgG bands. It is not necessary in this patient with classic clinical IgG bands. It is not necessary in this patient with classic clinical
and radiographic findings. Once the diagnosis is established, and radiographic findings. Once the diagnosis is established,
patients with an acute attack and disabling symptoms are typically patients with an acute attack and disabling symptoms are typically
treated with high-dose intravenous (IV) glucocorticoids (eg, treated with high-dose intravenous (IV) glucocorticoids (eg,
methylprednisolone). Plasma exchange is indicated in those who methylprednisolone). Plasma exchange is indicated in those who
do not respond to glucocorticoids. Interferon beta, natalizumab do not respond to glucocorticoids. Interferon beta, natalizumab
and glatiramer acetate are disease-modifying agents used for and glatiramer acetate are disease-modifying agents used for
chronic maintenance therapy as they decrease the frequency of chronic maintenance therapy as they decrease the frequency of
exacerbations in patients with relapsing-remitting or secondary, exacerbations in patients with relapsing-remitting or secondary,
progressive forms of MS. These agents have no proven role in the progressive forms of MS. These agents have no proven role in the
treatment of acute exacerbations. treatment of acute exacerbations.

file:///C:/Users/vikas/OneDrive/Desktop/print.html 171/207
5/10/2021 print.html

file:///C:/Users/vikas/OneDrive/Desktop/print.html 172/207
5/10/2021 print.html

What is the workup for spinal cord compression? What is treatment for babesiosis? What is treatment for contact lens-associated keratitis?

1. Emergency MRI Atovaquone + azithromycin Topical broad spectrum antibiotics


2. Intravenous glucocorticoids
3. Neurosurgery consult Babesiosis is a tick-borne protozoal illness endemic to the Most cases of contact lens-associated keratitis are due to Gram-
northeastern United States. Human transmission occurs via Ixodes negative organisms such as Pseudomonas and Serratia, but can
Epidural spinal cord compression must be suspected in any patient scapularis approximately 48-72 hours after attachment. I scapularis also be due to Gram-positive organisms as well as certain fungi
with a history of malignancy who develops back pain with motor also may transmit Borrelia burgdorferi (Lyme disease) and and amoebas. Contact lens-associated keratitis is a medical
and sensory abnormalities. The thoracic (60%) and lumbosacral Anaplasma phagocytophilum (human granulocytic anaplasmosis); emergency and can lead to corneal perforation, scarring, and
(30%) spine are the most common locations for ESCC. Early therefore, coinfection with multiple organisms is possible. Peak permanent vision loss if not addressed promptly. In addition to
diagnosis of ESCC is crucial in preserving neurologic function; in prevalence is in July and August, when tick populations are at their removal and discarding of the contact lens, most patients require
fact, the primary determinant of outcome is the degree of highest. topical broad-spectrum antibiotics.
neurologic involvement at the time of intervention. Pain is
typically the first symptom, often present for 1-2 months before Infection is often asymptomatic or mild, but patients with Patients using extended-wear contact lenses are at risk for both
additional symptoms appear. Bowel and bladder dysfunction are immunocompromise, age >50, or a history of splenectomy are at bacterial conjunctivitis and keratitis. Involvement of the cornea
late neurologic findings. Intravenous glucocorticoids should be higher risk for severe illness. Symptoms typically include the indicates keratitis, and the cornea is generally spared in
given without delay. Glucocorticoids decrease vasogenic edema gradual onset of fatigue, malaise, weakness, chills, and fever. The uncomplicated conjunctivitis.
(caused by obstructed epidural venous plexus), help alleviate organism multiplies in red blood cells, so patients may develop
pain, and may restore neurologic function. MRI is then anemia with signs of intravascular hemolysis (jaundice, dark urine,
recommended. indirect hyperbilirubinemia, reticulocytosis, elevations of
aminotransferases and lactate dehydrogenase). Thrombocytopenia
is common and mild hepatosplenomegaly may occur. Diagnosis is
made by identifying organisms on peripheral blood smear
("Maltese cross"). Treatment includes 7-10 days of atovaquone
plus azithromycin or quinine plus clindamycin (for severe illness).
Symptoms may take up to 3 months to fully resolve.

file:///C:/Users/vikas/OneDrive/Desktop/print.html 173/207
5/10/2021 print.html

What is treatment for metastasis to the brain? What is treatment for postherpetic neuralgia? What is treatment for retinal detachment?

Single brain metastasis: surgical resection or stereotactic Tricyclic antidepressants, pregabalin, gabapentin Laser therapy or cryotherapy to create permanent adhesions
radiosurgery (gamma knife; surgically inaccessible site) between the retina, epithelium and choroid
Multiple brain metastases: whole brain radiation therapy vs. trigeminal neuralgia requires carbamazepine/oxcarbamazepine.
Retinal detachment refers to the separation of the layers of the
retina. It usually occurs in people aged 40-70 years. Patients
file:///C:/Users/vikas/OneDrive/Desktop/print.html 174/207
5/10/2021 print.html
For patients with a single brain metastasis in a surgically complain of photopsia (flashes of light) and floaters (spots in the
accessible location and good performance status, surgical resection visual field). The most classic description is that of "a curtain
offers the best therapeutic option. This is typically followed by coming down over my eyes." Usually, the inciting event occurs
stereotactic radiosurgery (SRS) or whole brain radiation therapy months before retinal detachment. Myopia or trauma can cause
(WBRT) to the tumor bed. SRS may be used as an alternate retinal breaks, through which fluid seeps in and separates the
therapeutic option for patients who are not candidates for surgery, retinal layers. Ophthalmoscopic examination reveals a grey,
have a surgically inaccessible lesion, or smaller metastasis (<3 elevated retina. Laser therapy and cryotherapy are done to create
cm). For patients with multiple brain metastases or poor permanent adhesions between the neurosensory retina, retinal
performance status, WBRT or supportive care is typically pigment epithelium, and choroid.
recommended.

Metastatic brain tumors are the most common type of intracranial


brain tumors. They may be solitary or multiple and are usually
seen at the grey-white matter junction. Brain metastasis is
frequently seen in patients with non-small cell lung carcinoma
(NSCLC). Favorable survival outcomes after diagnosis of brain
metastases include age <65 years, good functional performance
status, and stable extracranial disease. Aggressive treatment is
typically indicated in such patients to improve survival and control
central nervous system disease.

file:///C:/Users/vikas/OneDrive/Desktop/print.html 175/207
5/10/2021 print.html

What is treatment for severe central retinal vein occlussion?

Intravitreal injection of vascular endothelial growth factor


inhibitors (bevacizumab)
What is treatment of CMV retinitis that presents with lesions near
the fovea or optic nerve? What is treatment of Morton neuroma?
The characteristic changes on funduscopic examination are
sometimes referred to as the "blood and thunder" appearance, and
include optic disk swelling, retinal hemorrhages, dilated veins, and Intravitreal injections of valganciclovir Metatarsal support with a bar or padded shoe inserts to decrease
cotton wool spots. CRVO should be considered in the differential pressure on the metatarsal heads
diagnosis for acute or subacute monocular loss of vision, but it is Oral antivirals are indicated in more mild disease.
typically not quite as acute as the vision loss seen in patients with Interdigital (Morton) neuroma commonly occurs in runners. The
central retinal artery occlusion. Cytomegalovirus (CMV) is a widely prevalent DNA virus of the disorder is not a true neuroma but is a mechanically induced
herpes family that typically causes an asymptomatic initial neuropathic degeneration of the interdigital nerves that causes
infection followed by a life-long latent infection. In patients with numbness, aching, and burning in the distal forefoot from the
CRVO is caused by thrombosis of the central retinal vein and is
most common in patients with coagulopathy, hyperviscosity, significantly compromised cell-mediated immunity (eg, advanced metatarsal heads to the third and fourth toes. The symptoms are
chronic glaucoma, and atherosclerotic risk factors (eg, age, HIV with CD4 count <100/mm3), CMV reactivation may result in worsened by walking on hard surfaces and wearing tight or high-
diabetes, hypertension). The diagnosis can be confirmed with viremia or end-organ disease. heeled shoes.
fluorescein angiography. Patients with no significant macular
edema or neovascularization are often managed conservatively CMV retinitis is characterized by full-thickness retinal The diagnosis of Morton neuroma is primarily made clinically. On
with close observation. Significant macular edema can be inflammation that moves centripetally along the vasculature, examination, squeezing the metatarsal joints will cause pain on the
treated with intravitreal injection of vascular endothelial causing edema and scarring. Patients experience blurred vision, plantar surface of the foot along with crepitus between the third
floaters, and photopsia (sensation of flashing lights). Lesions near and fourth toes (Mulder sign). Treatment involves metatarsal
growth factor inhibitors. No treatment is particularly effective,
but some patients may have partial recovery of vision within the the fovea and optic nerve may cause blindness, and scarring support with a bar or padded shoe inserts to decrease pressure on
first 3 months. greatly increases the risk of retinal detachment. Diagnosis is made the metatarsal heads. Surgery is usually reserved for patients who
by fundoscopy showing yellow-white, fluffy, hemorrhagic fail conservative treatment.
lesions along the vasculature. Blood tests for CMV (polymerase
chain reaction) are not sufficient for diagnosis as viremia may
develop independently of end-organ disease. Patients are usually
treated with oral antivirals (eg, valganciclovir); if lesions are near
the fovea or optic nerve, intravitreal injections are added. All
patients should be initiated on antiretroviral therapy (usually 2
weeks after beginning CMV treatment) to prevent recurrence and
progression.

What is treatment of Mycobacterium leprae? What is treatment of lyme disease with the following? What kind of acid-base disturbance is seen after a tonic-clonic
seizure?
Dapsone + rifampin +/- clofazimine [if severe ("multibacillary")] Bell's palsy: Oral antibiotic

file:///C:/Users/vikas/OneDrive/Desktop/print.html 176/207
5/10/2021 print.html
Leprosy is a chronic granulomatous disease of the skin and Joint infection: Oral antibiotic High anion gap metabolic acidosis
peripheral nerves caused by the acid-fast bacillus Mycobacterium
leprae. Transmission is thought to occur via respiratory droplets, All other neurologic manifestations require IV ceftriaxone Postictal lactic acidosis commonly occurs following a tonic-clonic
although cases are occasionally linked to close contact with a nine- seizure. It is a transient anion gap metabolic acidosis that resolves
banded armadillo. Infections are rare in the United States and Treatment of early localized Lyme: Clinical trials have shown without treatment within 90 minutes following resolution of
occur primarily in immigrants or travelers to endemic regions (eg, that oral doxycycline, amoxicillin, and cefuroxime have equivalent seizure activity. Seizure activity, especially a tonic-clonic seizure,
Asia, Africa, South America). efficacy for treating early localized Lyme disease. Doxycycline is can significantly raise serum lactic acid levels due to skeletal
often used because it is effective in treating potential coexisting muscle hypoxia and impaired hepatic lactic acid uptake. The most
Manifestations include >1 chronic, anesthetic, macular (often Anaplasma phagocytophilum infection, which is also transmitted appropriate management is observation and a repeat chemistry
hypopigmented) skin lesions with raised, well-demarcated borders. by the Ixodes tick. However, doxycycline is contraindicated in panel after approximately 2 hours. If the metabolic acidosis has not
Nearby nerves often become nodular and tender, and segmental children age <8 years and pregnant women. It can slow bone resolved, other potential causes of anion gap metabolic acidosis
demyelination may result in loss of sensation and motor function. growth in exposed fetuses and cause enamel hypoplasia and (eg, intoxication, sepsis) should be investigated.
Diagnosis is clinical in endemic regions, but in the United States permanent teeth stains during tooth development in young
patients usually require a full-thickness biopsy of the skin lesion children. Therefore, oral amoxicillin or cefuroxime is the treatment
edge (as M leprae is not culturable). Patients with minimal lesions of choice for children age <8 years and pregnant women.
("paucibacillary") are treated with dapsone and rifampin; those
with extensive lesions ("multibacillary") require the addition of Treatment of early or late disseminated Lyme: intravenous
clofazimine. Lesions often take months or years to heal ceftriaxone is reserved for Lyme meningitis and heart block,
completely. manifestations of early disseminated Lyme disease.

Lyme arthritis is the hallmark of late Lyme disease. Lyme disease


is caused by Borrelia burgdorferi, a spirochete carried by the deer
tick (Ixodes scapularis), and causes disseminated infection by
hematogenous spread and migration into various tissues.

Early symptoms (eg, erythema migrans, fever) can be overlooked


or misdiagnosed if history of travel to endemic areas is not
obtained. Untreated early localized disease can progress to an
inflammatory monoarticular or asymmetric oligoarticular arthritis,
most commonly involving the knee. Synovial fluid shows an
inflammatory profile with an average leukocyte count of
25,000/mm3; Gram stain and culture are usually negative, although
polymerase chain reaction (done for investigative purposes) may
demonstrate B burgdorferi DNA. The diagnosis can be confirmed
with serum ELISA and Western blot testing. Oral doxycycline or
amoxicillin is first-line treatment for Lyme arthritis in the absence
of concurrent neurologic manifestations.

file:///C:/Users/vikas/OneDrive/Desktop/print.html 177/207
5/10/2021 print.html

file:///C:/Users/vikas/OneDrive/Desktop/print.html 178/207
5/10/2021 print.html

What kind of imaging modality will patients with seizure disorder What kind of pain warrants a referral for a supervised exercise What kind of sensation is affected in anterior spinal cord
receive on follow-up? program? syndrome?

MRI Persistent pain for 6-12 weeks or longer Loss of pain/temperature

However, CT without contrast is the initial imaging study of Anterior spinal cord syndrome typically presents with sudden
choice in patients with unprovoked first seizure to exclude acute flaccid paralysis (spinal shock) and loss of pain/temperature
neurologic problems (eg, intracranial or subarachnoid bleed) that sensation below the level of spinal injury. Autonomic dysfunction
might require urgent intervention. may also occur due to interruption of descending autonomic tracts.
The absence of risk factors (eg, aortic surgery or dissection), lower
extremity symptoms, or autonomic dysfunction in this patient
makes the diagnosis unlikely.

file:///C:/Users/vikas/OneDrive/Desktop/print.html 179/207
5/10/2021 print.html

What kind of stroke is this? What kinds of lesions does pronator drift test for? What make a lumbar puncture unsafe?

file:///C:/Users/vikas/OneDrive/Desktop/print.html 180/207
5/10/2021 print.html

Cardioembolic 1. UMN Focal neurologic deficit


2. Pyramidal/corticospinal tract disease Altered mental status
Immunosuppression
Pronator drift is a physical examination finding that is relatively Lesion
sensitive and specific for upper motor neuron or Seizure
pyramidal/corticospinal tract disease. The pronator drift test is
particularly useful in patients with subtle deficits as it can
accentuate pyramidal motor weakness. It is performed by having
the patient outstretch the arms with the palms up and eyes closed
(so that only proprioception is used to maintain arm position).
Upper motor neuron lesions cause more weakness in the supinator
muscles compared to the pronator muscles of the upper limb. As a
result, the affected arm drifts downward and the palm turns
(pronates) toward the floor.

Cardioembolic stroke can occur in the setting of atrial fibrillation,


especially while patients are off anticoagulants; however, they
typically have sudden-onset neurologic deficits that follow a
stuttering course. In addition, neuroimaging classically shows
multiple ischemic infarctions at the grey-white matter junction.

file:///C:/Users/vikas/OneDrive/Desktop/print.html 181/207
5/10/2021 print.html

What metastatic brain cancer typically reveals a periventricular


lesion? What percent of adults over age 75 have cataracts?

Primary central nervous system lymphoma (a Non-Hodgkin 50%


What neurocutaneous disorder is this image associated with?
lymphoma seen in immunocompromised patients)
Cataract is a vision-impairing disease characterized by progressive
von-Hippel Landau thickening of the lens. Oxidative damage of the lens occurs with
Lung cancer presents with multiple well-circumscribed lesions
with vasogenic edema at the gray and white matter junction. Breast aging and leads to cataract formation. Patients usually complain of
cancer presents with single solitary lesion. blurred vision, problems with nighttime driving, and glare.
Definitive treatment is lens extraction.

Risk factors: old age, smoking, steroid use, UV exposure, trauma,


diabetes, Wilson disease, Down syndrome, metabolic dz

Cavernous hemangiomas, also known as cavernous malformations,


consist of dilated vascular spaces with thin-walled endothelial
cells. They present as soft blue, compressible masses growing up
to a few centimeters. Cavernous hemangiomas may appear on the
skin, mucosa, deep tissues, and viscera. Cavernous hemangiomas
of the brain and viscera are associated with von Hippel-Lindau
disease.

file:///C:/Users/vikas/OneDrive/Desktop/print.html 182/207
5/10/2021 print.html
What radiographic finding is associated with idiopathic What syndrome is characterized by a CNS lesion that causes What type of CNS injury is associated with status epilepticus?
intracranial hypertension? hyperphagia and hypersexuality?
Cortical laminar necrosis
Empty sella Kluver-Bucy syndrome
Approximately 30% of patients with epilepsy will have status
Patients with IIH typically present with holocranial headache, Kluver-Bucy Syndrome is a CNS lesion that results from bilateral epilepticus, especially those who are noncompliant with medical
vision changes (blurry vision and diplopia), and pulsatile tinnitus amygdala lesion and presents with hyperorality, therapy. Status epilepticus has been defined historically as a single
("whooshing" sound in the ears). Although IIH is frequently seen hypersexuality and disinhibited behaviour. In contrast, Wernicke seizure lasting >30 minutes. However, recent studies suggest that a
in young obese women, it has also been associated with certain encephalopathy affects the mammillary body. brain that has seized for >5 minutes is at increased risk of
medications (eg, isotretinoin, tetracyclines, growth hormone, developing permanent injury due to excitatory cytotoxicity.
excessive vitamin A).
Cortical laminar necrosis is the hallmark of prolonged seizures and
Initial evaluation of IIH includes complete ocular examination and can lead to persistent neurologic deficits and recurrent seizures.
neuroimaging to exclude secondary causes of intracranial Magnetic resonance imaging of the brain will generally show
hypertension (eg, mass, hemorrhage, cerebral vein thrombosis). evidence of cortical hyperintensity on diffusion-weighted
Magnetic resonance imaging possibly with magnetic resonance imaging suggesting infarction. The definition of status
venography (to rule out cerebral vein thrombosis) is the preferred epilepticus has therefore been amended to be any single seizure
imaging modality. Empty sella is present in about 70% of lasting >5 minutes or a cluster of seizures with the patient not
patients but is not diagnostic. Lumbar puncture (LP) is then recovering a normal mental status in between.
indicated to document elevated opening pressure.

Papilledema is not a contraindication to LP unless the patient has


evidence of obstructive or noncommunicating hydrocephalus
and/or a space-occupying lesion with/without mass effect or
midline shift. Pseudotumor cerebri causes a communicating
hydrocephalus (ie, pressures in the ventricular and subarachnoid
spaces are equilibrated with the lumbar cistern) and therefore LP is
considered safe. LP is performed with the patient in the lateral
decubitus position with legs extended. An opening pressure of
>250 mm H2O is considered abnormal and in the appropriate
clinical setting is diagnostic of IIH. Cerebrospinal fluid studies are
normal in IIH patients.

The treatment includes weight reduction and acetazolamide (if


weight reduction fails). When medical measures fail or visual field
defects are progressive, shunting or optic nerve sheath fenestration
is done to prevent blindness, which is the most significant
complication of this otherwise benign disorder

file:///C:/Users/vikas/OneDrive/Desktop/print.html 183/207
5/10/2021 print.html

What type of arthritis is associated with carpal tunnel syndrome? What type of imaging should be used in back pain suspected to be What type of stroke has the highest mortality rate?
caused by the following?
Rheumatoid arthritis Hemorrhagic stroke
Suspected malignancy: X-ray → MRI
Compression of the median nerve causes carpal tunnel syndrome Sensory/motor deficits or cauda equina syndrome: MRI Intracerebral hemorrhage carries a 50% mortality rate at 30 days.
(CTS), characterized by pain and paresthesias in the first 3 digits Epidural abscess/infection: MRI For those that survive, there is significant morbidity. Subarachnoid
and the radial half of the fourth. These symptoms are often worse Needs MRI, but contraindicated: radionuclide bone scan hemorrhage carries a 40-50% mortality rate at 30 days.
at night. Motor involvement in severe cases can cause weakness of Muliple myeloma: skeletal survey
thumb abduction and opposition, and atrophy of the thenar
eminence. The diagnosis is usually made on clinical grounds. Most patients with acute low back pain have a benign etiology and
During examination, tapping over the median nerve at the wrist will have spontaneous resolution within the first few weeks.
(Tinel sign) or holding the wrists in extreme flexion with the Imaging for uncomplicated low back pain does not improve
dorsum of the hands pressed together (Phalen test) can often outcomes and is not recommended. However, patients at risk for
reproduce the symptoms. However, if the diagnosis is uncertain (or infection, malignancy, or bony abnormalities (eg, compression
severe symptoms prompt consideration for surgery), CTS can be fracture) may require more aggressive or specific intervention and
confirmed with nerve conduction studies, which will show slowing warrant early imaging. These red flag features include the
in the median nerve at the wrist. X-ray is not routinely advised in following: age >50; history of cancer; constitutional symptoms
the evaluation of CTS. It may be considered in those with a history (eg, fever, unexplained weight loss); nocturnal pain; no response to
of significant trauma or evidence of advanced osteoarthritis (eg, appropriate treatment; significant or progressive neurologic
decreased range of motion, Heberden and Bouchard nodes). deficits.

Plain film x-rays can be performed quickly and can identify signs
of malignant disease, infection, or vertebral compression.
Inflammatory markers (eg, erythrocyte sedimentation rate [ESR],
C-reactive protein) can increase sensitivity and are also advised. If
the ESR or x-rays are abnormal, MRI should be done to evaluate
for possible cancer or spinal infection. MRI is also indicated for
patients with significant neurologic deficits or features of cauda
equina syndrome (eg, saddle anesthesia, urine retention).
Radionuclide bone scan can be considered for patients who are not
able to have an MRI. For patients in whom multiple myeloma is
file:///C:/Users/vikas/OneDrive/Desktop/print.html 184/207
5/10/2021 print.html
suspected (eg, bone pain, anemia, hypercalcemia, kidney injury),
skeletal surveys (including x-rays of chest, spine, skull, and pelvis)
are preferred to a bone scan as they better detect the lytic lesions
seen in multiple myeloma. Surgical intervention is indicated for
patients with cauda equina syndrome, spinal cord compression,
progressive neurologic deficits, or persistent pain after
conservative therapy.

file:///C:/Users/vikas/OneDrive/Desktop/print.html 185/207
5/10/2021 print.html

What type of stroke is associated with illicit drug use (e.g., What vitamin can cause idiopathic intracranial hypertension? When do you provide empiric antibiotic treatment for bacterial
amphetamines, cocaine)? meningitis?
Vitamin A and its derivatives, e.g., isotretinoin, all trans retinoic
Intracerebral hemorrhage acid LP is safe: After LP
LP is unsafe: After blood cultures, before CT scan
Patients with IIH typically present with holocranial headache,
vision changes (blurry vision and diplopia), and pulsatile tinnitus
("whooshing" sound in the ears). Although IIH is frequently seen
in young obese women, it has also been associated with certain
medications (eg, isotretinoin, tetracyclines, growth hormone,
excessive vitamin A).

Initial evaluation of IIH includes complete ocular examination and


neuroimaging to exclude secondary causes of intracranial
hypertension (eg, mass, hemorrhage, cerebral vein thrombosis).
Magnetic resonance imaging possibly with magnetic resonance
venography (to rule out cerebral vein thrombosis) is the preferred
imaging modality. Empty sella is present in about 70% of patients
but is not diagnostic. Lumbar puncture (LP) is then indicated to
document elevated opening pressure.

Papilledema is not a contraindication to LP unless the patient has


evidence of obstructive or noncommunicating hydrocephalus
and/or a space-occupying lesion with/without mass effect or
midline shift. Pseudotumor cerebri causes a communicating
hydrocephalus (ie, pressures in the ventricular and subarachnoid
spaces are equilibrated with the lumbar cistern) and therefore LP is
considered safe. LP is performed with the patient in the lateral
decubitus position with legs extended. An opening pressure of
>250 mm H2O is considered abnormal and in the appropriate
clinical setting is diagnostic of IIH. Cerebrospinal fluid studies are

file:///C:/Users/vikas/OneDrive/Desktop/print.html 186/207
5/10/2021 print.html
normal in IIH patients.

When is a carotid endarterectomy recommended in men? When is a carotid endarterectomy recommended in women? When is imaging necessary for the evaluation of lower back pain
during the first four to six weeks?
1. Symptomatic (stroke, TIA) men with 70-99% stenosis Symptomatic or asymptomatic (less convincing) women with 70-
2. Asymptomatic men with 80-99% stenosis 99% stenosis Progressive neurologic deficits, disabling symptoms,
or constitutional symptoms
Asymptomatic carotid artery stenosis can progress to transient Asymptomatic carotid artery stenosis can progress to transient History of osteoporosis or malignancy
ischemic attack (TIA) or embolic stroke. Symptomatic carotid ischemic attack (TIA) or embolic stroke. Symptomatic carotid Prolonged steroid use or IV drug use, and/or
stenosis is defined by the occurrence (within the past 6 months) of stenosis is defined by the occurrence (within the past 6 months) of Recent trauma
sudden-onset focal neurological symptoms corresponding to a sudden-onset focal neurological symptoms corresponding to a
carotid artery lesion. carotid artery lesion. The big fear is cord compression, which is a medical
emergency. Most patients with acute low back pain have a benign
Carotid endarterectomy (CEA) is recommended for men and Carotid endarterectomy (CEA) is recommended for men and etiology and will have spontaneous resolution within the first few
women with symptomatic carotid stenosis of 70%-99% and is also women with symptomatic carotid stenosis of 70%-99% and is also weeks. Imaging for uncomplicated low back pain does not improve
beneficial for men with asymptomatic carotid stenosis of beneficial for men with asymptomatic carotid stenosis of outcomes and is not recommended. However, patients at risk for
60%-99%. Some experts recommend CEA for asymptomatic 60%-99%. Some experts recommend CEA for asymptomatic infection, malignancy, or bony abnormalities (eg, compression
women with high-grade stenosis (70%-99%), but the data are less women with high-grade stenosis (70%-99%), but the data are less fracture) may require more aggressive or specific intervention and
convincing. CEA can be considered in men with symptomatic convincing. CEA can be considered in men with symptomatic warrant early imaging. These red flag features include the
carotid lesions of 50%-69%. Patients are selected for surgery based carotid lesions of 50%-69%. Patients are selected for surgery based following: age >50; history of cancer; constitutional symptoms
on their individual surgical risk and predicted survival, the on their individual surgical risk and predicted survival, the (eg, fever, unexplained weight loss); nocturnal pain; no response to
presence of a surgically accessible carotid lesion, and the presence of a surgically accessible carotid lesion, and the appropriate treatment; significant or progressive neurologic
experience of the surgeon/center. However, men with persistently experience of the surgeon/center. deficits.
disabling neurologic deficits, 100% occlusion of the carotid
artery, or life expectancy <5 years are unlikely to benefit. Lesions <50% are typically monitored with annual Duplex

file:///C:/Users/vikas/OneDrive/Desktop/print.html 187/207
5/10/2021 print.html
ultrasound. Surgery is not indicated at this stage, and patients are Plain film x-rays can be performed quickly and can identify signs
Lesions <50% are typically monitored with annual Duplex medically managed with pharmacotherapy (eg, antiplatelet agents, of malignant disease, infection, or vertebral compression.
ultrasound. Surgery is not indicated at this stage, and patients are statins) and riskfactor optimization (eg, tobacco cessation, diabetes Inflammatory markers (eg, erythrocyte sedimentation rate [ESR],
medically managed with pharmacotherapy (eg, antiplatelet agents, mellitus and hypertension control). C-reactive protein) can increase sensitivity and are also advised. If
statins) and riskfactor optimization (eg, tobacco cessation, diabetes the ESR or x-rays are abnormal, MRI should be done to evaluate
mellitus and hypertension control). Aspirin is recommended for the prevention of cardiovascular for possible cancer or spinal infection. MRI is also indicated for
disease (eg, coronary heart disease, cerebrovascular disease, patients with significant neurologic deficits or features of cauda
Aspirin is recommended for the prevention of cardiovascular peripheral artery disease). In patients with existing cardiovascular equina syndrome (eg, saddle anesthesia, urine retention).
disease (eg, coronary heart disease, cerebrovascular disease, disease (such as this patient), aspirin reduces the risk of a Radionuclide bone scan can be considered for patients who are not
peripheral artery disease). In patients with existing cardiovascular cardiovascular event and, in the case of carotid artery stenosis, able to have an MRI. For patients in whom multiple myeloma is
disease (such as this patient), aspirin reduces the risk of a helps prevent progression of the lesion. suspected (eg, bone pain, anemia, hypercalcemia, kidney injury),
cardiovascular event and, in the case of carotid artery stenosis, skeletal surveys (including x-rays of chest, spine, skull, and pelvis)
helps prevent progression of the lesion. are preferred to a bone scan as they better detect the lytic lesions
seen in multiple myeloma. Surgical intervention is indicated for
patients with cauda equina syndrome, spinal cord compression,
progressive neurologic deficits, or persistent pain after
conservative therapy.

file:///C:/Users/vikas/OneDrive/Desktop/print.html 188/207
5/10/2021 print.html

file:///C:/Users/vikas/OneDrive/Desktop/print.html 189/207
5/10/2021 print.html

When syncope presents suddenly without warning, what the most


When syncope presents suddenly upon exertion, what the most
likely cause?
likely cause?
Arrythmia
Mechanical obstruction to blood flow
Could also be neurogenic, but that's very rare.
When is the meningococcal vaccination administered?

Primary: 11-12 years old


Booster: 16-21 years old, if primary vaccination <16 years old

Meningococcal vaccination is also recommended before travel to


highly endemic environments, such as most of sub-Saharan Africa
and the Muslim hajj pilgrimage to Mecca, Saudi Arabia.

In addition, an increased risk of meningococcal meningitis is found


among young adults in large groups living in close proximity, such
as military recruits and first-year college students residing in
dormitories (although the details of this patient's arrangements are
unknown).

HIV itself is not an indication for meningococcal vaccination,


which would be recommended only if the patient were at otherwise
increased risk (eg, asplenia, complement deficiency).

Where in the cerebellum are pilocystic astrocytomas found? Which antibiotic is associated with idiopathic intracranial Which antibiotics exacerbate myasthenia gravis?
hypertension?
Both vermis and lateral (therfore will cause limb dysmetria) 1. Fluoroquinolones
Tetracyclines 2. Aminoglycosides
The vast majority of medulloblastomas occur in the cerebellar
vermis (red circle), which is particularly important for balance and Patients with IIH typically present with holocranial headache, Myasthenia gravis is a neuromuscular disorder caused by
gait coordination. As a result, symptoms include truncal or gait vision changes (blurry vision and diplopia), and pulsatile tinnitus autoantibodies (originating in the thymus) directed against
instability. Given the proximity to the fourth ventricle, ("whooshing" sound in the ears). Although IIH is frequently seen nicotinic acetylcholine receptors at the neuromuscular junction.
medulloblastoma can also causeobstructive hydrocephalus (red in young obese women, it has also been associated with certain Patients classically experience fluctuating and fatigable weakness
arrow), resulting in signs of increasedintracranial pressure (eg, medications (eg, isotretinoin, tetracyclines, growth hormone, involving the ocular (eg, diplopia, ptosis) and/or bulbar (eg,
headache, vomiting). Less commonly, medulloblastomas occur in excessive vitamin A). dysphagia, dysarthria) muscles. Individuals with typical clinical
the lateral cerebellar hemispheres similar to pilocytic features of myasthenia gravis should undergo confirmatory
astrocytomas, which affect fine motor planning and cause Initial evaluation of IIH includes complete ocular examination and immunologic testing for acetycholine receptor antibodies (highly
dysmetria, intention tremor, and dysdiadochokinesia. Limb neuroimaging to exclude secondary causes of intracranial specific). Those with negative acetycholine receptor antibodies
dysmetria (overshoot/undershoot during targeted movement) is hypertension (eg, mass, hemorrhage, cerebral vein thrombosis). should subsequently be checked for muscle-specific tyrosine
usually associated with lesions affecting the cerebellar Magnetic resonance imaging possibly with magnetic resonance kinase antibodies. If the diagnosis remains unclear,
hemispheres due to involvement of the lateral descending motor venography (to rule out cerebral vein thrombosis) is the preferred electrophysiologic studies (eg, repetitive nerve stimulation, single-
(eg, lateral corticospinal tract, rubrospinal tract) pathways. Lateral imaging modality. Empty sella is present in about 70% of patients fiber electromyography) may be helpful. Patients with established
emispheric cerebellar lesions may also cause tremor during goal- but is not diagnostic. Lumbar puncture (LP) is then indicated to myasthenia gravis should receive chest imaging (eg, CT scan or
directed activity (intention tremor). document elevated opening pressure. MRI) to evaluate for thymoma as thymectomy can lead to long-

file:///C:/Users/vikas/OneDrive/Desktop/print.html 190/207
5/10/2021 print.html
term improvement. The edrophonium (Tensilon) test may be used
Papilledema is not a contraindication to LP unless the patient has to support the diagnosis of myasthenia gravis. However, it is less
evidence of obstructive or noncommunicating hydrocephalus specific than the acetylcholine receptor antibody assay and is
and/or a space-occupying lesion with/without mass effect or difficult to interpret in patients with only minor symptoms. In this
midline shift. Pseudotumor cerebri causes a communicating bedside study, the infusion of edrophonium (a short-acting
hydrocephalus (ie, pressures in the ventricular and subarachnoid acetylcholinesterase inhibitor) prolongs the presence of
spaces are equilibrated with the lumbar cistern) and therefore LP is acetylcholine in the neuromuscular junction, resulting in an
considered safe. LP is performed with the patient in the lateral immediate improvement in muscle strength.
decubitus position with legs extended. An opening pressure of
>250 mm H2O is considered abnormal and in the appropriate
clinical setting is diagnostic of IIH. Cerebrospinal fluid studies are
normal in IIH patients.

file:///C:/Users/vikas/OneDrive/Desktop/print.html 191/207
5/10/2021 print.html

Which aphasia is characterized by disturbances in all areas of Which aphasia is characterized by good comprehension, but
language function? broken speech?
Which aphasia is characterized by disturbances in repetition?
Global aphasia Broca aphasia
Conduction aphasia
Often associated with a right hemiparesis. Often associated with a right hemiparesis and hemisensory loss.
Aphasia is caused by stroke (#1), trauma, tumor, or Alzheimer
disease affecting the dominant hemisphere. Most patients
Aphasia is caused by stroke (#1), trauma, tumor, or Alzheimer Aphasia is caused by stroke (#1), trauma, tumor, or Alzheimer
spontaneously recover or improve within the first month. Speech
disease affecting the dominant hemisphere. Most patients disease affecting the dominant hemisphere. Most patients
therapy is helpful, but is unlikely to be of much benefit after the
spontaneously recover or improve within the first month. Speech spontaneously recover or improve within the first month. Speech
first few months.
therapy is helpful, but is unlikely to be of much benefit after the therapy is helpful, but is unlikely to be of much benefit after the
first few months. first few months.

Which aphasia is characterized by impaired comprehension, but Which back pain typically worsens with lying down? Which bacteria commonly cause acute bacterial meningitis in
grammatically correct speech? elderly adults (50+ years)?
Vertebral compression fracture

file:///C:/Users/vikas/OneDrive/Desktop/print.html 192/207
5/10/2021 print.html
Wernicke aphasia Patients with gradual onset VCF can be asymptomatic. However, Streptococcus pneumoniae (most common), Neisseria
an acute VCF can present with sudden onset of low back pain and meningitidis, Listeria monocytogenes, and Escherichia coli (and
Aphasia is caused by stroke (#1), trauma, tumor, or Alzheimer decreased spinal mobility after bending, coughing, or lifting. The other gram-negative rods)
disease affecting the dominant hemisphere. Most patients pain typically increases with standing, walking, or lying on the
spontaneously recover or improve within the first month. Speech back. Examination can show tenderness at the affected level. Each
therapy is helpful, but is unlikely to be of much benefit after the VCF can decrease a patient's height by ≥1 cm and lead to
first few months. kyphosis, which is associated with decreased respiratory capacity
and increased risk for atelectasis pneumonia. Kyphosis can also
cause a protuberant abdomen, early satiety, and weight
loss. VCF is associated with a significantly increased risk for
future vertebral and nonvertebral fractures.

file:///C:/Users/vikas/OneDrive/Desktop/print.html 193/207
5/10/2021 print.html

Which bacteria commonly cause acute bacterial meningitis in Which bacteria commonly cause acute bacterial meningitis in Which bacteria commonly cause acute bacterial meningitis in
immunocompromised individuals? infants (1 month - 2 years)? newborns?

Listeria monocytogenes, gram-negative bacilli, Streptococcus Neisseria meningitidis (most common), Streptococcus Group B Streptococcus (most common), Escherichia coli (and
pneumoniae, Neisseria meningitidis, and Cryptococcus spp. pneumoniae (most common), Listeria monocytogenes, other gram-negative rods), Listeria monocytogenes,
(particularly in HIV patients) and Haemophilus influenzae (only in unvaccinated children). and Hamophilus influenzae

file:///C:/Users/vikas/OneDrive/Desktop/print.html 194/207
5/10/2021 print.html

file:///C:/Users/vikas/OneDrive/Desktop/print.html 195/207
5/10/2021 print.html

file:///C:/Users/vikas/OneDrive/Desktop/print.html 196/207
5/10/2021 print.html

Which dermatome is associated with the following:


Which etiologic agent(s) cause painful retinopathy with lesions
Nipple line: T4 in a nonvascular distribution?
Lateral aspect of the lower leg: L5 (anterior), S1 (posterior)
Lateral aspect of the foot: S1 Toxoplasmic chorioretinitis, HSV
Medial aspect of the foot: L4
Herpes simplex keratitis causes corneal (not retinal) lesions and
results in ocular pain, tearing, and discharge. Patients with
toxoplasmic chorioretinitis present with eye pain and decreased
vision. Retinal lesions appear in a nonvascular distribution.
Which cause of red eye is associated with dendritic ulcers on the
cornea and can result in irreversible vision loss?

Herpes simplex keratitis

Caused by HSV-1, usually unilateral; presents with ocular irritation


and photophobia. No pain. Treat w/ topical antiviral eye drops or
ganciclovir gel.

file:///C:/Users/vikas/OneDrive/Desktop/print.html 197/207
5/10/2021 print.html

Which form of syncope is associated with a generalized sense of


warmth prior to episodes?

Neurocardiogenic or vasovagal syncope

Neurocardiogenic syncope is the most common type of syncope


and occurs due to excessive vagal tone. The excess tone causes a
transient profound hypotensive reaction and bradycardia due to
activation of the autonomic reflex. It is benign and self-limited in
most cases, lasting about 10 seconds to a few minutes.
Neurocardiogenic syncope can be triggered by prolonged standing, Which head injury is characterized by brief loss of consciousness,
followed by a lucid interval and progressive worsening over Which nerve provides sensation to the cornea?
emotional distress, or painful stimuli. Patients frequently
experience dizziness, nausea, pallor, diaphoresis, abdominal pain, hours?
Trigeminal nerve
and/or a generalized sense of warmth prior to the episodes.
Patients with neurocardiogenic syncope usually have an excellent Epidural hematoma
prognosis without any increase in morbidity or mortality.
In contrast, subdural hematoma presents with confusion and
headache occuring gradually over 1-2 days.

Which neurofibromatosis has an autosomal dominant mode of Which neuromuscular junction disorder is characterized by Which neuromuscular junction disorder is characterized by
inheritance? antibody-induced complement activation and receptor decreased acetylcholine release from the presynaptic terminal of
degradation? motor neurons?
Both ( ͡~ ͜ʖ ͡°)
Myasthenia gravis Foodborne botulism

Myasthenia gravis (MG), a neuromuscular disorder, usually Foodborne botulism is caused by decreased acetylcholine release
presents in the second to third decade in women and in the sixth to from the presynaptic terminal of motor neurons, which leads to the
eighth decade in men. Patients typically have fluctuating, fatigable acute onset of bilateral cranial neuropathies and symmetric
muscle weakness that worsens with repetitive motions of the same descending flaccid paralysis. Muscle weakness is usually preceded
muscle groups and improves with rest. Ocular symptoms (eg, by a prodrome of gastrointestinal symptoms (eg, nausea/vomiting,
ptosis, diplopia) and bulbar dysfunction are the most common diarrhea).
manifestations, the latter of which may lead to fatigable chewing,
dysphagia with nasopharyngeal regurgitation, and dysarthria.

file:///C:/Users/vikas/OneDrive/Desktop/print.html 198/207
5/10/2021 print.html
MG is caused by autoantibodies (originating in the thymus)
directed against nicotinic acetylcholine receptors at the
neuromuscular junction. Antibody binding leads to complement
activation with receptor degradation, causing impaired action
potential propagation and muscle weakness. Most patients with
acetylcholine receptor antibodies have thymic abnormalities (eg,
thymoma, thymic hyperplasia), which may appear as an anterior
mediastinal mass on chest imaging. Patients with MG typically
receive a CT scan of the chest to evaluate for thymoma as
thymectomy can result in long-term disease remission.

file:///C:/Users/vikas/OneDrive/Desktop/print.html 199/207
5/10/2021 print.html

Which organisms are typically transmitted soon after an Ixodes Which organisms commonly cause acute meningitis in adults (18 Which organisms commonly cause acute meningitis in children
scapularis bite? years - 50 years) and teens (2 years - 18 years)

Anaplasmosis and babesiosis Streptococcus pneumoniae (most common), Neisseria Neisseria meningitidis (most common), Streptococcus
meningitidis, Listeria monocytogenes, Enteroviruses (particularly pneumoniae, Listeria monocytogenes, Enteroviruses (particularly
Ixodes scapularis is endemic to the northeastern United States and coxsackievirus and echovirus), and HSV-2 coxsackievirus and echovirus), and HSV-2 (in older children and
serves as a reservoir for several zoonotic infections, including teens)
Lyme disease (Borrelia burgdorferi), anaplasmosis, and babesiosis.
Transmission occurs when these organisms pass from the salivary
glands of the tick into the bite wound. Although anaplasmosis and
babesiosis are typically transmitted soon after tick attachment, B
burgdorferi resides in the gut of the tick and requires 48-72
hours of feeding before salivary gland migration. As such,
patients with a tick attached for <36 hours are extremely unlikely
to acquire Lyme disease and do not require antimicrobial
prophylaxis. Ticks should be removed with tweezers as close to the
skin surface as possible.

There is no need for antimicrobial prophylaxis (doxycycline) when


visiting Lyme endemic areas. Prophylaxis is reserved for patients

file:///C:/Users/vikas/OneDrive/Desktop/print.html 200/207
5/10/2021 print.html
who have Ixodes scapularis attachment for >36 hours.

file:///C:/Users/vikas/OneDrive/Desktop/print.html 201/207
5/10/2021 print.html

Which parkinsonism drug causes the following side effects? Which polio vaccine is alive? Which position worsens the pain in spinal cord compression?

Ankle edema: amantadine Sabin (oral) Recumbent position at night


Dyskinesia, early: -capones
Dyskinesia, late: carbidopa-levodopa Epidural spinal cord compression must be suspected in any patient
Insomnia, elderly: selegeline with a history of malignancy who develops back pain with motor
Livedo reticularis: amantadine and sensory abnormalities. Bowel and bladder dysfunction are late
neurologic findings. Intravenous glucocorticoids should be given
When symptoms compromise daily life, symptomatic therapy without delay. MRI is then recommended.
should be started. Agents available for this purpose are
levodopa/carbidopa, dopamine agonists, anticholinergics, and
amantadine. Levodopa/carbidopa is the most effective
symptomatic therapy and has beneficial effects on all the features
of parkinsonism, although it does not stop progression of the
disorder. The most common early side effects include
hallucinations, confusion, agitation, dizziness, somnolence, and
nausea. Involuntary movements (eg, dyskinesia, dystonia)
typically occur after 5–10 years of therapy in nearly 50% of
patients taking levodopa/carbidopa. As a result, it is important to
find the lowest therapeutic dose and closely monitor
patients. Choreiform dyskinesia is a later side effect of
levodopa/carbidopa but is more commonly seen earlier with
catechol-O-methyltransferase inhibitors (eg, entacapone and

file:///C:/Users/vikas/OneDrive/Desktop/print.html 202/207
5/10/2021 print.html
tolcapone). Livedo reticularis, a mottled vascular pattern on the
lower extremities, is a side effect more commonly seen with
amantadine and is not usually seen with
levodopa/carbidopa. Urinary retention is more commonly seen
with anticholinergics (eg, benztropine), which improve tremor and
rigidity but do not have much effect on bradykinesia.

Which tremor increases steadily as the hand reaches its target? Which tremor tends to involve the head? Which type of dementia has severe neuroleptic sensitivity to
antipsychotics, thus warranting the avoidance of FGAs in
Cerebellar Essential (and cerebellar) delirium?

Lewy-Body dementia

file:///C:/Users/vikas/OneDrive/Desktop/print.html 203/207
5/10/2021 print.html
Resting tremor may progress to involve other extremities, jaw, Pharmacotherapy includes cholinesterase inhibitors. Neuroleptic
face, tongue, and lips. Unlike essential and cerebellar tremors, refers to a drug that depresses nerve functions; a major
resting tremor does not usually involve the entire head. tranquilizer. The treatment of choice for agitation in the elderly is
low-dose haloperidol. Atypical antipsychotics (quetiapine,
risperidone) may also be used. Usage of typical and atypical
antipsychotics is safe in the acute setting, but prolonged use can
increase mortality in the elderly. Typical antipsychotics should
not be used in patients with Lewy body dementia, who may
exhibit neuroleptic hypersensitivity (severe parkinsonism and
impaired consciousness with neuroleptic
administration). Lorazepam and other benzodiazepines may be
used to treat agitation in young patients. However, they are
typically contraindicated in older patients, who are at increased
risk for adverse events (eg, withdrawal, dependence, motor
impairment), may experience worsening agitation (paradoxic
effect), and tend to metabolize benzodiazepines slowly, making
their effects very long-lasting.

file:///C:/Users/vikas/OneDrive/Desktop/print.html 204/207
5/10/2021 print.html

Which virus causes retinal hemorrhagic lesions that follow


along the vasculaure?

CMV

Cytomegalovirus (CMV) is a widely prevalent DNA virus of the Amaurosis fugax refers to sudden, transient (<10 minutes) loss
herpes family that typically causes an asymptomatic initial of vision in one or both eyes due to cardiovascular etiologies
infection followed by a life-long latent infection. In patients with requiring a carotic ultrasound and cardiac workup.
significantly compromised cell-mediated immunity (eg, advanced
HIV with CD4 count <100/mm3), CMV reactivation may result in Why does orthostatic hypotension normally develop with Essentially the TIA-equivalent to a central retinal artery occlusion
increasing age?
viremia or end-organ disease. (eye stroke).

CMV retinitis is characterized by full-thickness retinal 1. Decreased baroreceptor sensitivity The description of a curtain descending over the visual field is
inflammation that moves centripetally along the vasculature, 2. Arterial stiffness highly suggestive of amaurosis fugax. The most common etiology
causing edema and scarring. Patients experience blurred vision, 3. Reduced sensitivity of the myocardium to sympathetic is retinal ischemia due to atherosclerotic emboli originating from
floaters, and photopsia (sensation of flashing lights). Lesions near stimulation the ipsilateral carotid artery; therefore, patients with vascular risk
the fovea and optic nerve may cause blindness, and scarring 4. Decrease NE content of sympathetic nerve endings factors (eg, hypertension, hyperlipidemia, smoking) should be
greatly increases the risk of retinal detachment. Diagnosis is made evaluated with a duplex ultrasound of the neck. Funduscopic
Orthostatic hypotension is defined as a postural decrease in blood
by fundoscopy showing yellow-white, fluffy, hemorrhagic examination is often normal but may show embolic plaques and
lesions along the vasculature. Blood tests for CMV (polymerase pressure by 20 mmHg systolic or 10 mmHg diastolic (sometimes retinal whitening (due to ischemia). Patients with amaurosis fugax
chain reaction) are not sufficient for diagnosis as viremia may accompanied by an increase in heart rate) that occurs on standing. and concomitant carotid artery disease have an increased risk of
develop independently of end-organ disease. Patients are usually In general, orthostatic hypotension results from insufficient stroke. Other causes of transient monocular vision loss are
treated with oral antivirals (eg, valganciclovir); if lesions are near constriction of resistance and capacitance blood vessels in the uncommon and include cardioembolic disease, giant cell arteritis,
the fovea or optic nerve, intravitreal injections are added. All lower extremities on standing, which may be due to a defect in retinal vein occlusion, retinal vasospasm, and papilledema.
patients should be initiated on antiretroviral therapy (usually 2 autonomic reflexes, decreased intravascular volume, or
weeks after beginning CMV treatment) to prevent recurrence and medications. Some baroreceptor sensitivity is lost as a normal part
of aging. Arterial stiffness, decreased norepinephrine content of
progression.
sympathetic nerve endings, and reduced sensitivity of the
myocardium to sympathetic stimulation all contribute to a
tendency toward orthostatic hypotension with age.

Normal changes to the aging heart include decreased resting and


maximal cardiac output, decreased maximum heart rate, increased
contraction and relaxation time of heart muscle, increased
myocardial stiffness during diastole, decreased myocyte number,
and pigment accumulation in myocardial cells.

file:///C:/Users/vikas/OneDrive/Desktop/print.html 205/207
5/10/2021 print.html

Vertebrobasilar insuffiency presents as a sudden, unprovoked


syncope + focal neurologic deficit. Diagnose with a CT
angiogram.
Delayed sleep phase syndrome is when you go to bed too late,
while advanced sleep phase syndrome is when you go to bed too Tension headaches present with a vice-like pain that radiates to
early. the neck and are muscular in origin.
Delayed sleep phase syndrome, a circadian rhythm sleep-wake
disorder, is characterized by sleep-onset insomnia and excessive
morning sleepiness. Circadian rhythm sleep-wake disorders occur
when the internal clock regulating sleep and wakefulness is
misaligned with a person's desired sleep time or social/professional
schedule. Patients with delayed sleep phase syndrome commonly
describe themselves as "night owls" and have chronic problems
going to sleep at a societally accepted time (ie, prior to midnight).
They sleep normally when allowed to set their own schedule. The
onset of the condition occurs in adolescence and may respond to
treatments such as light or behavioral therapy.

Advanced sleep phase syndrome is another circadian rhythm


disorder characterized by the inability to stay awake in the evening
(usually after 7:00 PM), making social functioning difficult. These
patients frequently have early-morning insomnia due to their early
bedtime.

Shift work sleep disorder involves a recurrent pattern of sleep


interruption due to shift work, causing difficulty in initiating and
maintaining sleep and daytime sleepiness. This disorder is due to a
work schedule that is incongruent with a normal circadian clock.
This patient has an abnormal internal circadian rhythm that has
created difficulties throughout her life (ie, past jobs and schooling
requiring early start times).

Reagin(IgE mediated) disease is seen in vernal(allergic)


Drusen/yellow deposits in retina occurs in macular degeneration Drusen/yellow deposits in retina occurs in macular degeneration
conjunctivitis)

file:///C:/Users/vikas/OneDrive/Desktop/print.html 206/207
5/10/2021 print.html

Resting tremor may involve the face but does not involve the
entire head.

Resting tremor may progress to involve other extremities, jaw, Syringomyelia presents with bilateral loss of pain/temp over the
face, tongue, and lips. Unlike essential and cerebellar tremors, shoulders in a "cape-like" distribution.
resting tremor does not usually involve the entire head.
Most often asymptomatic. Touch preserved. Thoracic scoliosis and
muscle atrophy of hands may occur.

Reagin(IgE mediated) disease is seen in vernal(allergic)


conjunctivitis)

Syringomyelia presents with bilateral loss of pain/temp over the


shoulders in a "cape-like" distribution.

Most often asymptomatic. Touch preserved. Thoracic scoliosis and


muscle atrophy of hands may occur.

file:///C:/Users/vikas/OneDrive/Desktop/print.html 207/207

You might also like